PDA

View Full Version : Just updated 12.04 with recommended updates... massive meltdown



myrrdinoftheforest
January 23rd, 2013, 12:17 AM
So I had a perfectly good running system this morning, now that I installed the recommended updates from Update Manager, my ATI driver is completely messed up (won't let me purge it, or reinstall it). Also, it says my package manager is broken :(

Any ideas? anyone else run into this after today’s update?

***UPDATE***
So I figured out so far that it's got something to do with a dependancy issue with my ATI Radeon graphics driver.

This is what I get when I try opening the software centre:
http://ubuntuforums.org/data:image/png;base64,iVBORw0KGgoAAAANSUhEUgAAAYQAAAIqCAIAAAC rIaqHAAAAA3NCSVQICAjb4U/gAAAgAElEQVR4XuxdB2AURfffq+md9F4gFdKAhA7SUZqIVAX1s 4CKDT//gqJ+VkQQOxakKCpIEYEgHeklkJCE9N57vVzu9vZu/29278Ilt3eXQAhJmCEkd7NT3vxm9rdv3szO40kamggcMAIYAYz AvUaAf68FwPVjBDACGAGEACYjPA4wAhiBHoEAJqMe0Q1YCIwAR kDYDoIp0yZgUDACGAGMQJcj8E/cccNl8loN2CwNubm5sBl42vl4t761iYc06Ls6rm2W1vxamXWKZ rJo5dN81BfVvnbIy5FFUzVTs04W5qq6Yq6L+opUZ9EU3lZsnYL UETqNv5Vd3yVDUmtKbS2F/aDTzlvi6FxSN54rngWmfTvbNvgWOu1kQF9p9B/90g1MLM2kaB9uxba9qs7Smlzrqr6C9FaPKtcjGSsxl2BtSmuTA H3RibgV174wTfL28ahp6qZwX2LariP3rbRG5OCSqK3YXI1gKlW 3T3NdK12bj23F1lc2G19aWg7FGqAkNRkBE9nZ2aqlwH8wAhgBj MDdQaCurl4fHyEyAiaytcVMdHewx6ViBDACbRGor+fmo1abUVt 9C8OHEcAIYAS6FwHeyBExNjY23Vsprg0jgBG4rxFoaGjQnayxm hFWi+7rkYEbjxHoCQggMtKx1vcEwbAMGAGMwP2FANaM7q/+xq3FCPRYBDqnGcFWDZpWgSbFfDA+uYMksGuGTcn8hh/YxoJ+QwBQ4DebRhug9rmYTJCcR/D5Aj5k4fP5UBpbQo9FFguGEcAIdAqBjmpGDPmoVEqaohQUpVQC IzF0AOQgFArgH2KLtgElYOaAkBGyqCATBJoGOgE24QsEkBFejk PZEGepsyMmQjGoOu5cIqEA5YcXWdSM1qkG48QYAYxAz0SgQ5oR www0kJBcLp8V4DbU3d7H1srZ0qxC0pJf33SlpPav7FITExNgl9 ZGQg7gCvitpCiSJH18fNzd3Rwc+sHWStj1VFNTXVJSmp+fLxaL IRfSj9RaEqIilEtJkXLSzc0tIMDf1s7OytJSIpHU1zfk5OQUFx dDLiGiJAFSl3RIsGcCjaXCCGAEDCOAlvYtLS0MJGKIiFaQpKNY sHZ0qJulCa1SEqDiqFSg4cDMiccXlErkb5y5WSlXisQiKIqZwC EGUygUIpF47Jgx9va2oBgxuhHF5wsFQCQCfm1t/el//1UoSJFIxHAKqxKpFAoKmGbEiJHOzk6QoV2u6uqas2fPkQrSRAR EBnzUXiMz0BZ8CSOAEegJCEgkzbpL+4iMLCwMkxFiBzcx/4uxIWa0QkXKdBvDF5u28ETPn0yppAghox8Bf8F8Dlhm0qSJAiF PQSqY6ZrazMSDqRrBA+ZSUvSRI0dh6icUiSCO4S9gKx7kEpuI2 Fzo7R2G3lpzUQrVP/8cAZoCpgNS05UHx2AEMAI9GYHmZg4yEnh5eYgZdUZfAB1IQcrX x/pYU82UpEGlkOv+KGXNQloZ7Wj9V361CJERTLRUcpIcM2aUSCSQ y0mlVkCWIyYAB4Hpx9nZMTs7VwQMxAN9SwUzwVGjR5qZmujPRQ EBubg4QS6kYGEy0tdzOB4j0FMRgDnT4kWPt5POiM0INBKKoqa6 WDjRUkV9PWSulJLVLYoQB7UylVkntRQJYO6mbGlysbSd6mp5rJ oEM5BCqQQjkampqVTaojZHI+M1UnFYQw/QEMywQA8yMzN3c3erLK8EJYeiVC4uLpYWFkwu4tSpf6H2dhJPn DgeWqLOVVEppG8Zqnoq8lgujABGwDgCRlbTgDhg6WyMoxnV3Ki iFFBehUy54VrRaxGuIfbmmfUt6xJKX4jycjFFlyDNWEezwxUts FAGhh5XN9eWlhZQieASY3diqIgJaBkMBTTDAm3Izc21rLQMPkN iOMNEJkO52LQTJz6gnevUqTPARCAVFOjh7sbkUpfJlox/YwQwAr0UASOaEVh2wIDsacpTNUvBYg2NDHOyfX36mHUHTs/xsd2X3/DStFEDeY2qpjqGVpQeFmaQHuVSKUHBIUk5xLeyCfsZSEgTo4TP sHBmYW4OlASsArnAgAWrb61ZgM5ac7EQQ3r4AGxlzuTS8FsvxR +LjRHACKgRMKIZwTYg4AVztJiGaAWCqr4q0ErxyNCBW88lLh4+ KFRVSzXWqgvj8UxVCkb/YX6BVsVMsoBxLly4rAv58OExKB1Nm5gK2PTwGxKD7sPmgt8ong mtHxilCUXCXgLmA9aMdKHFMRiB3oeAEc2Ij8iB19QiN0HbHJFm BCG9sHRvesPCYYP+ik/1CbQJthGz8bDSL2mRMdMv9KtFKgM7NlyC72PHjmZNRWxKxEA0A VoPbBCAlXlICemZbdY8mUzO7joCmxIkZumJzcX+ZmzfiIPA1A3 psWakDQ7+jBHovQiwZKRXuQAdB/ZKZzXJQgUwhUIKS3YT9Xlm04sThoaZkIFThq07fOGlAZZB1mg9 js8T5UjksMAFygrYp2Fzo5WVBVAYfJZKpYzBGgWGiWDOpw5AVU 1NTUwutDO7oaGRyQX1gkUJJWM+oIAIjIAt4Gi3EpQjkTTCHicm EgeMAEag1yNgbCmKpsUi0dma5lBHRAPQXHMz8xUTgoLJSkVVjb +V7crJsSaVeTQlRUgIxKeqKbHIHDgCdhtVV1dZWJjBrIrHA0JR wRq8NhmxL4cArUDKyspKkUDYLheQF/DMpUtX22EMGwJAM2Jzwe6kXt8DuAEYAYwAg4ARzQi0ENiaeKpR MM1a4cHjqyjSzUxJSEtZO5GyvtofFsvEKpWU4ovE1yT0vxKBpS UiCKFQXFtb59Cvn4lYhBQZPk+gRLulmYUwpOPAfmxYYQOtp7au vr6uztzcEqZp2rkoJR0REQ4UxszX4Ae0JKQoSVtagNWapdKa6h ozC3OVUj15xB2KEcAI9GoEjGlGDAWYmJl9Vk695yy3hsUvSYN2 g5X1VfCVLxQ1KInvq8VmZqaaWRXPRGySl5vv7++DpmZKWslDa2 fosybArIwiqeKiIrGpKXo9hCEdyJWfV+Dn5wvr/hJJE5sFcRizCQCIDHhNRfOgZJHYhDEe4YARwAj0BQTQDuxWowx ng9BMSixqJvgnGwQuAoWbCNbg0aYh+A3pEUvwBfEy8Sc11jKxm dhEjN50BfMPWKP5fJijVVfVmJiIhUKkH2m2YTPZCaJZ0lxYWAg sBC/ZMluOWnNR8AIak0sIuVrzsdQDb5kUFBQiqeAqs4kJB4wARqB3I QD3L/cObKPNgIV9sYmpgi9YXy8YKpKMMWvxFyqs+KomFT+HEv3bYnZF YWlubmaiseCw6g9oOiYoF1lcXGIJwcoCNmSDuRqM1zKZTNIEr6 dITEzMYBrIKj6tuVBdCrKoqNjGxtrC0gKoCmiK4NGw/wj4CyzcYogSixUUZiKjXYcTYAR6DQJGbEat7ZDLZHBqh6WVVZL C5FozBS+WgZ4CdAOv4ItEQitTMSIUrTkTIg/m6CKxyASM06RCUVVRo1RREAkpBfDivkhgYW4Jb/wzC/jMSSBoZ5JWLqGwRSZvampmZmewfAYKHLKLm1tYgJ2IBGs6UxRe Tes1Yw0LihEwiIBRm9Gt3DA/ApMRvJkKShD7Ar3BkhG/ID6CRTG+wFQgoMUwrUN2aPY3sxlJnYbVjNjSWnOpVAJTEwFYx1 mKYyaGYPZGx5Ig/mL2UhoWAF/FCGAEehECHdWM2CYBI4Cewu6QvoeNVNPTPZQAV40RwAh0NQL4M KCuRhSXhxHACNwWAp3TjG6rCpwJI4ARwAgYRwBrRsYxwikwAhi BbkAAa0bdADKuAiOAETCOANaMjGOEU2AEMALdgADWjLoBZFwFR gAjYBwBREZxcUeMJ8QpMAIYAYxAFyEwbdpk3ZLwNE0XExyDEcA I3AMEMBndA9BxlRgBjIAuApiMdDHBMRgBjMA9QACT0T0AHVeJE cAI6CKAyUgXExyDEcAI3AMEMBndA9BxlRgBjIAuApiMdDHBMRg BjMA9QACT0T0AHVeJEcAI6CKAyUgXExyDEcAI3AMEMBndA9Bxl RgBjIAuApiMdDHBMRgBjMA9QKATZ2DfA+lwlb0VAVqaf2bPX6d uFMudRj390gw/k75yXrmiJjFu77GrWbUi7zFL/jPZh2kYZ2SHuk7ZkPrP73vOZDc7DP/PitkBpn0Fpg41vn0iTEbtEcHf7xQBWl7y789rfzxfDQWJQqYO9 7pDJqLqMm80ekR5m9/rO1XVmPzH2vVxeQpomMmgqVEeqGGckR2CkJbmHv5m3a+JTZDaM Tzc8/5mIgChz5IRWXHxl292Xq+lrbyDvUS1pcUVDaKgWcufeAD3eYfu lNtNRDen7/n0o305FtFznhnmbuMcNMjxTsaYSpIZ982GP5Kcl36xZkI/we1K1RX5yIL9n3wal2825Jk3pzkqzX2DnaBhnJEdqY2WZu58//2/Cx3GPr0sxkJpF+pn0pFsfTrNnQyUngsMPJuvnEohXb1tiq7lJ5 4rFLiEjxg9fdaMoc4iracr1ZiTkGUZHu0s7rkt6SmSdRArqvrU xo/35VDi8CVPzYq1uWNVhpak/r3nBqgOTYdOlIye59VVPdXB5mihr6q79Ov+fBVhN+zB4YEBajk 4IzvUZWTOnq//LqTF4QvmjRl05zh1qM4en6hvGrB5Ju4j5z+97Mkp7gzZiiMWv/jsnBgtJqJq43//5NXn1uwpJ/omAl048DqBFVXx76E0mMQIg8aEWt8xE0EbeNbhsyc6EaYh85Y/5Nk1TNSJ5miBSEsyz2VQBGEWNMS9VQ7OyA4hL88/eb4Wmuc7PLBLcOpQpT0+Ud/UjFjYFZWp2VL00TPK27RNT1A1F/88kFxL2EYF2vVlBLpi+HUCK1pWmlGJ6nSP8O0qA4+J35zVH0+1 8bLrGioiOtEcbfTkRfG54E2d7zvY69ZQ4ozsCOaKiuQ0ZClyi/S37ArO7kidvSBNH74VaUlucgXqAudwf8s2+g8tK2SuiH3DXduN cWVj5r+Hjl5IKhVHznv2kXAbTT6qPu3fAwcvykctfyLGsi7lxN 8HrxGTXlwaZVpx/Wjc0evSwc8sm+iqBSbVkHZ8X9yljDrriNlL50TbG8aZrL5x+Pe/LpRZxj727Kxg7fFJk+Xxh/Ycic+tIU0cnF2dXdz6D5syxt+CGcI0WZ1y6mBcPDHxxcejzOpT ju3ef77ab/FL84PZ68YTtI5QVXPBpeOnLiVmVIsCxi9c8ICPhkwMYKUzvBU1 ueXgNpiw8nVvSdi76Wh8AR308HMLhzhot14fwjqloQbKSq7+c+ BYqvXDLy8IZiQy2mR1MfrwN9AcQ1CTZYkZ6LnmHuF3i2U5I5n6 9bZRJS28+PeeI5cz8tD4swkKaQONWnbdP5y9Q8vKEk4cPpZARk 0NKj91IrnBLnbJstkBZr2X3QzfJLqw9KIYWVFCERLXckCok0gj N6xgnPht99kbyaByE2T+3g0fxPEENmEzH58eYs2XZOz9fN2edB mTuGj7yRFrZ8M8D8b1sW0//napAhzZurgmxV3Zv/MSev67eZzZcfDgkQwJfLZ1k7JublFOWl5wcN3/fk+Ts+VsdYgMWzJAn3mSJisu7/ji2+OFSCBiz8+nYj+ermY1sHyd2/7FptOl6qIry/LTbMcNnYluB3Tn7N+6bX9SHeTyDEw7dmHXrxfKUUJ5FUkEWxhP wAiHAi0rOvfH91uO57LyEkWb1ypd1j8dak4YwIrLlExLizLQCh ohubZx1ekWpvCCn3YOGbQ8WN16mhthJmW7ALjE7/tp84GboEGYDJkOrsyNNllTgh78DXa9AaihWGVDZgrMqgi7YC1N mjMS4NTXRlqWf2LTui3xqMeYIPaLaP8s1Fy69Zezdz6dJ76ye9 tvp3KZoXoj7RyTXsY30zaJ6pbV02P6rsWErEhmbi+hT6SHWv2h 6m7s3vjVvquZOTVMv4gVNeU1CpdRc6YFWwuo8iOffbQnXTzyhZ XT+qGrZtamgA6Zt/OtD3YwTAShPO5Hlongc+nhP1gmQipWmLOG1mlp6q+f/Z4mGPzshx8vH+UzcNHrC/rrYyJCVXdl0ztfHi90m75sURCUYOFoqSlHVX/xqzXfnS61G7fymy3frRpng+p3HT8t1Aq4SJ617d2NLBNBbNGuL 39hmQjoMsK/QwmY5iBSKzz82dubjuc6THj+/XVvTLRHkY1pKdWkIazUmdv9IUtuFLOFNvH8hg5xZwirubJeriZ TPQhzFiZP+3H1RoaJIHhGeJgab7K6HG78DXa9QahRsbKixDL4I/IdpMUenJF62yjP2/P+6i3xisFPvL/2hSFmqFSPaG/mr/6gp3eunvnpo5/PNZirnwh2wcNHD7IXusRGo/W9Xhx6t/QGgFc1Zt9kKMctsnXKIbQLn7vq80earny87ItUlVnsyi9eCFNb AGhJ8s5dWUpewMxxqktfwvPdbcqCGHs+qA3NlmOf/T/nKxu+TSAJwnzA1PlDC375NRWstFYhD84f2bDzh3ONhNdgX/XEiCBaMuPOwsPPhKTErqOe+XA4Dx7qegLcNzs3X5YQVqMfmxnj ktZiv2R4rJd6GkAWHPo9QU4IQubPCbfjV5zPaoBCrINC+6Eeox W8AfNXTTE/vGZjAkjCc45ZMD8w/svtmbTdIGYp3WgCtURU2dFNO9JJXsCSlYtHOAtbmrzFx2pJwtz OXGjnpQ8rPa2hqjNy0VRGNODRt1Y+5F3+6ytXS+oIc0dbdpuRP oT1lGbqP/35kcK4dTvyCKcwPyseLTfSZE053Pgb6HpQkA1AjYoly1MK0d4i z0gt2yNXpN42KqvPbf2rgBaFPb50nK/qZBVSG/uF+rW1Huggoad3zOxcYpZ9/KTFof/78Fwz4ffk/1Y94KCqHZOpcm2dAOgU1Ssi+qxm1FKQWIJ6wDZ4gF3bOYWi4mY psmy4ht16zNFNN48mgs4rqDu89ttzTZ6TX1+9IBTZbnjWYTPnD LMpLQYmIjwX/u+tef0l1Whgej323pvzwxTFjfDZffCAW2siQst+lhAnT9q8+v0/kurB6qkv0NL0o1dgkmc+cIS/mdAhasIwDROhGeTJC8ClwqBxA215tDSPaYzIN9KNUfJ4lv3HjB 1A5JcgSbwXrl27YrxtYRkoIOZBUcxij9EErEwt6X8dyAfijJ4x Aj1UaVl9I2qnTYCvNYsZF1ZsVp3fdHN+CjKECAZMGRtgwSNrSx E0PI+BLozIehHWKYiJMPGbMjNSkIP0EXP/UEdRR1sEkBnAn7M5hqEGAZQN2WlobtUvRMv2yBWpt41U5cWjWT QhHjQpwpYvK0oqhdLEXiGOho3y+nqnf8jAkUPdpBk5zVCMS2Qg MmwK7YNDmOdUbw59lYzIsqR8dKeKfAc6t31eKBtzs+rhikOgtx W0npbXNyposuRaNkpO1TS6T16x/n+LI20UZZklLcwEg27KTalCOQaF2QuaC9KYzwODbQUthYloWNm FBmuNA4HnrOdn+6NxQeYc3PhDouSWMQkVphXIoisZ6N53G+jeO o+jpBISMigqElOQKuQVGwiTLrIkqRCV4hbpbU4opDIKfVE1ZSU h05Xz4HAnMVWTlglTGoHvEGaxh6YhhdEEhLzg7A0Y0DzfWGAPy CXJiUdWNquIWPUyOidWWi3Q/kiWJhcxQkb5QGHygqto9YnwGxnK7qIxiDBnkWT5jSx4PvA8It3 U8BhvERRkAH/O5hiHWlZ4oxwKFnsGOgF7qMgWBbSTK1JfG6m6pAvoYeIzxA9Yu uJmPup110Hu0FVUc50MPRk5gsHeAcWfmTlaB4f1eg5qbXsfJSO qNiMdMQ7c6FpLsUyr5SUpqBeFbkEO8pyTP7392sZrEqqxsJCZY Qx89tP3Hh/qTFRe2Lxm9fbUZmactBRcK4Q/ZgPCnUUteVfyYCiaBgADkOXJaFgJvQa6UMWX406XKMiyS1s/WHvWesZbHz4RaQ55KJLSM9TggdtUUICM32I7e+ZFANB/Tm5a8/HRCoqgmwtuIqIxcXQSVMX/ueHrsyihpYd14aH1qzacq0c3vawgAVlowDzvKIQ3wZJRo9yivI XlV//8YtP5WpXxBFRddi4jgK01mkkpqy8fQFNR5/FT/NWTVy6sUN0cASZpzJPaPjTQTkBL0k4kAKAm0dMHw1wXgtIgwhz lEcq69BQ0z3YO8xKWXd3z/fYkSYuRJtNG8OdqjtIo1NDNzCTNOdhJlnl809tv/5ZLwsxNN1JvG5vzryMusvV2FTemH999BtnCrf08+IWnvn/nw7hSbuXZYO/AWLlaAKWI/VlNmAvA3hfX2zU7bsTh5mR0FsI+KEA94dAkpOrzS9CDicr6Y83 zZdWUQ+wzIVaErJFZl1A01VXmXr18cu/uM4XiiKUhTF6yJCEHcvB9orxMyML4LPjM8470MFXWZzHaO6/myMcvpeRbTn1VtPbLbWmU5ehGpcjdwQqgNYmYEKR3I4lK2sDsg iLL0jOyFBXnD+49mi5xnfAG7HyimirQHIeQJ37/+uVaJhUEyfkv18PwmzULFSkvjM9GTOiDRiOZy6pOjZe/ee2XXInf/E+s+fIcIwnAsNSCbjJCXlpQR/rKjn//G0wlnCY+9aBmeyEnVtyPL6DPZKQ+8GwdVDXZp/dsviQlTMIfXxKpnr+qDCGsbl+bP3RzLnMHE/WXN764t5x0mf4hXXTMUJMt68+s/b8fDeDP2RzaGNTKxuwMZgGs4sTaFb/VmobMec1dzBmpquMeReaSK9WIcKRZf7y3Io2ZWsOzJG3zylNlC q9pq1qXPpj41mCwd2CHE6PJe0b79OKl/LYNhsHcPqLXf6dqU8+cOns8g9FIFBUXDh7KdvSJigntp56g80W sOURWVu047MlXnhoH3Uk3eoPxr1JB5O/66F3IJvScsOLVx2LAVAGsVX0zDc2YPKL9LBTlCSlogcc1AjarU Zm5jIVcUZxSGTxn9QszXZM+QGNOcmbdygSyvolwm7TyPzG23Dc vqsTKrR+fqFIRxfvWvQPfLYKmv/LMnGgnMY+gxGbs3FJeK7XoH+VceJ1ZGDQPnrl8+cMRaG8KqZYE JkXmPGVNfgHSSoiGzDz3sc++tWSUu4gsYEXVmwBUMbsAbxOiUk 4U7li5dAdktwqb/ery2SGttniCCytUj24gSxOZSRqdu2PNSvgr9pnwwkuLYlqnEEI b/QjrlgYxiqqMMqYLZSW1vhNfWDZ/mGP5Hwz4+lokrs9VGsafqzlUhRGoqepmds8DWUGGPbLqhRnoEU XVc0TSKu428ivZKsj8tBKH4AHytEzQR8nyxpCZry2bHenQ1orQ CobQQO8oKm6kIp3WJbJ/6044ThR7V2TfIiOYnO/56vO/M9U7ZuCVpsR/DslnvzLi1v5doWPsrJgjW4t9J86ZO2OoO7vSw7OOWLJ05Prt50 rofoHDJ82ZMzHMXj1G6KachDJC6BIzMdKBrj2SUE0IHaPGR8Nd xrPzcuadL/YYOf/JRRMCYYTSQ5cuuLJ+V0ItX+Q6Yu4zs6dGuQCx6A082yGPzb/42c4bjRbeUWOnzpo+3MdczVxCh6HTIg9tLXAePnPRwtFWF9dlV pExC5+cN1rz5rqyIf1GNWHuPXpaJJjn+WJrS2BYm/CZS5fMimbeejGaAMTiWQ58/LkJNZtP5MptfaPHTZ81NcazzXOWEyvO9lBVadnNPGt7i+ZaqWX A6IcXP/pA/zYqqQGEOQskhA6hoY5nUu1GzF746JgAKwGhrDLSZMLWCP6czTE KtdA5dnzg4T3SyLlLF08KUjeKM1JvGx2GzhgS93OR5wOPLp4zS LL7vQ3x9qMffnTGCB8LvU8qg72jrE+7USN0DBv/CKyAcsPXK2N5I0fExMUd6ZWyd4vQVF1GSo1jiL898IqiKiml0S vUz1bNMUpJeTXP0dmi7Wpdd4hF1aanNbkFe1uzYxF2CKYWiAb0 1zAo6HPGEnSxlFRTSTHp6O1giH3vrMrubtGdSXtXc4Nxqohy8U ZDsreGadMm/xN3vJ30mIx6a3diuTECvRcBTjIyoCf23pZiyTECGIHehwAmo97 XZ1hijECfRACTUZ/sVtwojEDvQwCTUe/rMywxRqBPIoDJqE92K24URqD3IYDJqPf1GZYYI9AnEeALBN2/S6ZPIokbhRHACHQUAU7awWTUUfhwOowARqCrEMBk1FVI4nIwAh iBO0KAm4yEeJp2R6jizBgBjECnEeCkHYjENqNOQ9l9GbAv9u7D GtfUfQhw0o5QIOxL7/12H5rdUBP2xd4NIOMq7gkCnLSDNaN70hfGK8W+2I1jhFP0WgS4 NSPOyVuvbWOfERz7Yu8zXYkbwoEAJ+3gpX0OpO59FPbFfu/7AEtwFxHg1IwwGd1FxKFoqiH77K6v1rz9U3y93pP5dSXAvth1M cExfQkBTjLqmzYjQ27IOf2gU/Vp/x44eFE+avkTMZZ1KSf+PniNmPTi0ijTiutH445elw5+ZtlEtdN p9ZDgdH+uPVzIon3rttOPPFT1y37w00jsjZ8dNcFB/fYNpwwo8236Yu9LwxS35X5AgJuMOCdvvRgOqjHr3z/1uSHn8oPu3Jx2bNuPvzEOrF1ck+Ku7GcdWLt5nNlx8CDrwNrWT arloYfT/TlyTs+jai78tOGPhGq5iqYVzVJxzAonL1c34mwmnFBvKmQPCeW SwR2taN6eL/Ze3FVY9PsXAU7a4XOusfVekGTpW/S6Iefyg67M2/nWBzsYJoJQHvcjy0TwufTwHywTAZH4hrUefK7H/XlKNQU+f3L/2nEmv7ZJ0tzcLCUdJr701BAbWgYXkPcv9ix9LhkYdem2fLEzQu OAEeh1CHDSTh+zGdGE9dBlH294LoTxQojckC9/7rV333xqjKuojR/089Xg8mbKghhbebPl2Gf/b3kk68fIfMDUJxeHsH5BrEIefPqZkdaoHK/Bvhr3PW3cn/u5Doj0ZnKCc3o+eFH9Nyaikc4AACAASURBVL7BcsiiZ2eHWhK2 456dC05/FBWZyBujU+xQFyiWUwbk5bCNL3YzSQd9sfe6IYgFxgiwCHBO0/oYGfFMvYZxuyHn9oMusA6bOWeYTWkx8uvoufB/b83rL6lG3vG8HnvvzflhimLkS9F98AC1K0JCn/tz5JxeWZ+X1+IycYJ14pGbEveJDwaaIy/PF5PBYOQ3ZbQb4qKmm0cTwVukoO7w2m/PNXlOfn31glDkkfH2fLHjgY0R6K0IcJKRkHPy1lubyMjN7YZcy w+67+QVK+YPcRbJyzLLbfu7mzblplRBPodBYfaC5itpzOeBwba ClpuMW1q70GCNL0KD7s8FzuNefNmx8OqWfVLCc1ask5BQ1V3de 7aesJ+wcBR8Q75przF+QKmaxjYy+Im1fbHnt/PFTpnZmeJjp5iuxaGvIMBJO31MM4K+4nZDrtcPuopoKbhWCPnM BoQ7i1ryruSB+cg0INxJDO7U85EDaa+BLlTx5bjTJaRB9+dQhN g1xDLlCvhCNvfwtRUq66/88ksSaTv+ubnBoCQZ8Dd/W77Y+8qwxO24HxHg1ow4Y3szPCSnG3L9vt7BD3QOcA7fJ8rLhC yMz4LPPO9ID1NlfVYa8rHOqzny8Usp+ZYPfTiBVhlxTq9qKsxB Ezt5fVH6mYu/fX9Z7jfn9YUaf9H6ZLg9X+y9uY+w7Pc7Apy0Ay/K3q239sFgrlIqafrWojiPx9P+elc6RI8bcr2+3qnSm2kNIIlHt J+FQu3A3jXC35JHZebWIAkVxSmVwXNWvzDLR8yXGHFOz+OJGDS VadvXpRGigJn/99/ZAa0eo/XJcHu+2O8KerhQjEC3IMBJO3fLZqSQNkgPblbwnWweWcy2js/ny0kSGFHA5wMl3SVi0ueGXJ8fdLopJ6GMELrETIx0oGuPJFQTQ seo8dFgJOLZeTnzzhd7jJz/5KIJgayPdWPO6Xl2kXMneWw4Vm3Tf/CkR+dPDrbVPhJBnwzEbfli75YxgyvBCNwVBDhtRrx3337z1ZVv GK2w09yhUuV8/4s4ItxzWAQUDjtvFApKJBI1NTVImiUDAgZA5N3Qkjrrhpyqy0i pcQzxR37LFVVJKY1eoX62ah/mSkl5Nc/R2eJuqY5GQccJMAJ9FIENn61duXJVu8Z19HWQTusyfL7/siVsZbAduUXaUl1TQ1GUrZ2NUkmlpqcGBwbfDZwF1l4+nSlXaB cYYafOIHIcFOmolVlg6eLcmbJwWowARqBjCHDajDq3mgb6kXZd 8BUmXxDYD/CbTaCAQDEbj8H+y+PV1dXDV1lzk3nG1bwjxyFTPwdHyKUrtlAo Kqk9I1c0617CMRgBjECfQYCbjDgnb5xtRsqRFoMwbMK/ll6Smp4rEIg05MKryMjIT08h5fAP7SSEXLa2Ns2l2dKfN1XWmN sMjhYJTQiaUKnav8UOJeQUZPOUviYiC04BcCRGACPQNxDgpJ3O aUa0SsVSmkAgVKqIHw5l3EzLrko/fvCXjc1SGasfmTm4Cf6OkzdUy+VylqHqCjOaduyrj5xuFhMEMa amJi6a+Y+kDr0swYZGSa2Dg72Xq58W3G0Usb7RDbgVGAGMALdm xLnGpg8sxuSMpmag8Ry4kFMnkY/pz+eb2blayv/adwDpTTxCYCaQTJpe9ePWpsZGVv2xtHF1fu6VwdOH2dnZuTg7m 5ubK5VKqIKSS3L/3dZal6W5raWpFUWhPcqaGZ/Wu/L6ZMLxGAGMQG9DgJN2OmrAbm2sSqWEDUTASmWVDUXFlVebUscH CRLKZZXCfjD5ApuShYV5SGSISWwUpES5eDxTO1t3O1v4yOcL4J +SoiC7UCisvLnbzjMYItUpNXUA2UEChvhwwAhgBPogAno0I0Hn vIMgkoDJGl8QNcCxtJm3o3xwfnHjhRLz2IgAUJfAAg1KDRAN8A twSmFRUXJyMpAXa9iGBIArFCASidEfvrtZdUJR4rFWyVTKFllt DlSAmagPDkDcJIyABgGw8+iCwee0JOmm045hJ1kjInwfG+1qS1 RuSzHvHz16RIQP6Dg8PszVYFcjH/gIiGffvr+uX7suEorANAV8BBKowNQE77KqlEWFBSRfpPQdXyY3 Z6mHWYqj04tLsUpkGH98FSPQ2xHgpJ1OT9NYFGDHkFgsfnRi1C MTInlgKOIRzJsfwDI0zAZB5+FT0s0//BAdFnTl/IX0QzvsTGmHqLF8GxeKRmREkmRpRUVwcExNTa2dkCyvqABbErA YQVhEDBrV24HG8mMEMAKGEdA3TTOywVhjS75VODARLNwzW4wEM GVTAb+g1ztA+0E7jpBbSFqWfP2qRKa6/s9xBSzkp+aZugYKZFIq5zTKxOf/tX9/WlqGpYWllZVlgL+fm6srZIbNkHh2ZrgL8VWMQN9AgJOMjGtGQB BozV6AeIclC7AE0dkHawWuAruA/v0D7O3tWcICBQmmafAfXk8X0NSK5U+l7zr266XTNbZ2vrbWPFM zKiePJ6sWhjzy6KOPMluNlHa2tmBmAnxbWppNTEyrqqvt7TQbo vsG6rgVGAGMgA4C3GQEb8rqpGwfARxEK5WQH5gGGKmpodEh6WS S6UhnHzMLCwtHR/QOBcNDBN1cUf3NDDKvysnRuvG6o5edx8tTPFVuIyy8w4B7hHae tLIJjN9qZQoMVmgpTSWXyyTN0ouXLrU01IwbM1xk6dRegl71XV GTGLf32NWsWpH3mCX/mexj0vndUsqG1CO79p/LrKRtB85atiTG3ngn3RuIFDUpx/ccvJBZLRgw96WnYux6qpx3C50u6Ou7JVqPLpeTdoxrRq1tArs1 TM0gg41QIKYczWBPkEzq5WTNl9ep5E08M1tCxatYN4SQyKkKvr JJyq8pk4uSaBlPYPUz5X9E6DxQ5BpMy2phmsaj0f2JqI0GWxNV lPyzhW2QtZVVY8rv0loPWytntEcAUWCPXt2n6jJvNHpEsSftq2 FSNSb/sXZ9XB7aLGUyaGqUR+eZSCXJPPD1hp3ouFqwoXkvCrLtoXe4Sp K2b8PavZloo735kEd7rJx3667sgr42KhrXGDOaqRck4NaMONfY 9LUGZmqUSunv53h+2LzBji7ZyVftRAoeXyQwtSX4YlrepKyRm/R3cd+QrJI2kkVJipJkRUmWNHlPxaYHPd7JFTn5KSVWsN0IvZbG 0JFCrgBK8gxZ0JT0jWdDtfvQUfa+McBBui+L6BPpHsUDY8R9s+ GPJOelX6yZ0E/DFmTB/k8+jcs3G/LMm9Mclea+wcxps50JtCR1x7sfHS6zjln0/BCzRrn78Cgbjpf4OlPk3UlLNyVte/eT4xUm/SfMH+Zq6RkTbdN5BfDuiNY9pd55XxuTk3uMGcvVO65z0k7nbpb mYz9QVYkERYR7BNn6RUQEPqySNUHreSZWPL5QKW2i5TxaiQYl3 9zaNHAk/MDnltVxqrpqmmrhiawEVk4EraSVCpVCSrU0gKaFtmxb2oljV9W mHrEZMIHdN9CTEKUacxKyLMOjnVkPIoxowBl/77kBTW86dKJk9Dwv5pKq7tKv+/NVhN2wB4cHBmil5mwNV7GEPGfvd4fLCCLgkcVThmlcPnJmv7eR qvorm784XkGIw599ZUls76Yhzo4wBm/n+tpYaZzXOccYZ8q+Etk5Mmo6frIl6TJPQPAEh2t++F7Yz9Kkv 7+pd4DI1Vvs6EWoaEJO8EiKpmQ8IeMsiA0KHq3QemzyBLRSQpB NAjj3saVGKa1XNRSRlVkmlLKlNF0UOFXgov162r1EmqqN//OH7QeSzRdtiGwjB886fPZEp4ST/eYtf8hTTTu0JPNcBpxVYBY0xN0wE+ktVp53/Dw66tZr+EDbHqkPqUFQ1Scejm+BiWj4pEG9mYn0doSxQdfxvjZ Wkv7rXGNMf+q+cKVzZKQiWwi4SdCyGfBLC1XZoqyull6+zIOX0 oBtIJ5PKJOait8aa+IfJPYKEPbzENg68QQiWs5XyesEIkvQmVA yoCqxFUzu5Hmnqcp0BcVryS6WnD2uqJM6vRV+d8hIJa2oUPVzt eyEAYaqufjngeRawjYq0K49UCZ+c1Z/PNXGy66Vd+RF8bnwAgzfd7CXFhFzDBK9xZLlianIUuQwMNCmE2 JyVHF3o2hpzsU8qILnM9jb7O5WdVdL19sRRmvtcF8bLclQAt0x Zih177/W/h4z3CKeUg6kA4yDKAlYhc9DvxE3MV+ZSyqlXJ6bK8/LhTjm7Q9UJGxFyn96pNDHG3xwCF28BFY2AjNznpBPK6Qqqrmmp F5UkEErkNWXZl6UvcNAk7Wppw8cukpPenFJhDUhr7hxbM/Ow7XTPljt2loyTVannDoYF09MfPHxKLP6lGO795+v9lv80vxgj cNGWlaYXAHpxb7hrm1VHVpWcvWfA8dSrR9+eQHr+QM2cpYlZkg htXuEH+MLRG/QW6yqKfsmOJdEvkmcxVRD5vnDR07n2M54YVGkttlI2Zj576GjF 5JKxZHznn0k3IBFiW4piT/697Esp3nPP2SecmDPkevllsOfeOZBfy35aLI8/tCeI/G5NaSJg7Ors4tb/2FTxvhrIABpyOobh3//60KZZexjz84KRl7eCLLoWh46rsotOsCSR8tLL+35LS6xUugVO2 vRzHDNgprRkmlFZcLhfUfis6saWkglDBOB8wMv/3e2H2vy73gzEdAcQqrxNyCG3o5gshrIaLCvVc0Fl46fupSYUS0 KGL9wwQM+GrQNt1ct7q0/XGMMXi5vSDu+L+5SRp11xOylc6LtO3f/6lTSsyI61xhaAdMwpBYhVUitIqm5SU1SLFVpbkb1X5aPKBmZmS FPzwB6Qtsk4Qf2bCsJvp2dytlVaGkChiQ0BpTqU9luEyfU5Qe2 bdmbUAsFBMSAqiJN/u7Nz67ICZ+lwWpfjGic7d+6bX8SmhJ5BqYdu7Dr1wvlSEp5FUk Ew2lKtDT3xG+7z95IRsKQ+Xs3fBDHE9iEzXx8erBpdfy+nzYfu AkGI5Mh0xENM0HZkJmCqrQL1tWi1EkMFRsCh2zLipNKUFKP/vLz36/65Wwxqty3RgsPWpKx9/N1e9LBESSEou0nR6yd7c7VhUpJzpmdW3ecyEUp++3+7HhSBvI6 QBD52wYNe/cB1hpFy0vObf9i0+lS9ZplZVl+mu24oTNbuYomKy7v+OLb44WM CHt+PhX78XRXIUFVp2cjBc4uLNi66dovazccKUYlCC1NHdR6p9 GSCUX1pe3rvj5ZonWolc2opyb6MkzU4WYisfQIyVzS20D9/Ys6wkBGVKzevqZlRef++H7L8Vw5k4wo2rxW6bL+6VAA1GB72dS 3fkOTuMYYLS84uO5/v6expRdtdYgMWzLApH3mXvy9c5YJHoU0o9aZGloRY35a1SWGpN RbIFv/sNM39jdLYepcoFjxCGVNnSr5plKKaA5+7pCMyJzf3lvPMhFBuE b2t+KR5TeyUPe5x4ap18jlWdve3cgyERoxu778hWUignCJ8Lfi EVTdjd0bv9p3NTOHcQ9CiBU15TUKl1FzpgVbU2k/rt7IMBEEzwiP1gmZrCgRDM+EyHdQOy2KSYmCoWLRpIysYr03Et m/rt/CMhH4xR40wFrTR1T5kc8+2pMuHvnCymn9UIlm1tzeHcmC3W+t2 cwyESSrTsqg/IfGejKsVV/exLKbqv7iV2u+O11qN27lN1u+WzXOBl11HT8tFBBggqruyqZ3v jxe6DZ92aIgyGvhaMmUQDcXpDAKo4vp1Y1vrD/Om/zqZ+uenzrz5RVTPBjP4MZLbrz+87tfnixxnfrS+5+uWzPbA5UW/Z8lkUzVHW4mU5ceIQ2JYVavv39RRxiVn7OvabLw8Gdvbzqe6zD h+ffXvTHRHonXmJZSTYFbUUPtRenaBDnnGKOlqb9+9nuaYPCzH 368fJTPwEWvL+jfl5gIIOB6rLYH59Z3lYpUU08rB7FztFuUxIx k9XBWZ0SUg2Jgxf4WeaHv8A3Yh83LTucg9o6maTSp9Hj4v++ax H3wbQplFxbaT0jVpqUgBchpcITaLywY0wfMXzXF/PCajQmgi/GcYxbMD4z/cnsmbTdokCMCxC587qrPH2m68vGyL1JVZrErv3ghTEM6clP/6c+PFMat25FHOIX5aW5cgixPKUSKnWektz6DkdBAsZBTJclLRZ M0JNHQefMH3vxuczJpMWCgM3ODI3UheeeuLCUvYOY41aUvIaXb lAUx9lwPE1qpcnnwlfd9Sja9swtUrX6xS1Y8NdGzetd/LxVVEVbOVkyfkwWHfk+QE4KQ+XPC7fgV57OQ5mQdBIipq5Om7t x8WUJYjX5sZoxLWov9kuGxXozORJbdKEKaEJm2/2/xgEffWTkTzfsWzWPzGS+ZIAsPbj1TTwgHzpkx2N+s+M/zxZDVZ4gPY37qeDNRYr1CGhJDaKK/f1G7jCDD3ddU2dFNO9JJXsCSlYtHOAtbmrzFx2pJwtzOnG+wvR rUtP9yjrGWzLizMJBNSErsOuqZD4fDs54rb2+O4xrM+tsDBh01 Bq1AtNIQ8wEFrRiEl248W77mkjqNplJac3i2fikMXOFZBo4fH0 xnoImFSUCkm5huzk0ohQy2gyJd1Hc1z7L/mLEDiPwSRB7eC9euXTHetrAMbi/zoCitZTBFxc1SNIlwDdNWdUz8psyMFOQgJcjcP9RRXSQo7tmMx 8d+If6WhiHlLhayykvZSZrf0s8+eXmyU2UZ7CUU+EVrJKKbbh5 NhEmXoO7w2m/PNXlOfn31glDGgqMTeGa+YyYOdmnJQ9wmjl76n0l+FjyypgjRj UOwnxUSkMw/eQH0PmHQuIG2PHDCm4gmiCJfQIwtjpamH70CczHzgSP8zYQOUR OGsUyEfHNnZN86o9zU2b7VLxyb01jJhDz/xHmQTDBgdLAVT1l78xo67NM1giX2TjQTcZE+IY03kNDXEUbl5+ zrlvS/DuTDmIueMQLtLaNl9Y1oL6hNgK81Zai9LGZtf3OPMaFlP1j+Ie RJm1e//0dSPXNWWB8Lhu+c9o2llQjhW4pP672gJiBWA2Kut3KNpgxm1sZ 8aaUnTVEom2az9R1O06BIqjYjHbl19YzyMiPkRdfygWdEPowNW kkyllJQQ5qyktA94DwY/FhTNWmZMO8S+A5hlsHYfd/KxtysevjmEOiN7l5aXt+oYG0raN4HrMDziHRrVZNlhTfKIbHYM 9AJqoFFR3VaVFeboLdYRVVqHppNesSG9xOpJLmMKdstkjV/QsVkybVsxJ5UTaP75BXr/7c40kZRllnSwsrUvhrwPVmdXoDKcxvkCY2ipdln06DvbKNiGLp RVCSmIG7yig0E5Y4sSSpE5bhFepsTCqmMosFGfSUD9bXbQPfWR lJSCUnT0oJk1FR2EJBJu08Ut1lxMF5y+fUkVLXLIB8LZV3ykaO F8MXCP5BVyTrVTL1CdqCB+jrCqPwEV1/LC87eAIbm+cYGINs/LcmJL4JmWUXEehIG24uQ5AgcY0zgOev52f5IbyVzDm78IVGir+ c5iuslUZ0jI75Kwc1E2q3V0JA6ZetXNg3LR/C57Qc0hWNTMmbsOwi0JJvRhfoFedI5x7ZsuwT7YQinoH6NCfs2 vLMpBX0jCFlBApobWA4IdRTS0vxkpOm4RXkLy6/++cWm87WgEclLUlCk0C3IQZ5z8qe3X9t4je1+ZV16CrIlOYd5C cuu7vl+e5IEDOLJzCTNOdhJlnl809tv/5bLsLZu0FcsLclPrYLkdiHI/g1GCSSeXXCARX3ygU2bzlSTjYWFaK1ONPDZT997fKgzUXlh85r V21Ob27s1UFcJnJGK2NY+2B+4VFV77XACEKjLuPHeiFvA7HMTX TVxdBJUxf+54euzyCBt6WFdeGj9qg3n6uENwoICFCW2szdFHQW 608lNaz4+WkGRpcnIXA0z3QXByMBSdmT/Ta37wmjJdHMhMxsV2FqV/r32jU+PIDl4HmGsSgZu7zreTP1CGm+gvv41Kj9M0nT7mqrLzmX gsrVGFnhl9eUDCTAAnMdP8Tcx2F7IwxXajbFNPx05vvWDtWetZ 7z14ROR5pCDIik9Hc9VXG+JU1sIOiQuHPGBlro6xl9owDKldoz AYf+jWoY7shlBGWRpEmPSaLz6xWsHYXGMCWVx773eKLMaunwpM 6mQF8ZnwxWhD5qXkbmsXtB4+ZvXfsmV+M3/BEzGVH1+CcpLZf2x5vmyasoh9pkQZn4D877rzJpX/eWNL+4tJ12mf2iiaszOQJM0ouLE2hW/1ZqGzHlNz7ZHvcXKy5hJmtgPaXBkaVoBqlyWue3/jubUOj/0oTVP1sgsoSma6ipzr14+uXf3mUJxxFJm7YcrgLKD6Ixn7yIm6 7JP/bQtheS5T/vPFHaHJtVUgXRHQp74/euXaxHJoSA5/+V6Quw/a5YlT1XZwMSSZekZWYqK8wf3Hk2XuE54w46oTWBW0lwGRYz2L/g77UKz9NKf5+aGTXFmR5LRkpXVVSg/obz54+c3eZZgJ4LHg1OIegEcznvoeDNV4LWYU0hY8DPWQKqWu3 8VxjIqufqarmthHqHy0oI60ld2/PvfsmjCaeJTD3qKlSWG2osy6Yb2YwxSnINnxehGpcjdAZn8TCI mBHFP0XUL60UxnSAjWs50fSu5aH/QZhz2M9zzugkghjM7ilZfoKk7mw3TZEM1O0QrqhzCRzplnUuDr 6pGm2FLVy2d4G/BGEzKE1LQephbFNwDypr8AsYG0pCZ5z722beWjHIHSxAFL9wx3 Sgrq3Yc9uQrT43zYW0jiqqMMuahJCup9Z34wrL5w9xFyuJmdrm VrCDDHln1wgy9HEHoKRYmVTlIaq9oNFOU1RagGSLRkp9jGrXon aen+oh5jd6uIqJSQeTv+uhduCT0nLDi1cdiWo1WKLlWUJQnZ6H y6Owt/31uC0rvO3P1G3M1W6h4YjPW2iWvlVr0j3IuvM6sRpsHz1y+/OEIByFBW7n14xNVKqJ437p34IpF0PRXnpkT7SSSXGa0SHPfwH4 2/WdOdbuwu5TIO3Aoe+yTQYzh3njJ1q4OAqIcOtl59LKn/Y59sDWbMPUN6sfKI7TpRDOFeoQEh8CUsQbq61+j8lPVHH1NEwG gcVbKicIdK5fuABiswma/unx2CEzZaIPtRZDphrZjbPlCt+MfbcukJWfWrUwg65sIt0kr/xPTo3fo6zapQzGdICN4DQ1O3AfWQO/cs9ShRS5Is4GTj0Ad0th/1HSkYZ9b7+Br5WLLQVYa5gRItLgGM8E7CTwzv5hg87yqgCmLn5 gdIbr6ZWpF08Apsx6eOMhR7bUarM3pN6oJc+/R0yJhhx5fbI12x9iEz1y6ZFa0s4jV54SOsbNijmwt9p04Z+6Mo e633r0XOoSGOp5JtRsxe+GjYwKsGMYSOseODzy8Rxo5d+niSUH 6lBUmJXexdHN+SiUh9ho7OQwt5MMdAf0iDpj0+H8eHeHFkqB1x JKlI9dvP1dC9wscPmnOnIlh9hrruS5coKolo0kfCjwL14EPzFs ye4iLpv0ghsPQaZGHthY4D5+5aOFoq4vrMqvImIVPzhutOYGAZ zvksfkXP9t5o9HCO2rs1FnTh/uYMzRekUMFxMwYOW48vHsnchv/SOTfX8IN8u8fZx98ayJSjoQOYQEEUWwxYOyjXCUr65MOHEhTEt aRj73x/CTX3G+3IhE9Bmmsb7zONFOfkFCi0Qbq6YgOZOTqa57lwMefm1A Duynktr7R46bPmhrjyXSbsfai1uuGdmOML/dYkLR+V0ItX+Q6Yu4zs6dGaXWlbu7eG8Pbs3PHxCkPdqQBipLM 8pcno8V9OOpayOwngpfUYIgyGyA1P8y9zN7QrYUC1SC60ex1RN sdQVlR73sEHyKw+9EsxEtRWqhqIaxmv2S74OWOyHPbaaja9LQm t2Bva5aJYYtZaoFoQH8DN/dtV9XBjMq6jKSqfmH9HdRUSFbeSJUPCGcHdAfL0E7WfP3TFzYk kiYxb379UliXvrAB/ai9pKwoO/Lpqm2pYEn0fuR/b8Pu6ebkn9/4JGXUx5/O99LhSrL8zPcfbrrU3H/mq6/NCbMWKAp3vfF/+ysJp5mffDpXN/lttLtnZbnf2tsZ9I/9c2jylBntcnRCM+LJm4FBkC6DyAURDqMNIWVJHYmUG3RBvaimq UqbiVAJTCGInVhliS1Qpd5oTDN+aO9qENoHDWS2pLGBJ3YO7X9 XKzRauMAuMFL7gEuxU3iE0Uz6E8BOoDyEonurxqE/bSevtNvcInKd9NLr9Z99vj+rYPfbz53ycpYXloBJKCmz4VEvmO dpBSW854+YKPTx91+fzFjUYFX/OrJeOwwZrNl10UlhenLy+629XdEXnSAjlUwCLqkRDQF9wAeYra FpFcMsjBUFlCO0Job2MTJcxQYNeaE0ag5Sk5c6L0NJqEwmoDdO cLgjBGCrQjpaPO8X4mtk09MdVcNm5lmGzHvn69HXjx+9mJpTVC nvHzFkyNQ5Y9syEaGqOf/t1+fAyO8fHWQFu9VamipSj23ZBcvfdiOXPujd9s2/LpDqXhdxv7W3a/DuHBnBSUYQGE2HoSE4M4RhHZaG0DX4xv6wn1mKYThIzVmt3ASl MDM1ta4kFLD0RZOYjO6ka6m61ON/nUArc4Sq+srh/Yl8C/8RYwfe1TcqeSau0Q8uiTYw2VdUpbEvuOVsX7V8u6Z94oCH/vvSIxEG3vW9EyTuZd77rb1dg3UnyIggpYg7kIFV/TIaKEGszYiNYbUhxDsQNJqO+jOrE7HKEfNbbTNi1CL44ZtYsyx GK9i3QLumefdZKVT1pZ/e++oc+0odUXvpr13xAQ+vmqhz/km3w2ISMPe52ZLfj90okSjhKD47z/4Dhz4wdcoQT8Ys3vfC/dberunBTpCRilSbdRhdiJmPoSNEaHSQCKsNsQMLpm+M5oRIS60 HgRKkNaFD0zrmPbXWH4jRHMaG0Sp0HQAAIABJREFUNaM76Fhhv 9jnvop97g5KuFtZhfYRc1+LmHu3iu9x5d5v7e2SDugMGcllLMu wC/hohoZsQzxCoGL5CBk32bkW/EWaEXAMrJRBApSSPY4WHBQhkxKsoKE5mmamBryEPDiigMmoS/oVF4IR6HUIdIKMlA31tACM1kwbQRlCx+YTcMo/DQoTxIthroUW+9kpG1KFENeAjRtREhPQ6UWsyRvN9cDnESqD4S/EWepE6DBJHDACGIH7D4FOkJFKIkHcwbxDht5Rg6yMV2ueCHYeM YoQqExweAsiIOadWHRUNkEDPSHH1wwjqc3e8BkxUetMDvl9ZOe AMJlrZs8Kuv+6ArcYI3B/I9AJMlLW1YNnNIALNB5EJMAsMNsSMVMtigeUBM6ugYD4QtCXwB 0REBAP4lUUHHPHpFSBjqQCWxMyKCGzEqMTseYmEV/RomA3/KmambemcMAIYATuMwQ6QUYiD3+T/oPIgiyCmUkhDoJpFtAQ6D7APgqahtX51oOx0Qdm5oWWypC1GgJ PCV4bYScR8Bk72WMiIZNYTMopq4BopQyUKPRWMg4YAYzA/YZAJ8jIZu5z8APTMKqiSJ6bThZkKPIzyPwMqqyAIClKCNoQw0p o0yNj20b+rpmpnNocBPM1ZgWNsWtrDEmImGiB0FRoqkxKBOXK9 o0f77c+wO3FCGAEAIFOkJEaL/C76OoNPxYjJrMxyEN1ZQloTIrCLPhNZicrSnPRghlaa9NwjtpE hHJoDEjqz8BUQrGFmHmTyWLBSnHEaHVF+A+LgLIh9Z/f95zJbnYY/p8VswOY84VwwAj0PQQ6T0Y6GPD4AvA+BD9EzHj2Iq0gFaUFZFY yWcgwVFYyVVvJGom07yS1woR2PJrzGitNx883n/G0TvG9KaLLPaODF4vD36z7NREZ9R3Dwz1vj4losir13xOZ7g/NCumDp+D0pgHS02Xt8gHcqQZ3ARnp1scTicXe/eGn9ZJK0qCtOslzUmmZ5kwvmMmZmAsdnC2efEe3qN4T0/We0Wlp5s733/+70GHs08tiLJR2oX6tJ8B2CBZaVpZ86eK1xMSExGxweMSzvnzF wcbGJSB86IgRkb7qMws6VBJOdB8g0PUDuLOg3RUy0hWCb2ljGj oYfthL8KqkojgPWZ0Y25PIzMz62TU8oc6hE7oF9diYrveMTubs +frvQlocvmDemM46kVY2pB35ZdufF4oUhBD2WyAd1M7XzYYqz0 rOy0y+GPeH1+SX/7s4Uu27qceC2jMEoxpzErIsw6OdO/g+b2fT94xWdv0A7nS7uomM2skFvCP2GQA/xJjpnRa5Z2bocs/o8vyT58ErJM93eKDa92RHG64oOfHV+z9fb/Ycv/yTR4KLvljxTTpNDJj/6upRdnAw6rkfP/j+QkPhkY1fe6x/c3w/9cb3jpbds9Mh1zJCYN8uC1Rt/J8/bD+QbL5oQ2RHCu1s+o6U2U1punwAd17uvvmeYudx6IIcjGf0df +d0V/LN/TtF6uoSE5jjsaN9O+UnYeWpv2+9ufrjdajVr61dKSnuDyRcfzh OohxBcQzcRu55IlI9Iin0g9fqFS/bXj7YmrnpGW11VJ2E0enC1RJK8rgHdrbDlRtwp4vVz/71MotN5tbF29vuzRNRqrm4p8HkmsJ2yADboK1auls+jsWsEsL6 NoBfBui3RvN6DYE7VQWsJYknDh8LIGMmhpUfupEcoNd7JJlswP Yk0C5fNXfsVt6Ts/oesUw5EheJS28+PeeI5cz8pDbVpugEDiSuuOBqjzzy1HwvuEwc lIwkBgcZZGNbHNWgSEa74w8c99BbkRCPkHUFdYpCXBXfSvo9RO vIwAAdvXIgeNZzgtfmu0jpsmam8f//P1Q9aT3V41p3SemrzSarE09feDQVXrSi0sirAl5xY1je3Yerp3 2wWpXnXo4I8iCfeu2kgvfnOcpL7x55Vqe1eiHoqiyK5eRQ5NL5 4sXhQaqjWvIj/mhPUfic2tIEwdnV2cXt/7DpozxZ58WNFmdcupgXDwx8cXHo8zqU47t3n++2m/xS/M1J4WD97PCZMZ3ri/j6MpoMJZeHyC6BesdOZCUc/B08wDWlbgrYjoz0LuivrteBtWY9e+f2347xXiZJ26knWNqlPHN 0AZvLifuLrI7dEvP6RldvxiEJGPP5+v2pLMnpRRtPzli7Wx3dT fQsvwTm9ZtiWdcjaAg9ovo0G2gSa+sSzxdgL7Y+ziABY6W5Cah u0noo1UMrH6yExlTy1unYsONp9dPvKZw9V+lJOfMri2/nkQIW48lQQ+RJn37+rorcsJrUZD6bCJ9pZlRlQkHtm1h/Y8HxIAqJE3+7s3PIK/P0mADs1Falhv3w9YjGdUk0ryo5kapyciW2rOfrNyUrrIc9fY0o VDpYsMjwC2MqbV6xZGWl5zb/sWm06VqRamyLD/NdtzQmcgrLiKp/Vu3sS7OPQPTjl3Y9Svr4lwO/mSCLSCFNPfEb7vP3khGuiOZv3fDB3E8gU3YzMencztbMJZeHyC Mj942Qf/I6SEDWEfi9i247e99bZomS9/y0c/nGszVphC74OGjB9kLXWKjwc8nlxN35R27pef0jK5XDIfqI599t CddPPKFldP6oV4zszbV9IE8b8/7q7fEKwY/8f7aF4YwR1d7RHt36ghrZWMF+2ofJUdWa1lRAhymCJM0tTNIZp goJdWMsyDC3ltz5JoBP/FMSq1AN17duOYnhomA5LzCwd8ZWXI9A52I5zx4oD3CXX9pZM5v 761nmQgJ1d+Kh7wVorzusWEGrOlU2bEfd1zOra5vZIJUaTVkyR yX/HPZQE1mYSN8TAjwcSRFrGMRNIixM6vqL3615rvTpXbjVn6z5bt V42yQuK7jp4Wiyao8a9u7G1kmgtiiXV/+wjIReGCK8IcEVN2N3Ru/2nc1M4c9GUqsqCmvUbiMmjMtmNPbgrH0+gFBUrUNekdODxnAyF/u3Qp9jIxownroso83PBfC+M3xe/J/q5Y/99q7bz41xlXUxok78q+MfNXbqRi39O896s4ADG7p/7fxvWUz2QOpud3SN+e3c0uPPKO/vsgX5XcKUztp1ieGVdrOXVlKXsDMcapLGhns2T5QVp/b+lcBLQp7fOk4XzNJFXrlpl+oX+cOjuWDYgCWIYfwKOSqg6xIY VwQOQwMsmk1VNPSwnTkRRHIYAByRQKhjZ94P9cBkew5sMhPPHN dO9CE7ZgX3l49ywVFukV4m/Go6tRUdMqtXXikM1oP1V8aj1R6PPzfd5eHoQFtFxbaT0jVpqUg NdBpcIRmGtmmNvYLWXLmeBHhOmHZf58dCwzO8523atlI8/yzmaC2iENG+EKTydKbyIWS1eBx/qjryYJDvyfICUHI/Dnhdvx2XUYrwAXlqo9fjmRXb3nOMQtfenwAeuDbDRrkCLIJ7cL nrvr868+eCUHtN4td+cW333z+7n9GumppktpiGkmvHxAueHv2A AZ/WXcv6MJx9+rqhpJ5pl7DhrpJM3KQJzSXyEA0axDaB4PBhNuJO//O3dJzeUbXI0awOONoIqgUgrrDa7891+Q5+fXVC0LV5mmq8uJR8 PwnHjQpwpYvK0pCTnHFXiGOHbFV3AJW6Dhs0asffv/563NCrfnKhoxk9Gg3GxDu0loM3ZR2IQ/pEJ5jhqkdLxrwE6+71sa3DhoV61SZUg5F2AYNsBMgD76IB8T+r GHFQGlCy8Dx44PpDHQCrUlApJsYvBUy3n9tB0UaOJOfqkm+Vm0 +/LGHHJP2nq4mfOYve9BLJM06k4r8cAaODICpTkv22SRgb5/pD/VH5iIy/+QFaLkwaNxAWx54w00sgUiRL9SIoOJZ9h8zdgCRX4KcYnkvXLt 2xXjbwjLAxDwIOfXUBEXFzVI0KXQN69hMWV96A4Dowqtn5PSQA dxqeGwFqUs/3EWlq0vl7ERh4DQsEd0d1sFhWk9bLSfuvpNXrJg/xFkkL8sst+3vbkbdgVt6npmpkJlrAMfwPCI1/r+geg4xDMigqku6gO4YnyF+Fjwy/2Y+8u7hOsgdHvNUcx1lZtc6mTOMhNg5Klqdgm7OiS+Gz+aRY5D uwAbgvENJcBOaRs8eqeYiA37i9TChND++EIpDdl0ReJq8WoDK9 ojwRLUYK42qzUhH5zJ4RnmZEfLca/lAAiIfhseU4BhGJFYbtDQCw1+hy+glCy0sbS98908VIY6cPcZV SJUd/jMeKX1+wwdYgR/OK/suNRMOkx9/wAUNaEVFYgrS1bxiA2HSBc51mQVFt0hvc0IhlTFdpmrKSkKuSZ wHhzuJqcq0TJjdCnyHIBea6Ix3OAJH2ZibhVxpOgR6I1fCtLy+ iW9jrfYlBfHtg770xgBpXw7nyIFEBgZPtw1gXVm7NKaPaUaADT wImbtD7K/9mDPgxP0O3dLDTVSXnoI0EOcwL2HZ1T3fb08C5/McYhiSoSWfcZpt6+0qbkw/vvsM7DAirP08+IWnvn/nw7hSdtFb1ZS6d9N+ZuZlPKhqrx/PgCe726SHAjULXGTZqZ935tCESfhjS6LVPkkN+IlvpbC2tclLr ucgrnQOc1OVXNy9+wb64jgoAN2zxkpDehTShfoFedI5x7Zsu4R mo05B/RoT9m14Z1MK98F6POuwB0dQp46B/UsUMi7EsiX7r293MgzoHOZnqaw4/eP2NMpx8rI5gcx6Kd1ccBMRjYmjk6Aq/s8NX59FNjJLD+vCQ+tXbThXj5hJVpCAiNpyQKijkJbmM25y3aK 8heVX//xi0/lawE1ekoIihW5BDvKckz+9/drGa9Ct+oOe9MYA0S2QY+RAIkODR1qQitprH+wPXaCqvXY4AR6 NLuPGg5fbDqHR0QGsK2vXxvQ9zYgsis9G+rdntNohNQOYASfud +iWHi1ZMURC1F/e+OLectJl+ofggJZDDAMyKBuKqxHhSLP+eG9FGjN/gBsmbfPKU2UKr2mrQIehybKzP338/YVq3nWPIe885GZ47k41Zh79aUc6ZTpw6cszfUDroMm6/OtHftvydyppFbH4zRdHtyqNNBwlhWrj8BPPSKETwM6TwZw5VXP ms5d+KWFXBc38QxhX28ZKI0uTYMmLIBqvfvHaQVi5YkJZ3HuvN 8qshi5fynrPZWPb/FZUXL+BjEumJjWXfv30l3/y2f1MAirvxPe7tyfToUteXxCsXpqimioY+eSJ379+uVbz1pHk/Jfr4Qk1axaaF8sL47PRJM8HPbDIXFZ1arz8zWu/5Er85n8CtjSqPr8EiUdl/bHm+bJqyiH2mRDEtvqCvvTKzsLLNXKgUgODp9sGsL62d1V8nyM jRcWNVPQgdInsr+0CR78Td0XBnbmlJ8iqjDLm1pCV1PpOfGHZ/GHuIkJRqCuGfhkIpciMYRcyP63EIXiAPC0T2kCWN4bMfG3Z7Eh mlV5RlQJOueFD/ukL5ZMf8dTDRoqKsz9+svlCBctnvMxd773wp0ohayER11kHTX5 64ZzRbaziQju9fuK5B5mquY5VYKQFJY6Rw91uXsglCY9wd2Zvj 5HSaLKhmmEHsqLKIXykU9a5NPiqarQZtnTV0gn+FnrvdlrezDB X0+Vtmwm7YYvnlv/6Zx5BlP69aTthPnDRmpcnsvYglAa8g7PgyGulFv2jnAuvM8qke fDM5csfjkDbtsjyhBRmR2mUjzlM8vILkImRaMjMcx/77FtLRkH/ERRfxFp0ZGXVjsOefOWpcdrPNpS8XdCT3ggg7UtBc0zdkQOp9A +e7hvAusJ2bUxfIyNlfdqNGqFj2PhHRqhNIixeep2437FbenDP HhrqeCbVbsTshY+OCbBihjCnGHplAHOFw9AZQ+J+LvJ84NHFcw ZJdr+3Id5+9MOPzhjho7k/eRYhj8wNvrqtJnb5/83Sy0SVZze9u+kyspewgZZLpZTY3M47PCgseszE0UEaB9q3RpF eP/G3krT9JPIYMcLzzBlR9JTZcyd7JLz3ygWYaIX4sIZ4I6XxzPxi gs3zqgKmLH5idoTo6pepFU0Dp8x6eOIgRz0rVeq6xd4THwq/siNJ4Tb4occffyiYd6Pk/Jenqq0CYh6aP29CsK32OBY6DJ0WeWhrgfPwmYsWjra6uC6ziox Z+OS80d5q1UnZkA68bu49elqkHRzAJba2hE6zCZ+5dMmsaGe1V UjoGDsr5sjWYt+Jc+bOGOoOuq7hoC+9EUB0CuUcOZBK7+DpxgG sI2wXR/D27NwxcYoB93tdXN/dLg6m1kWUi7e94aGtJcXdcUvfaTE6hIuyqaRa6Oasdy7ToUK6N JE04bPn118nLUe/9cVzwYxq1BsCVZue1uQW7K0+uAB2raYWiAb0t9ejbHZrkzo9cn rTAL6F5LF/Dk2eMqMdsn1NMxJYe/l0ZvDcJbf0nRWjYyILrNydO5aym1LJixORKds25oFOnm7STfLp qUZoHzTQ/tY1ntg5tL+epN0e3dmR06sGsBE0+xoZGWlu+8vd6pa+feW98Tt Vl3zmWoOtp5dHP1FT3vnfzzYQtqOffDig12hFvRF0AzL3qQF8P 5PRvXBLb2Bc9YJLdGPiL19svnLrWDzCMvqp1U9Gay8V9IJm9BU R+9oAvm/JqMe6pe/BdwpVk3CuWGwulkphbiZ2CBg6Zd78ySFtjMc9WPo+JlofHMB9z YDdx0Ycbg5GoE8iwGnA1ruxo09CgBuFEcAI9FgEMBn12K7BgmE E7i8EMBndX/2NW4sR6LEIYDLqsV2DBcMI3F8IYDK6v/obtxYj0GMRwGTUY7sGC4YRuL8QwGR0f/U3bi1GoMci0JfJCByHXytgjmnHASOAEejxCPTVHdj33nH4bXQ9 3VJ08dA/lzMKSupoW9/BU+c+FOWk/5zT26gAZ8EI9GAE+qhmpHEcTmcfOsEc2KcVwBX61WsV7SLvfRc pyo6se/XNr/f9m1SlkFUWpF7Ys/79X9NbsGJ377sGS9A9CPRRMmIdhxOmIfOWP+R561h5cIX++yev PrdmTznRIxoOvuFbyUZgIpY0EW7TVn31+UfvLvJC3V+bnFLVpQ 6ou2dM4VowAreFQF+dphGM4/CpNl52Wh4uWl2hR3XMdfptIdqxTOAbfv+vu49ca47974dPhCJ/y3zbiIceeszl4RBLQpqbzXgPFPRzx2+hdgxPnKoPINAjFIQuxx E831/Zv+WHXw/naNuvjblC1xYDHLEnH9m69sOt1xtVhKI2Je6HD1d/9Htas+6sCRyoX9i/dcN7b676aPOJfE57OfiG//C9nXkkQTUV3jix768rNRShYHzDw6vX54vVM0a+3dD5k71EioqL 23+8gA5ltox9MBw5QMUBI3BfINDnNCM4QzR+30+bD9yEA9dNhk znsTezMVfobXnImCN2TWr9DtQ77RseTgr6+YNv41uEZF096zna e/orSwYhlQkHjMD9gUBfIyPG8/1F9elfnhEeyPUXuELf993W0wVNDawfesZ1ulPkHG7X6Ywj9n8Z XzcQkCN2zUhgHbFrvjEO1NfsSCfdJjy/bLLlmY/XHqslGtNSqqlQLx7jGx7cfGkC6xt+s5Zv+Nq2vuEVVYkJpRKoV Wzt7Os7aOTU6Q+E9uvwQd63KsKfMAK9FoG+RkYE8nw/Uhi3bkdeq+d71hX6I01XPl72RaoKuU5/IUyPf0LkEgg5Yp9ifnjNxgRw9wOO2BfMD4z/cnsmrXbEru7qNg7UnYUtTd7iY7UkwfinJ4vVvuEf80/7+fvTNaxv+JTPtHzDy9v6hieUkioJIQx7/sv/G2aLtaFeezthwe8Egb5mM+LyfM/io88Vejv0OuiI3ZAD9c77hidULbUNCnD44806/LmTHsV5MQK9FIE+pxkhD30cnu/BPXDHXacbd8Ru0IF6p33DI3+hUp6JU2Qo8jCIA0bg/kSgr2lG4D+Ry/N951ynG3XEbsSBeid9wxPg7P16cpm8NrdIgvy+4oARuC8R6HNP YrqZy/N9p1ynG3fEbsyjPHJS3HHf8GTOn5//ngJG97Tfd90c+hxeQrsv70TcaKLPkZGCy/M9dLQeV+gcQ6ADjthpo/7pO+MbXuw9YZLXsZ2FFuFzHgxUe2DmkAtHYQT6NgJ9joy4PN9D F+pzha7bux1xxG7cgXpnfMMTIvepLzwnKguaOLhfT/CwrIsJjsEIdAMC2FVRe5B7siP29rLi7xiB3okAp6uiPqcZ3XHf 9GRH7HfcOFwARqDnItD3VtN6LtZYMowARsAAApiMDICDL2EEMA LdhwAmo+7DGteEEcAIGEAAk5EBcPAljABGoPsQwGTUfVjjmjAC GAEDCGAyMgAOvoQRwAh0HwKYjLoPa1wTRgAjYAABTEYGwMGXMA IYge5DAJNR92GNa8IIYAQMIIDJyAA4+BJGACPQfQhgMuo+rHFN GAGMgAEEMBkZAAdfwghgBLoPAUxG3Yc1rgkjgBEwgAAmIwPg4E sYAYxA9yGAyaj7sMY1YQQwAgYQwGRkABx8CSOAEeg+BDAZdR/WuCaMAEbAAAKYjAyAgy9hBDAC3YcAJqPuwxrXhBHACBhAAJORA XDwJYwARqD7EMBk1H1Y45owAhgBAwhgMjIADr6EEcAIdB8CmIy 6D2tcE0YAI2AAAUxGBsDBlzACGIHuQwCTUfdhjWvCCGAEDCCAy cgAOPgSRgAj0H0IYDLqPqxxTRgBjIABBDAZGQAHX8IIYAS6DwF MRt2HNa4JI4ARMIAAJiMD4OBLGAGMQPchgMmo+7DGNWEEMAIGE MBkZAAcfAkjgBHoPgQwGXUf1rgmjABGwAACmIwMgIMvYQQwAt2 HACaj7sMa1/T/7Z0HYFTHnf+3S6suoZWQhCTUkYRoonfb2GBjY4LB4BqX4Dh2bM eX6iSXnHP555xc7pJLLpfk4thJXA6McQFTTDHGNNGEAAkkhFDv 0q6k7buv/GfeW622vJVWxUgz/F6IvPt23rzf7/N9892Z2bc7QAAIDEIAzGgQODf1JWfXuXf+3z+99LuzffzNOC9v bfzi7z9//smX/nrVelNOGERSvLO95K2fv/zM8787beCCKD9EEd5a//nffv3Ksy/81wk9O0TZYF4eUXjDkHVE9QcTOCll5Du3v3PnmrWkhDtx4+Q5h leoPLyd52VyeZDx8va6/W8eibx305w4jTZME+xhQdbuMLTaI5MiVQPFeVP531/9c+fKx29Pis2ZmR2lDLKmL7MY13f5/TfPTNn86NxIRhYWPloIrOHCBx90LHj0rjQNz8vlzu4breqp6YN lyjsGKzOS8IYj60jqH2s5Bicwpmc7uH/P6jXrfKqkuGfEGmuOfbBt+7a//faVZ59+9gf/9UmVaQzebj34OdpOvfWLbz/96KMPP771uR/+5Xwvrp53dJz/8DcvP/XMa2eC7eLwlqtv//5E8pqFUyLCxsqJGMOl93/9/WefePThRx594qU/nunzSJ0znH79dwdMM9etmlc8Z2I4kYzvK33zD6d0d8ydHKIJH7 UTyZj2w797vaZgYWqIXCZXyCwVb7zykz981s4EblD8oGVGEt5w ZB1J/YFzGdkrgxMYWZ3DO8rj7XJ4B07o0pzp2qf/t689b+OWzctC5bIHN17Z9i+/ePc3oen/+UxR2Nj0Oxj9uV2fOxY/99pmVef5d/5QUvjgvFjs7HKNLm+Ks9uuuzsrPMgzWar2lXDTfzRpcC1YU8O1 jpjczMHe3F2icD0Xtr93LeXOra9sVrTs+Y/fHzM0GlhZnPjGw+lPv/mXs5aI5UunhkwYEXljxYEyftr3kzRjEpKjbt+u2slb8qNEBeRh Ofd9/buKoqRBCA9aZkThDSqrt5ojqn9MSHlUMiiBsT6ZZH0U9oxQ5/ijX/+5Zubjjy9PQU6ENkVE/n2bZ2uMZacaHZIUhr2TM5z8617VsiVZMWqlo6ZEv3jzQsGK8OZ oLL3BJM7Jjw5y8ONovVCnzJ4xedB26Kjd9sN/+bBDFoRerP7kPw4nPrBx5czs9JRoW1+0Tm1s1tvF4NCL7x6wRC lD8uanTRwvktmbztXKps6ZMiiDoFVytJ690JdYnBPhfjcISSme lTTEyG+QMiMJbzBZfdUcSf1B0wi+4CAEgq9kFCUHea8YRa3jeS jTduj1vexdvyh2uwOKRh6alBEru2JjeHvbhX07dtcv+NYLs6wX 9ry3syTk0Z89UyC0S7a36vAH7+8usSz8zk8eyUG7eEd72cFPD3/6Rfusb//syXztQE/H0Vxa1ci2Wvlk+eXt+1SrfzjwpuvsuFxpippelKjGdTK91cc+3 nlcvfGl5V27/7HtUJ1u40++tzYFv8bbm0s+2XPos+NXexSFL05xWwOjL/909/Gr1RXlTdaoggd/8K27kjVsT2V5b8JKj85WwNhQt2jfhc4Fm/CsNG+6XhG2qDju/cttRkYWoWK7TrxzOnm5ruIGN2+q1q2S3xnvnKQvC0TJrzAKT8b 0XNn79ntHa3ocTqtRMfMbrz67IFbuRW+aoqPii/17P29IWzK16eixhqyXfvVMYagrBGdnRY0j+Z4MLWu4cvC99w4Y Vv3k+0tjBdxSp+MtN47s+PjzU5dMc7/9r09P9+2Bcn3Vlw0xRfmxwsXN21uRznu77/j21umh9jbJvDzLICy+9TMBw5NQIZCsbtroQvNRM3D6EvW76uHt 7b48t+b0el2ubOBrz4/qnZO6+ykVKvxrzmgZDLhHaqN6GMQ77ajqv+kHO1qOHmjULZ0zy btfwjutaFpUdePQ7sOf7S/pDOk8/Jc/flBSeqEpvEAYGvD2hr2//XNVzoPPPjQvLU4j42W8+eqO3+wwLdi4brrGaGJ4rzGXalJWopb pMdtbPnuvKmNV4cB7MNd37WJHyLS56E2eM147unv3ru2fljfXH 3r/lGPGvXdMsVwra3Wi87H6M3///Wfq25565ScPp6mTijNdLQr16n75m7LMTV//zqs/fSSFC8kv0nFNJ3a8/uePGjk44W3rAAAgAElEQVS24sN/vLuvwsAMGhuaSWcdLaVVeJbIWneJm1mUHikzths5GbKi7edzN8 3qrDImzs6JdCXkd8bCkGsHAlCS+RUuStDIHK2H//iXa4VP//Q//vNnXytOKX7yq/Nj5d702L7rV/VK29WLrbKQ1Lue/cEPv7khp9+JsGfWVOgj02Sn3/rD21+UXbjOZ6aLQCVP52w58Kc9Ife++NNfvfrChmmeToQaaNm+ bW+98cbHdayi/cj2bZ9evnF270f79+4558hKVhnKJfNius+7y+B3BL/6A4UncYUEktXdDHibhJoB0peov78etteX5/rkWs/LlTUFvvb8qBaqLg9Q8qt5Q4Y+EPAxbt20mREygEtd6uRUn5kV3 tZW2xs2bdmqe9fNj1OqQ2Xaooeff2xBpDp5Xg6eWGA6Dv/vXtXqVRnhkxZ/7et3p4fInU173jydeFdR3xcHele++GRhmBd41eQ1P/71d1fF1u090JO/LHNgwMNbakublBlCv0MRmbvi/jUz4lS6Zfc/9uTG5UXZaTGh0VPi1Kj1lb21rWXhvcVxKqbzuj4iN0sc07Fdx/++n7tzbV6YnDdWlrREzCiMV4dOWXLfshhZxPwnn3vmyYfvLoxV DRqbMnbeo9/+1+8sR1NE9qZL5rzc+EmxKkuH0dZ1cvvF/IdXhlwt7YyeXuiaoZI4Y8Kk6aulKUkUjldx+pN/3dkzf0WGVs7pz+04wc+YGuZHLyI6Z1YG19AdNuv+1UWJEbFTc+ M9RmS2povNrIphM9c//8wdCerY/BkJuOsoeTok1elDZTXXG42yqLSMOK+evTwkcdbdWx5cEs1oCh/a+uRjW1YXZc67Z+2MEHnirNwodax0XqpJxe4yqDn41x8gPH8VA snqcenI/dWUBV3/QD1KX54Z9oNel2tEwGtPgmpCwgABtW/NuTGGgMDBjAYlwJg6TTKl0qsbg956jVeONcSuWJ2rZQzXrtlzN z5yW3Yk11LWGDmzIBZ1oZxtJw61JC/MHBi5OJqPHe9UGo9+WKbb/NKmgkhf05artRqFvf6L80bddDRMcW+O5gs3nAlFGf1v2Pa2yq6 o2QuF8ZWjo7I5pGBusprvKdtfqbttRoxc5mgrv+6YXCCO6diuc 4fqJy8rikFnszacr1fnzUrGrdLRfrHaMaU4xeV5Q8Smips+Z2o EqsJef7ots3CSJlwXzhmuHHj3YsGW5Tpz5ekmTc4s1wxVgDNKU 5IszOrPHaiJWzQ9WoEs9lpJS8yM7Ei5VISOtsvXmanzPUaH/dAcHRW1DNJkTX6MrKe62pGOu5XYiyRpqBIW3zvLfvBX3/3lrjq7xB1Szo7yKlvqQveEERpRN4TkTddh25LOCwH2KONXf4Dw/HMMJKvPBeujJrosJNP3r9+7Hi+ekoUlr70AVD0J+CjlB8Qnn7F 76tvIxq7m8alJoQpVycz1N3o8PsXlbTV7d+mXPbM+K4Q3X7/QlVCciT5SY7quXuMyihLRdY8mGco6QpMmoc+B8caYDOa+G1f0I ZlLN391Q7FOw5maGvr8b5vjzE11Rk6tUQxYn7Pj0hVjWFZu/ydjzs4rdfzUIjwS5K01h08rl65BwxPLjdN14XmpqPtjbTx3zjC pcEqYzNmrtxhrz7eEpCSHo/31x/ZdtiYWJqvser3ZUHm5VzcTGxxj6bMzxiBiQ2nY6892FcybrFJG xkdwLYdK07+yVKc0VZ+qk6XOThVtlzdLn7G7WoKSZGFb8/mTTZopKRFM14WP/vL2FVV2PuosSUTIGCrKepMWZPtO8CD6phtoPmxONhqZockadOL ZqGuFeEnG1tnYziaseOm1766OqC6pRmNP3w1raUiYlx/lurIdbaXl1imz0BsGzzImKfWRFw2UcfT1ct71BwhPIkejtKxW7 yDZXi81+eDr97oCvXhKBMPKJK89aar69tpzbkoOvadSjLnPBwh qIO1H/vTLt6tcn4n4KjDy57RNYKt0cxYm7vzo422n5n5jGTYa1lh9aNt Bzebvb8QTO+YbJY2RRY+hQRFvqEFzOznOqs93t4Wn9Vk4U+P1d uMkrvrM2VpF4W3T+uycTW8wW/Vd5SfOdSSvXJ3iB1mu0mrlfM3nZd2zl05SoQmL8qMH9xzskMXe gXtbeOOM1Zf1EYURPOPsurjzI9u9Lz6SjmZ7u5raLNbwioNvf1 zTXtXKhtmvfLrrYuzspamtPc6+nkPvv7m/m2EabXxoxa53uvPuuD2yqt3ZV1Oy790+a8qytYtCBo2Nt+vb9A 7OWHvsM+3axycjgUOiQ9Vxq55YjXpZtsrT15nwuQmu+xtYo+QZ F6aclqRk9A+vPdrRw5jaPnhz2/TigkkKddpM1Ow5m1+EyX1lZ9onzc2P8f+I0dZ4sT1iegEecdmb y5pksdq203tPyqYXtvme7v1fvH60KvPZ/351WVhMXPry9XPj/GpDw+Tz7bHFM+NdFzZqsxe6uKiaw9uquKmLp0ipHz1/eqtY5h8Hdnx6Jecb//2TpR71WwKEN903x1VyaVlXLorzvHSc7V5qLlcFW7/XFcj3VXnwlACeIn3tsZ1SimdNl5d1cdEipeIod82O+p0//UXlPa/90BMIfss4UWpatEn4hGYsN+XmTQ9kZeeOZZXjW5cyOrtA11d9f PcHB89eb2xqaurWzrxv3cLUMPxOaa/Zta00Yd2G+ToV03zkg0M1jtgZd923PC8+pOfi8VPnymsNITm3r 72taHKYNtxedfzzz09f61Jlr1p/W26Uymfkh2pDtxSlqm6cPbL3YGltY32jXjVlznTFpaPXTCZDj5 mJTk2Ndlzdsf1EzdWjez5Db/h3bn1oQSL+eFkut9eeudgVMfv+R+6Kry7ry1q46p47ijNjNHJL dclVe0Lxuq9uyG4tbZ68fO2G9StyY1TO9kvn2jXpi9ZvXFUYr1 GGDhIbazj5+x/9aufZ671xyx9ZV4inzuRyZXjWsjvQXJOc6T738b7LfVZ9n91qU 8QnxYfYJM6Y3r1HilJupEThlbmq6yUtOeuffmhFzJUduzvmblw zLVLlH6Gt7uRV1fy7FwvjR6/N0fzZByXhd29clKDmTRW7Pj6tV6UvvW/NLJ3aj8YD9xb1XTp2pFxv6WELH1g/Ew9nfWur3fN+RdaDX5nhurGC6S774ooic9bye9cuTev4RCqvHK 2hv8yawr5LX3zmVX+g8BIi/FRQSMuq9r5y0Adknmo6A6QvUb9nPd48Ff7AZeYrkteezI8qusY y2fJj/ZSm6E8PKMX1XDvzuTcQNOdxafs+y6qHbk8Vb5zxlSCo5zeuV2d n5/kUvaW+DsI7ejpMYQlx2BF4W2uNPjoreYzugZRUwF75vy//F/fCb5/Nn0C39EhG6rkzeEpsT/neXZWJq9fNQ31Q09lfvfA33Su/eSrXYwptyJOhPr+x1aBK1GmxsTD66w2y9Ew0xT/CzVG/7cd/Ur/4swem+FcRfF6eJx/T8CSy+rLq/7KuPd54+d3/LS3Y+tjs/oGwRFJD75L8Oghtw7RBMcg1MYn9XWZ051F28qClR/2is/1ylTVlY8rwGueoTzvaCoKk5Gw//tZ7hhVfeyBLuP/KWnu6PmLOV9AodJibKjJJ13+IKi47c5iHexV3NByrzNj0beE2L r8tyLx8jhvL8PxCQju+pPq/rGvP0VLeNWfrY+4pOamURrzPv6M74qrgQG8CXG9VmSF+pjAbS9 3mqNv52p/KIjLixds3287u2s/c/fLD+FbRcdh4p8Vs7m2+sOugev1Ds/rvoBqHQCbKKb+0a0+Tsui2L8eJELpbqmd0Ey8VRn/505376x1cyrHdn+Ytv2PmEN88u4mhjcmp5Jq49Cns0X97/uikqZlJSdNuX//iSymuTyPH5ATDqITpOvrbH7xZE1O0+smtGwvRPQa3+EbstXdLz Rnd4hcppA8EJgoByTmjW/5tZKKoA3EAgVudAJjRrX4FQP5AYIIQADOaIEJAGEDgVicAZnSr XwGQPxCYIATAjCaIEBAGELjVCYAZ3epXAOQPBCYIATCjCSIEhA EEbnUCYEbjfwWgHyo9+fa/f+eb/7zjhtQP9Ix/gBIRDGM5MImjJ/auW379svGSB+7AHi/yrvOinzn66HcfRTzx8q8fHSMtvvTVr/qXA3vp519Bq7xR9mWX/vXLvvvLDXj5NnizvontA2DfRNgSp8LLB+xhFs1PGCMnQj82M/QaYRJxBL9rOMuBBV/rBCk5EdYvmyAoxiEMMKNxgD5wStZw6VT75IV5Y/fdTnH1q5fvcS1XgtbnunpD4lcqR561uBzYjBF+127s4/HIZASVex/iWr9szlgs3zaCYEYuCh1H3hJmJPFLyQHUC75kgAqGt5s3VZ9pi 0M/Fj+WMnisfuW7PtfwopMsPdhyYJIHeO4cMp7R8B+ycv/wfA8Zu/XLfGv2Pzfs8SMwVqMDv4rHbQdnri/ZvfMou+7bj2RrmPYT77xzJf/pr82PRj8g3XPlyCdHzpw9Xa2Y99SPvrF8Etdxaf97uxoWfOv5e ZEs+nmed6pnb33C/xco/NZT81/Ma7CFxjxJ+FRlazjXEJK7SVgLA2++EaIfT/Vdwwv9UHTA1dy81ghDv3Ltuz6XjyYSi2cFXi5tiOXAgljGS+4b D1oJXIp/hMN73a7Mb/zT/Prd0kvdeWQklawETy81xmD9Mlyfv0ZBBLMssnuohc8om47zufz 8n47lW7J/7Td9D9N59tN9B3d/clGWhVu4o+nIjsPdGZkRctZw4R+v/bUma8Nz3/vB5qTOU/tLm64e3XvwwCclpvRU9OtgzpajHx4157jW6xqI2389Nf+1wwZb aMyDgH9VqNXVcv3rqPpFWNbLSazhFXg1N6/1v6TW5/JSw3/xrEGWSxtqObAhl/Hi0Xo83qu/6Q3XJPlz3ut2ff/5leGVJySXuvMQybdytLScTJKn+xgpPsNYH80Tpc86a36ZSgZzv uGaz0JyPgufea/Td9Mb0rickDIzUunmrbk9O0SdPAcvDsQarl7sSVxYGCPrPv6n1 6/Pe+wetCyIOi4jNVwTHa/LX7m6SCPTzS5CPwXP9lwt60ldUoD6T16b/3pqUstOBVxozLMu/6o4U015b2Kx8OtrrH+EcaEKvzW8/NfqGjiF1/pfUutzeUQzrCyGXg5syGW8wvziiYvNleTvU1XczA2PPiC51J1H Nn6Vx6qkeQ4cI3HISNYvQxX6aiRRs0QwusQ874XkfBc+816nz/uqpPUZZWaEV7i5fqY5auY0vACIsfpsW2RBfjzffOSj6pQ1i3R4 MQnO2NzMZSzM0KIVai7XoaXJ0GwlOuhce/zA+jb9avuvpxZg2alAC3INXDb+Vcls9RfaI/Iy8a+BOaUj9F2ySnJ5LI9L03P1K9/V1jyv4GFlEdxyYEMv4+W7XlhA/j7rdg3NFqXmU3kAnl6t2DeeEa5f5qtR0MEMTcwr3FvgCXVmhFY urJFnFCZoeGdb6Rc35FNnJSv7rl/sDJuSKCxm4Gw5driveH1xjAL1m8qNybNTNJyl7sTh65qMnDgVa zc7BiZR/ddTC7S0meRCY56Xj39VVntbeT2bUpigYk2Nl86d94+Q812ySnJ 5LI+zeK4RxjI+63N5lAuweJZeOosAq7x5Lwc29DJevuuF4X6rJ H+n9wpr0kvd+TRNn8oZacW9DvKNZ2Trl0ktKxZcMEMTQ++sjIM ZzZw+YQZG2wS2o6O82uSMr9j/j9O9TCdq7FvSQjm70c711tX3sRnamt1vVc35xvNFaA01U0NZi8 NcdeCN0+2MtdnOhV748K0LcfPvW5WHVw/BG2fr8V1PLU9q2ak86YXGohffMbv/E3D/qu6Y0VjZbWfO73q3Rlc0nWd9IpzW/cE/+y5ZJbU8lsf15rVG2J2rvFdbWx418PPUw1subapJcpU3r+XAhl 7GS+a7XliUU5r/vcXtXiusWSSXuvNkixD4VL4oVFJxr7bpG88I1i+TXlYsuGB4/VALnzmaD/7Pv++LfubnTxTQdmdpAJOkbd00zlR75pw+JmvJ+k2LZEf31hU+ cN/02PBovrbk8yOlLX3W8Dlbtiyfgtez4O2t5081ReYsWv/QmuTaktrIwiVrv7J6VqLHDzkrQtQ+66lpJZc2C7DQWE7kwBqD/lWFcvryU826JRs23ztnSmysX4QSS1ZJLI/lOcfluUZYTpTCe30uz4JyycWzAmSRG+WUXOXNazmwIZfxEj4E9 FwvTB6IP1/vtcJagKXuPNmiS9uncmWolOLeTWAs1i+T0CjYYIYmxluvf3bMk LZwwbRJ/e+OARoxgbtvsXXTbBW/f+kP6m/95stftGxkC3JNtCtoYmZx06L6stYvG43MPMPwKhV1MykYieRvY NM2THNrj8ZrdeELn828CWvnjGxBrtFcpV/GsRMzi5sW1Ze0ftmolJKrJNYxHlWNE/tgqmyXt3U2NOstDGvXXz95qGvFc5uyb4IXTWyBITogQAoBmnpG 9pqPfv3q7mZWHZ2cu2jj01tXCCvbwwYEgAARBGgyo5DsLb98aw sR2CFIIAAEfAlQNUzzTQ6eAwEgQA4BMCNytIJIgQDVBMCMqJYX kgMC5BAAMyJHK4gUCFBNAMyIankhOSBADgEwI3K0gkiBANUEwI yolheSAwLkEAAzIkcriBQIUE0AzIhqeSE5IEAOATAjcrSCSIEA 1QTAjKiWF5IDAuQQADMiRyuIFAhQTQDMiGp5ITkgQA4BMCNytI JIgQDVBMCMqJYXkgMC5BAAMyJHK4gUCFBNAMyIankhOSBADgEw I3K0gkiBANUEwIyolheSAwLkEAAzIkcriBQIUE0AzIhqeSE5IE AOATAjcrSCSIEA1QTAjKiWF5IDAuQQADMiRyuIFAhQTQDMiGp5 ITkgQA4BMCNytIJIgQDVBMCMqJYXkgMC5BAAMyJHK4gUCFBNAM yIankhOSBADgEwI3K0gkiBANUEwIyolheSAwLkEAAzIkcriBQI UE0AzIhqeSE5IEAOATAjcrSCSIEA1QTAjKiWF5IDAuQQADMiRy uIFAhQTQDMiGp5ITkgQA4BMCNytIJIgQDVBMCMqJYXkgMC5BAA MyJHK4gUCFBNAMyIankhOSBADgEwI3K0gkiBANUEwIyolheSAw LkEAAzIkcriBQIUE0AzIhqeSE5IEAOATAjcrSCSIEA1QTAjKiW F5IDAuQQADMiRyuIFAhQTQDMiGp5ITkgQA4BMCNytIJIgQDVBM CMqJYXkgMC5BAAMyJHK4gUCFBNAMyIankhOSBADgEwI3K0gkiB ANUEwIyolheSAwLkEAAzIkcriBQIUE0AzIhqeSE5IEAOATAjcr SCSIEA1QTAjKiWF5IDAuQQADMiRyuIFAhQTQDMiGp5ITkgQA4B MCNytIJIgQDVBMCMqJYXkgMC5BAAMyJHK4gUCFBNAMyIankhOS BADgEwI3K0gkiBANUEwIyolheSAwLkEAAzIkcriBQIUE0AzIhq eSE5IEAOATAjcrSCSIEA1QTAjKiWF5IDAuQQADMiRyuIFAhQTQ DMiGp5ITkgQA4BMCNytIJIgQDVBMCMqJYXkgMC5BAAMyJHK4gU CFBNAMyIankhOSBADgEwI3K0gkiBANUEwIyolheSAwLkEAAzIk criBQIUE0AzIhqeSE5IEAOATAjcrSCSIEA1QTAjKiWF5IDAuQQ ADMiRyuIFAhQTQDMiGp5ITkgQA4BMCNytIJIgQDVBMCMqJYXkg MC5BAAMyJHK4gUCFBNAMyIankhOSBADgEwI3K0gkiBANUEwIyo lheSAwLkEAAzIkcriBQIUE0AzIhqeSE5IEAOATAjcrSCSIEA1Q TAjKiWF5IDAuQQADMiRyuIFAhQTQDMiGp5ITkgQA4BMCNytIJI gQDVBMCMqJYXkgMC5BAAMyJHK4gUCFBNAMyIankhOSBADgEwI3 K0gkiBANUEwIyolheSAwLkEAAzIkcriBQIUE0AzIhqeSE5IEAO ATAjcrSCSIEA1QTAjKiWF5IDAuQQADMiRyuIFAhQTQDMiGp5IT kgQA4BMCNytIJIgQDVBMCMqJYXkgMC5BAAMyJHK4gUCFBNAMyI ankhOSBADgEwI3K0gkiBANUEwIyolheSAwLkEAAzIkcriBQIUE 0AzIhqeSE5IEAOATAjcrSCSIEA1QTAjKiWF5IDAuQQADMiRyuI FAhQTQDMiGp5ITkgQA4BMCNytIJIgQDVBMCMqJYXkgMC5BAAMy JHK4gUCFBNAMyIankhOSBADgEwI3K0gkiBANUEwIyolheSAwLk EAAzIkcriBQIUE0AzIhqeSE5IEAOATAjcrSCSIEA1QTAjKiWF5 IDAuQQADMiRyuIFAhQTQDMiGp5ITkgQA4BMCNytIJIgQDVBMCM qJYXkgMC5BAAMyJHK4gUCFBNAMyIankhOSBADgEwI3K0gkiBAN UEwIyolheSAwLkEAAzIkcriBQIUE0AzIhqeSE5IEAOATAjcrSC SIEA1QTAjKiWF5IDAuQQADMiRyuIFAhQTQDMiGp5ITkgQA4BMC NytIJIgQDVBMCMqJYXkgMC5BAAMyJHK4gUCFBNAMyIankhOSBA DgEwI3K0gkiBANUEwIyolheSAwLkEAAzIkcriBQIUE0AzIhqeS E5IEAOATAjcrSCSIEA1QTAjKiWF5IDAuQQADMiRyuIFAhQTQDM iGp5ITkgQA4BMCNytIJIgQDVBMCMqJYXkgMC5BAAMyJHK4gUCF BNAMyIankhOSBADgEwI3K0gkiBANUEwIyolheSAwLkEAAzIkcr iBQIUE0AzIhqeSE5IEAOATAjcrSCSIEA1QTAjKiWF5IDAuQQAD MiRyuIFAhQTQDMiGp5ITkgQA4BMCNytIJIgQDVBMCMqJYXkgMC 5BAAMyJHK4gUCFBNAMyIankhOSBADgEwI3K0gkiBANUEwIyolh eSAwLkEAAzIkcriBQIUE0AzIhqeSE5IEAOATAjcrSCSIEA1QTA jKiWF5IDAuQQADMiRyuIFAhQTQDMiGp5ITkgQA4BMCNytIJIgQ DVBMCMqJYXkgMC5BAAMyJHK4gUCFBNAMyIankhOSBADgEwI3K0 gkiBANUEwIyolheSAwLkEAAzIkcriBQIUE0AzIhqeSE5IEAOAT AjcrSCSIEA1QTAjKiWF5IDAuQQADMiRyuIFAhQTQDMiGp5ITkg QA4BMCNytIJIgQDVBMCMqJYXkgMC5BAAMyJHK4gUCFBNAMyIan khOSBADgEwI3K0gkiBANUEwIyolheSAwLkEAAzIkcriBQIUE0A zIhqeSE5IEAOATAjcrSCSIEA1QTAjKiWF5IDAuQQADMiRyuIFA hQTQDMiGp5ITkgQA4BMCNytIJIgQDVBMCMqJYXkgMC5BAAMyJH K4gUCFBNAMyIankhOSBADgEwI3K0gkiBANUEwIyolheSAwLkEA AzIkcriBQIUE0AzIhqeSE5IEAOATAjcrSCSIEA1QTAjKiWF5ID AuQQADMiRyuIFAhQTQDMiGp5ITkgQA4BMCNytIJIgQDVBMCMqJ YXkgMC5BAAMyJHK4gUCFBNAMyIankhOSBADgEwI3K0gkiBANUE wIyolheSAwLkEAAzIkcriBQIUE0AzIhqeSE5IEAOATAjcrSCSI EA1QTAjKiWF5IDAuQQADMiRyuIFAhQTQDMiGp5ITkgQA4BMCNy tIJIgQDVBMCMqJYXkgMC5BAAMyJHK4gUCFBNAMyIankhOSBADg EwI3K0gkiBANUEwIyolheSAwLkEAAzIkcriBQIUE0AzIhqeSE5 IEAOATAjcrSCSIEA1QTAjKiWF5IDAuQQADMiRyuIFAhQTQDMiG p5ITkgQA4BMCNytIJIgQDVBMCMqJYXkgMC5BAAMyJHK4gUCFBN QEV1dpAcEBgnAjzHm3rlVpOMdY5TBDf1tLxSLdNGyCNiZHL5iE 8MZjRidHDgAAGO4xqbmjo7Ox2OW6LtaTRqXbwuNXWKQiE1tuB5 rrPJcvmEtb6KNZtkI2+ehFxjvEwZHqFNnxY2fZEiIW3EfgRmRI jeEzvMxsYms8U8e/bs8PAI6fY5seMfVnTIec1mU1VVFco6PT3N/1je1GO5fMrZq0+ef5sqWidTKv3L0LSHZ1m2t6OjqsJSXhI+P0o eGTuy7MCMRsYNjvIi0NbeXlxcHBqqtVqtqK3K5XJe2KjBhDJy5 4LcFmWam5tXWnpe0ozQ6MxUfzV17nJVaLiss4FnGJlcQIGAiPV 4gpng/SZ3qGKcPI/FdeeCHqHnKpUqSqfLzm8+fyyiaLEMzIia657EROx2e1hYmOhEK H6x6SJXcj8Wrl58HU/w7NxBup1UNFb0Vwwe7Ud5oS08PNxms0unwzGM2aSKmsR3taA5I 3QAyl7GYxqCPwstWHiIcHj6knRt473X5UL4PzhqAYhMjoan+KE c/eE5Vt7boYpLcpqNMo4ZcbzQMxoxOjhwgIDYhlFvXWy6og2JO3E HyaOLJBaYsOxQeGLw7ghFD3Ibq/gUZYoKeObllZH4AmqlrEPGcaitYidCU9qoDbvsCBsRLoVauNCL nMhAxPBwpw77EYoVT5PhnORyJdqlUPDon9OhlCtcPaaRJgNmNF JycJwHAcFw8KUotlLxFdSA8Vun0NjEv+KDiUzOnYgYuZiCGLCY AnqMyqCRmrukZDpCsxS7QsiDWNQ5QiaHu0cyvBP94YWB7Ijnei VP+mXsdNskfiD253CnSHgsl3FyhauLhybykduOLgIwo9Hxg6OF 61Jsmbh1efQXPBu2+yXxARHYPJNyOxF6IOboys413vJOSJxVQX 0i3CFikUOjkQxuqxyP2i+2IKFPhKZeEK+JPnDtH0jixPGoEoWs wB08/BRngB7iJFCvCOUrXAwjFhfMaMTo4EAXAXQ5oh4EapziX7fdiN0 K9NSzAZNCTewZiY4j9oOEhuYaUg8FXYkAABPxSURBVIkPcILis MsvK6FTxGHnYTEZ9AC3VZaTycX+kauJC12JUfYn/M491juEwaToMsKbjWBEeKdCgR8IloQxCHNjo9nAjEZDD44VCOB LEW8iDvGx2FxFJxLHbqIlkYIMRe6eJ/JMzdOk8H5JJ8EIUB8IOZBQRJg2EoZp+BGa8kV1C50KPExzVz5x yYidQWSkwv+QLfFocCamhkesohmhsSey3UDmHFRyYEZBYYJCgx MQm677LyqM26PQJxJntcU9g1cyAV8VM1J63ygk5iV2AwPFLHQh cJ9InDASGirqKOGekTCRLTblQEdPyP3C2FMcmMk5NG0kPEU7UW q4uyeMQ0cXOJjR6PjB0bhjNDBnJDQ/3KdAl6nYs8Ctsr/T5PN43OHx9rbzn3ywr9Q0+5mX1qaHiLMdnj04MXK3n6KAxVs6x TTxX2EexS8R3FCx9YgDNDxaQ90g4QM4YRob7xePkjzar7px3iF yQUNSod+D+aD88Ef7aFiG7ucUnEjIVwABc0bjLNetfXo8IhF7A a5GinCIj8QH/s17IvDibfUH3tgf+ciL//a4mrFa7ayHZfbH52mjuJ31j6rE7LDbSroJAoIJCGaEiuIJI/SBGh6wYTfC09ji+A73LaS8TAIP57RVGZU5ceqx7D7wXLfBpo4J i5L6TotHEIK/4D/4/7i7K3aSxI/V0JANPRW6ftiTRu5FsrFMTQIh7LoFCOD2FHhzN2B8HUu0dylAv LXlwrHjl2801VbX9UYUrtv6xMoprn6LVPER7WM7jv/fYf7uV6MdJqPDswYxSNFAfQJGT1HPSMxIzDiQl+BEkQHh44U/OHlhPhv7kchB7EYIfwNvPOe41tzXbLMevdxuSEos4kxXe2R5BR lPZoRqAh8V3Cu8paPulS+s99ydf3/E4BYiBIl7RmIieOpavDsBf4gmpIj/4XyDO3OAUmBGAcDA7uEQEFum+FccneFmJ7xTuppi0E7E9VYd3l 8RMu+uzVvXhcr6zvzxR3/5rKR18YY09XACGrIsZ7ha2jl5+VQ1G/AzILeN4q6AMImL/oojULcfBToPbpWuOSNMBR2GG6rQWAcarPgoYAPm7b2dr1+0Tl8 yc7m6df9l+50rF62P05Tt/eC18s6VKam5o227cm3K3OceDCtQNPHOgEHgBD2dChUU3nxwImg qXugQu1KCOaNAVwPsv5kEPM1IdB/RjMQ2LEbidqVBAmP1pdu31xd/bXORlnFa0Rfe1dPu++oLodPTkWcM2l4GqVPyJd5Ud6kjtnCqds Aspcq5Y8aG4nEHjTtfqYPwPhwsdjnX0Ay3V/HWR/GmR7ELMagZ8Yz5vVPN+iWrV4Z29Da0doYmFNgbmSbWaGVlMYkx SmHuaZSb0zRLa+TMeDIriE3oBaKTolEmGqeJNMSZI2H42T9nFE RNAYqM1l0DVAu7bykCvhPYKHmxuXpSGNKMeHv9vjePxD76nXy1 3e76JRJWoSvK5Rknuken99qxw+dq6q9Vt9ojstc+8+SSmJ6rX+ w71DzzqccKbFeO7D1Qpln3rS3ZwuiFM944eWD/Zxess772wrqpaBfv7Lpy4tjJ42e7pj318gPZofaWS82a7Puih/hkPVDMHmYkOVATxjOunhFG4QaEwQh0cJSDmpGjr/eUWTlD0cHZHQ0tRsXkvMmstbux8X1L1gsbJicYOlEdTlPvwRtd BxscMxbmPhGvZO3m41WtJxTJL6Tb91xsOdyr2bA8+55I3LFhrM YD1/SVemNFHxulS/7eAl2ctXfflWvdUzOfTpC3d+oPVHc3RMdO7es+3hv+wqq0Amlj wGYkJixmgfngXIS/rp7R4CM+zyvC9/EQM1e+xeE5EPAn4DeBjcYy4oYuVvfjoR7YW4/uOBO39vYE1omGTv0b67TbGdbWeODPb1RmrH/ym6+8simZC5mWr75x7PDJU1+U9Wh7Tm5/59NLV6+0RhQkqtBhrK3pyJv/V5e1/qlNxWmTQtCdhqz5+p4399mK71+br7HYUJvh7F3VjXxaYbxiqKC E1z2zcNuQuDPQfK1wr7VrkgibEqpkmP84pQY1fM7m5BnHhU6nw lLzRknT8dAZ//pY0YLeDo7lTJ3Nv61UL1w2p0jNW1QyY1fXJ9WdO2qMLVb9zpbQ mYsKU52WixZ8Xpuh9d9PW7OWL/veY+se08lDMtOU7e0f39DvbdFk65S9XSZDaEhlu0MembR67T0/vr84RxV8tHh6Xpibx3PYOOtRbGBGo4AHhwoEhDkEiQ29JLE30C 5r3ZFjhpxlU1XCLcteGxq8fXRMfvuqNJnF2Hb5NHKdnKiYabev mRunUoXIQgs3bt08J0KVNCs1DB3GdJ/a8YXmjuVJ8pDCLY+uSFLLnK2f77w8+fY8w7HDvUu/viU7lGfNjdXG+BmTVb5nCvg8UC6Sjc/VWRLaqNBlcG34FgihV+SxL+AZ1bG5P9r6wPNJCqfVUm6LuTNTX trOxyVYQutrWAfqKNr2XjJPLtYZzpf3FK18MlEVmZy7vnhqnDJ y+bx5Ty1KLtDIozXa1CgFct4T5d383Pycnlr7tdIT3aEz4526r On3Zmvlsck5Mj4qOWUqa+4OmXJ/vnaSvj6ZNyiFmXepTYhc7NL1d/bcyYg9I0kgQTYUMKMgQUGxwQhIXbjD22dvOX/VkliQoPSzIp7Vl59qSViQFYJsytpa0azOzo9VsE5zZ/UNe/a69YtSFH31l1siCjOwFzk7S0s6JhcnyRiOdZjNNoZztJVe0Cv7 ju+r1H1l65opclSLveVqZ3h2sjBhNLotEBQ8isG3FOFP94Uvpg 2MZYSdQ/+TOw3J+grG5OzT93bFpC9OTsvi9Edqe/GEO885jX2nLMq+i5WXdLNfKjBqzQ7e0m5uaukKS56vbGE7Ouxd Hc2qhLlajrWYPuuNWRall7Gc1WhqUOlmquy8samhuickKSyOY3 hLV2t7L5uoS+0z8Iydt1iDCc+VkXiTkZgg6hgFwhHcfjCj4DhB qcEIDMwZDQyvWPz26jkKElt9gHERY2xttSi0WtzX99lYc9OVds 3kOCXLWprOf1Fp0+XEyqw9Bn1dRXd8USJvczgMN2rZtOxoNOhi TfWV3SEJ4a56nKYes7Hpek9I+vz7N6/JD7WZelqajdbO6mZucla0nDG1NfU6fc7ndidxv5iFz193mUD31 QhTKvgTNHygMKci3IiDnrm7i0Kvyd13EjsYnn85G+dgkLtWtCl vWxGbyMjn6/jqRotBOKjPZNar45Yunb8hsVvWqW/sQz/gxnXqTbwuMsHpRP3Dmka9sjAtEw1RTeZWdcRkp4NjrMdrDdaYm ETGqTfbSjuY1GQO9Q0ZjrnSak/KkGuFmiX+ibuEaHFaYgauNw2xoyTmMVozkp6nGuzCg9eAgB8Bj 8bZf83ingG+DxtfvuJV7HeUxw7hq6SsocXIZoUM/EgrZ+s1KcPt3UbG2Hfi0w8dvbyszc6HVh76pH3qrMnlbRH56Vp 0A6G5CRlQpu3GmaNd2mSrjTM31XWbw9n68opmRc6CTKsTzZp06 w0dtuoLFYbEBUs0VbUGO3Pp0J6auIKlCxJ84vKM1v0YPRA/zhfT8cxXMi2cMe4p4P8Kbdw174s7HXjrrxh/vUKyAlepzvae5sJFW9RNvEWRNzWSP9tz2Rq5XCuzoZsoHabult oOu/FUO5ucrUviHDUdtohUB+PkOzu7d8unv5hnURh4o5VxWjoPVfQ6 bUquj+E1LbvK5ZmT1RdNfHRz8w4rm5Yces4YURztEL7DK7W5by 9yDdGQpvgDNdGUBPsSBqDi7QtSFQS5D8woSFBQLDAB7wlsVM6z Dfs/lapIHpldGC87eOjjk+mbF03Rssau5rrqhp6QtNmzI8NSUidFdS oTFj+8Tnn4f7oj5yxesyhLVf3OXnXubTGoN+TsadQ7zYrrjk13 r4hnGztTPz9w8J13m+esWLN6bazMYY1cmHPk44/++rf82cvXrlk1Cd2+Ex0flxyaseCehUkquw19UicRkfuDfDEXt xN5WhJyW+kJbPcd2OKkNW6u6LfIcEcRdywEfxKxoP9ITrJwDvM HZR3nTY52uzo94eQnaZPujA/RTU5P5MuPdjkWp6jj0zLyr156+5RldlrqxgUpiaZ2xmQr7WHbu y69WBE6O6/o2RXa8PYO9IXciPjY+HCTUpf3eIFyzz55RE7W+syIsNbrB+N0a TEp98yLUdVVXZiev0De5/o0zA+GiAcljnN3vYqtB8euEEagLm8W7tiSzMevTskd8p3b37lz zVrJ12AnEAiGgEKp3L79vbvvvttsNqP2ib7JJV6cYs/I05jcj/2rlStVlqt73z9Q1mSLTElLSpySP3/xzKQQ1mF38pqImHAluu3IbJdpo8IVdosFDc1sdvWkSSqLGd2vx 5p7uPjUaNZidbAyVVhUlFbJOR12GyqGjEau1EZGh6nQbxHabVZ UAu3RRMZoeQuqj5HyISE4V9sTXMO9xIB4+zX6i746i352dt++f Zs3P4gS9klH3l537t3/WbxgvtVo5Jx2zunEP8bKojEXvk1Z6H5gQxI82+PmJc9alNGqlD RlqFauYnhzL9vdyJgcMo2upPTEn7vifrwsPissWZORylh6FLZO tkuP+lv2nu4fXIh88Zll+XILq29gunuF2OXyyCxNUijXVcf08M r0qfLeRsbQx4ekaqZE8N1NTI+R16apo6xMO75XwH9z78OBisEK 9xnJlPi7skg19E+mUis0IWERkSdPn5n78HN84lT/enz2HNy/Z/WadT47oWc0JDcoMAQB4cf+BjbP0uJe9x53d0OiRo6LnLHhmwu2 KFHDZRiWQR/p2+3oexrouneae3pcR1j7+vAjuUITHiG3WtAv3aPX1dE6hd1kR e0GPWGtfQZrf/3CCTmb0WDz2oOmkoT7mAaLx69/h/sF/YM1d7Yod3Hc5ZkRBoKe456G8E19Yboa94iw+6D/oC9WCHYkbNLjNLbH2SDG6FGxs3f+0nklO0tev6b94bTmkHPl/3nWsnRR8pJQPGLqMJisk1J010swh4GN543VNqPrOVdb7npka7B f7y9kqXdYPA/xf9wfr5CV0Pdxs+jPDvePcBcT5e7rzf71BdgDZhQADOwOmoB4T x8eswgN2L2JT91/xZYvtEfJjWdspl63aUgW8djJswM//I5vihyqfPCvux1KDFX0IM8v64sFUL5CphLDEgGIMHbFHTPBdoQ GjN1IcEBh4kjgIHH0IJHaZdbob3119YmzFz+60mtitAUr584Na cNf5uC56g57fI48BNU7vDoHOZ33Szh0PDDrDx3/F+WBn2MzFOeMhMRHuoEZjZQcHNdPAF19Yst0/8XXqbCJRdwPxP2kkHOHLfoRChs9ED1X3IPHoVLJYCA4Vfwq7hj hx2JB4YsUws+rCbMv+HcSpXtGUtXiffYutj1q6YzZy1QK9Gv4f F8b04cqZCrqug/qWb6jZW+fZklaROyYfkiOLQgHi91I/JFH3P/Bc9bo+7Lo22n41yvxi8KUmCSQQNn47AczChIUFAtMAP3Wn4f1u HtAaJ/YaMU97iYduKIJ9IoYMwrIPwsxSlfKksMS/OOHwqdpyI6Fn0XETRXP/uLpFuGb7i4kQvseZtaMke3qH3ehQ1FNqogZ86fNvS9e1tvKWPt YgzhbNMxqBykuTBXJ0Q+G4HSQHwk9IbQHWxL+Ar+wCT0j/HzkdgRmNIgI8FKwBMSWKXYlxL+ojYib2JjFitwtPNh6x6OcO0j 0QAxe3CM+RRGJD1xmFCBC3Cxdn+KjfoMCr+KD7nNQYh8Se0z4O NegLUAVw9jN8b0N9t4G1xFoUnkYxw5V1N1zw5bqmsXGD4SHwtd l8Uw2ziXQp3FDncH9OphR0KigYAAC6J0StUz3GE1sungI43pHd TUNnyYdoLJx3i0G6R+EaEDumSNUQMzXNYTxPgB1InDbFD9lQyX QeIpTiiMZYVQm2jVuwbh1u575n3NC7BE7RWIo7mQFYdGGf5Afd 4XQaA1dAvhTVKEDNdINzGik5OA4DwKhoaFmk0kbFuZwOMTGhT7 89uwTkULLM+ZA8SNjUqvV6D4GrTZUOi+lCn3YZ3I4tCqVlVfhr hAeqgnjMdyg3Z0N4Rke90zcDQWt8AywP1bsOzgdIX65IlSpMNm dIeERMvRJ/0i3kR850jPCcdQRkPOZmRmVlZW5eXno7ptAnQtq0kYmhe5Rqqy qysjIkJ7yCYuYnDWtqqMtL1oWoVGKn0BRk75/IsiwTA72Wq89MXuaTBvuXyDIPWBGQYKCYgEJoInL4uLic+fOnT 17xoK+ZnkLbGFh2qysLJS1OGnrkzEfFp02dyl/7nhZzVWb2XwL8JCFhocnZeWnzlnCh8egRStHljKY0ci4wVEDBF iGUapUixYuQtstxAXd+Y1uvkZfUPXb0D3oyviUzOX3ZMxbhlum OEDzK0bPDjRYUyrl2kguLNJzffPhJghmNFxiUF6CAGqTrEyiWU oUvTV2scioQsJl6N8ttaElc0exjXzqexQnhUOBABAAAr4EwIx8 icBzIAAExoUAmNG4YIeTAgEg4EsAzMiXCDwHAkBgXAiAGY0Ldj gpEAACvgTAjHyJwHMgAATGhQCY0bhgh5MCASDgSwDMyJcIPAcC QGBcCIAZjQt2OCkQAAK+BMCMfInAcyAABMaFAJjRuGCHkwIBIO BLAH83DS0b4rsbngMBIAAEbi4BuanX4/d0b+654WxAAAgAATcBGKbBxQAEgMCEIABmNCFkgCCAABAAM4Jr AAgAgQlBAMxoQsgAQQABIABmBNcAEAACE4LA/we7qeF+BmfnugAAAABJRU5ErkJggg==

Then when I click repair, it tries for a bit and gives me this (full text below pic):
http://ubuntuforums.org/data:image/png;base64,iVBORw0KGgoAAAANSUhEUgAAAg4AAAH1CAIAAAA RZOEpAAAAA3NCSVQICAjb4U/gAAAgAElEQVR4XuydBWAUx9fAd/f27mIXd/cQTwgS3N2t0FJKS5UK1IX6vwpUqH4VqFCjWHF3AgQIxN3d75Lc 5WRv5ZvZS8IldzFIKCUzTY+9vd2ZN7+ZnTfz3swsPnrUcAwFRA ARQAQQAUSgcwJE5z+hXxABRAARQAQQAUgAqQpUDxABRAARQAS6 IYBURTeA0M+IACKACCACpFBItlGQy5sREUQAEUAEEIEBTkAiMe 9AgFSpNG2nXnv5pSnTZw1wRij7iAAigAgMZALHjxzc+NnnHVUF juMDGQrKOyKACCACiEAHAoZ6gTQ8haghAogAIoAIDGQChnoBub UHcn1AeUcEEAFEoEcEwKiiR9ehixABRAARQAQGCAFDvYBGFQOk 6FE2EQFEABG4eQJgpiwaVtw8PnQnIoAIIAJ3I4GOegEZoO7GUk Z5QgQQAUTgFgggA9QtwEO3IgKIACIwUAmQODJADdSyR/lGBBABRMAoAUO9QCJNYZQUOokIIAKIwMAl0NFVgXaWHbh1AeUc EUAEEIGeEkCrtXtKCl2HCCACiMAAIYBWaw+QgkbZRAQQAUSgLw kAtzYKiAAigAggAojADQKGegG4tQ1PImSIACKACCACA5iAgV5A G3sM4NqAso4IIAKIQM8I3C4DFMtoc7O0KYnapHi6spRtbGwTj7 C2Jl3chVEjRJGDSb8gjLhrtJe2Pu3ErgMXc+oEgUvWPjzcRtCz IkFX3S4CnLLw3O69Z1NK5aR9yKxHVoxyuPFGyF7JoK1POrT7+N VcqdBr3MpHpnmLezNSZxozjvy161xes93IR9Ys8Dfpzb29khJd jAj0mIBhLex3AxRTXdm8b7vq5CGhg72Jn7ckOlgwaaTAUtJi+O I4pknOSGXqsoymM4fo+nrTybPM5iwRODr3OFO3dCGnqbp+KsVq/BR/U0M4txAzq8j857P1u3MoEIfZ0HsGWSM90S1NWpaT3OQ+2MusT0 uik2Q5TcnpHzduia/nf7cLWRFh372eMCYh25S6bf2nhwq1IBpxxIzB7r3RE5yy4PA3G 39PkoObHSIjPZCe6KS40OnbTcDAANX943HTIrKNssYfvlJfOms xJMbh/vtIC1NOLec0Ck5Wqq3T6iThOAwXCAmhifkgX4uoSFqhbE5Jq37 iPtNR460eXUtYWt106l3fyGnq8tMzMrNSr5y9lK/ALY6edLR39ouZMH1ipEtvHnXjqXDylF/f+fhEtThg8rIRLhYew2OsbkfrZ1yY/8ZZVpFz6JvPtqU4PfjFW5Pt+1uvcprCvR++vSPfauwT70xx1Mg 51+AgSTdFZFxCqnjvxxsOFZkOfey1mQ6MmU+wY8+fKE6Z8/d77+0rsRv/6Orh5oxNqK/4v1FYSMqBSKC/tgtUnjoq/XqjJCLC6aGVmEbBVhVpqGZcQIA/jMBxAud0TyaHsawKYxs5ppJjWFxsbh7oZx4eIb+WWPngYpu1r5 iNm9zHxUI3pB/547ddl0pghx8G3HGQn600PfNaQea1M9fu/+iV6a7CW0mTbbiy5YsT1Zgo8vHnVsYiJdGRJd2Un5hrERnjJLr xC6fI2LcrGfSt5QdPlo9d6qn3U8fb++C7Jn/n5zvyaTxw0ZKRgbY9M3galZCVxf++t4jFbEbMGhnk30uhqfxdX +8r4USR9y4dF4GqSR+UK4qi7wgYDCr6YbU2p9XWffq+7OfvHOb OMw8KoHLSqPxsprGB0zLgj6VojqJZDfhj+D/+KzjJ/8o0NFAFWeAW86BA+3lzpZu/rtv0MccwfUaAqjj55boP/rpKjH3ivfdXeMN48aAlT7/85odvTLIGX7SZh06WteqQm0uVbUg6nKACxojIqagB6ICQlib89 fHzT7y1q6pDxcMtIxdMccRMQpY+Odujl01ub4uJU6TuOVEH7vI bF27dMz0Bq4kRCTlFTlw2jWGmg4a69VpoTdGpC1IQr8/IIMtuBjS9zSG6HhHoewJ9vF0gq9FUrXuW0Kgcps9gqkrphlqcF OACHOM4juVwggWjCg4oLPiEtg0rMN2vYFSBMSxHsxzTrMlIJqw dnKbPrL94ofKVtc4ffEqIb3l0zqnzdn225VqT5fhX1j0YLkj9o hLydAr1NAcC2XoCY1cDhrE0c0sPLqfMv1QIc+c9xMu078vrPx0 jXX9px/5UKWY9OMimo51G7Lvo9Y9mWHna9LrN7S0SZe6ZdOhYcBsSaNVj TQGuN5RQU5pQALoxhM8QT5PeSqGtTs2EHgrXaD+LW6pwvU0YXY 8I9IBA/24XyNF05RsvEhqtzeAhVGYqp1FBcxOHcSyBAz0h4DhoemKhAQo +HC2qArgrMBYqEvgJVAXDQYXBsHRVJSNrtIkZJktMrHrrZZcPP 8MFt2TDZhsS/j5SBXTDpGmDLHBtcWaRBkhh5jMI+jO1TVXwwcVEXiFt1maOqko 4uOtoQkE9JbZzcnFydg0YMX2cH1AsfKDqkg//tedipUXsisfnB7c+71TptULQ0cRcY/wtcE5TEb/rz0NJNaRn7Pzl8yJbJ0F1EzPQndqaxMP/HE3Iq21UUYAIJnCa+OzLC3yBH4Vpyjl78NjFlApR9NLHF0f2oL Vjm4vjT5yOT8quE/pPuu/eid6835hTlScc23c813HpU7PN0vbvOnq9ymLkQ4/N8tPzKneSFqeuTDx5+HgiNXjGoKrTJ1MbbWJXrl6gmxjQRdY4d UlqNWTsE+nSXiFw6vKrR/Yfz7Bc+Oy9we292saFB+JTdWmnDxxKwKY888Bg04a04zv3Xqjz vX/tsuDW8tGVksGnpvxaHhw32oSHGjiyOxfemIRUZVK2EsTkFuXbw RPfCTdeFlZZcmnfrqOXswshC6tBIXYdtaaByOgEInDbCRj0X/qymlZ+8hHX0GATM0SdlopRapwk4EiC43CWxVgCuiKAi4IAigKD A4u2AAYcLdoCXNaiLcDYAigMVtXIpKVYh4bXXL1a+fnHri++fg vAOEXWmQzYiNt42QoxrqEooxbG5hrhCoYrmpKLV4A1ALMcMSuC 929ymvK4rV98d6YCiAZDTWVRpvWEYfN0bQJHVV/+44tvT5TA+LBdP52O/WiOC4+SrsvKU4B/bcKCLeXXflv/2dEyGANpYWJnoVN0XccM49DWxW/d+PWpchZ+0QWrMQ9P8RHjnCJ79+cbd2Wp+bOlW0+NWr/Arasi5NSlcdu+//lEAdSK8JYt6xnnT1d5VZ7/+5c/ThbAeOx3fnIiJVs3ebno14gR70y04/vaxtOim3LP7vj1z9P8rVhyZhwfrZowFQJqnWcNzPQ5+efO88mp EBhVtPuz9w/hAquweQ/MCTapS/hn85b96UBTi4fOaVcxjAr/aKiptiph7y+/7k2Rgcg8gjKPX9z++8UqiFlTS2HB5rxIBoFTFl3Ys+dCSWNtVh P8ka48v/9AOqYxD5syLtBS0LnwGFV91ZiETGNOGqwzNsHtxkjGuenE4dRF J7/b+HMCFJwPIt+oDlqz9Rf0LyJwZxGAPfy2cCuiNR49rLyeYBs9W J2dwSqVHMuyvPuBA51iCrol4Cc4hl4K/o93UeiO4Unow2i5DN4CvBrgXhCJUqnOybQfHKO8cqXp5LFbkJB prpPz7b6qQc1gmvLUcvjFIcTLXFt98ZevDoFZk/aTnr4vGE6aZRsuffXW/52psJnw4jc//9+6CfxELJdJM0N5NcLKrnz39pcnSlznrF4+CDTU5g4Wrc0111y cxnecnU2ubnrl0xP4tOc/2fjUjHnPrpnuzvvKu44ZXtB0/ad3vjxV7jJj7XsbNr61wB3GFvPIymgJTlcd/eTDXVmi0U+/ONMeRmZqadKVBYWjSg5/8uZ3JwrsJj/13sZXptjCe5oyrydse+OtLTo9AU7UpWTTfsNiPfgsNFTJee2Hd ZKWOuvnD3+KazRrGd7ZBI8cG2FLOsfGgKFYp1kzbUjeuemrf67 m5Otmpoq09VX1Wucxi2YGW9KZP76+idcTIHhEubcZcjoRPq2O1 uT++s4mnZ4A95Ru//I3nZ7AMOcoP+PzmFhF/onNGzb9fTG3tCi/ks8hzhRfPXW2wCJmRADQE50LL6E6k1BdmgQtmEKfCL3WvhNufP Y0hbvee/3nBO2Qh95b//RQ3jrpHoOslDwbFO4wAvp6QXfcVZe058LTDbKKrze5jB5FlRQx CjmIGowlcNCMcbqBArQ78dYnMKoARjDY3rZNloWpgGFFmw1KZ4 kCX/kAzrPyJqq02D56cNmmT4OGxgosLXsumN6VAjNbXX9TWlhLa5WZ ecD1jGGyi189d6hSRmO2kYtWPzE3lLcjUcUH/0rUYIKQZYsibYjqC7mwz205SGew4JQZf2+5rMAkY1fMG+6cqbJ dOTLWs9UAQVUml0J9RGXu3ScKvOftF+dBe87ypa1ydBkzvIgqO fDLuQaMDF80d4ifadmOC2XgpPdQb1PQy0/9e3sug/vPm8DGfwncsq7T7x3e1fQduvLYd39kUbj/yhfvH+VEquReouNSCjOVmLrOeu497/Lv3t4OtKV97Mo1D0/xqNv+cnxpLSZxksAa0VlaHGY5bPVHD5gffPWDuGbMd9X/1k20Y6XjclgXIUYVdgaNFEcuWff5YvmVj1Z/kcGaxr74xdNhrSpBY+I356nR5KGNfxRijmG+bS19J8Kb2ZjhWk 3gsnXTzQ6/tSkRuBxwp+H3LgtK+HJrDmcTEdHJGjrCwm/yo+9MBjlTpX3x1MeXNZjn8vc/mOHYZtDsqlw6kZCqSiuBLg+PaK8b+q2LMmLq4n7ZU8wJwx54cI IPe6oWVj/7UF+LrpR9a7VB/yIC/zqBvlmtXfF/X1t4eeNamqquhnqCaN1ZCqgMoDNYODUWnucd2uAAtkY6ExTfz+ dVAv8POAD/wU/+g1cYQFtoq6vEEisLd8/KH791f+HVm6KGm/sPD3SziF65alawsPFUSg2MBbcws/INGxE7eUqsj6S14aCKTl0E/V9y0IRwa5yTFybB8YfQJ9oVGtg5ZdaxK8DEZBY+ys+UNBk8eYS eNHR9dl5z23cTJ9sO6/q6jBnepyk6eQFoAUHg2GAJztSkX4NSukSBJpSTpx9LAlYfUnZ4/bd1tMe0l169V6fY9NLXP1Rl7dlfBIw6MXNHQe8Lp25ogiZ6q4D QyIkBIlVKHJwEJIp58JGpvmZYc3YpVId2wb4SqOA7TcvEc8Qwu nx3Psykc3QQdJQQtsEh4BuV0wU0KJe2Or0CmtRcwvRNLmLf6fN c0r7cAn4w8wt1aJ2l3Jnw/j6WpIX9uPHa0u2bYTvtdd/6d2fZFf64H9Qjs0GDu52HpK3LLIS2OPvIEP1FkV2VC25cQqYxL xPakexD/Npa+865gWFazaVjuRwmipgaZU2ok1MqwL0izxCHfvfiQ/YoIAK9JKBrnvVv6oPV2lRFRcO5814Tx1E52bBxB1oBtO9g9AD8 FHDoApVFi38C/MTgUDvoC6LTFlBF6LSF/meLwgCaQ1NUaOUfWHT8pON9K0UuLr3MOLycdJryxsfTSNgUNhd dK4WnXJe8+d5cnZPhRoTa6qQ02G56xsJlWVR5SgmU0DXaywzTK tXa0ivZsMl1DXdrm5JFKxWsqYUI55TFqcBvzmePw6iUnSfLhiz 2uLFIo+uYcVMTtup6CkzaOcLbnJGlHj1WAr6Y+wUB/ysFvLGwcaTrm3ymrVmzbKiTUFOZU2Ud4GZ8mbmm+HwyaNBxn1h/6OflFPkJMMuSqFg4GVVbl1UMm0zXCA/QJeaUeeczQaasBw/n1WGXabFNebzhxTI4TM8t3G3W8KaCXDDBDLML8uK1kaZBTlhZQ g8HVZWcC3Qg7h4NvUZ86Fp4YC6S5/Kq3mlIpKOIrsnMAQYsgc9Qfh4SqDK6zkhLXPr/cIrCDHifWVCE/qqO7oQnDCXE1CXJsKhFHkGOABlLqRihKdt5GbGylIuwx+E91Nc cp4rSi2AdcolwAxLTzTLa1KZLU6KRrKBTiEC/EtD3GvIJ9cHwt2rXDomHB5itxChVcAoTC/6Adxr+C50Nuk+mxV/NAmc1OMXPcdL9we/8xCfwlQWfbbfwkbREyLAgcqaxydLTs3r3rptFREA9AYK2PLEAN rvWIUamnwB3QzpsTsQOjoLahB2ffX0euqkt3C1LDn667rNzxUX F8LvIxpbfhAFsI3Tqu7c+OlbN+2srUqETGw9cdm8wHKJUHt2br uA1IR+6jjmugeOaSzJgX19gLanYt/6VDUehHLh7GGi/maaSEjjbRhj++IZ3HxjmhNVc3PLW61szmvV83y3J8P/QsrwCXk5rS7j6nKm7vD8RtE5Ok6b78bqhmG8ybYP9QLvNSq8dT gSNtfOESV6wse4yLZDhq8UwYj/9Pny3WQPOoTSoYEjXQXaa/FOb33xh0zUdGkaWlQZ9GE5hnmTl1V3fb01RdC08uFZdnAgNcxa BoQ4kcFenwphdB3uRVVd3fPHdBWknTIAKKk+G95G+MZ5ter77c qENJQRTpVJ585NTsKM658R3b775ZwHVBTdOVXQdagprLxdRU9a Jnef4ORS+7kTJ6e/f/uBQRd+tHALxooAI9AOBWzVAgfVxNQcOeg0fQpWWQOUARxFQAcE RRbtRBRhowOEGbF7BeKPDsIIfUej+B9fprE837FE6MxQHHs9Kc zeP0gMH3Fc/id/CroJ0bUYmnARDeoQ53+jyt8Kl5dX8DBlN0vcvXZbC5hkGxYUvP wXN4/xZRCN/iqrMys7VVl84sPtYlsJl8itwnQAtzeFnPzlHRI31K96XebFZGb 8jbknYdCedR6jrmOdb4ExdLbwfY9J//DwdtwCOT2DQdgyBk1tZdRM/50grl9UUXL18avfOcyWiqAdDoEvWWOC0KtiSYZqKYhnloz7x/Z/A+uE45eFZ/AI3MFSCTSZu6yyiZHmnN/+aRuFuMx+Zrlv91mVaVGkCP7rxiPHWG850mzVaWlQOO9J07ra3 nqqso+1iHwuBowugPwv4RhRruLzpmd1VlPOcD8RdCw/nqyXA+a6kN9RWVIFu2Nd0+ZsXfitQ+C772LKz/o+2NrMAugjch7Sb3dqN8OZKQwnB0Cqbn8ZUfXL9mj+lJiGLXnA TsbJOy4hpLKuD6kCZu+3dNZnlfNFg6swtL56u1HrOXNdSQfizK CACdwABfbuPTpxbNUA1Z2WRFhICmFsUzdALAZ5SnX8CjF+A0uD 9Ey02KF5BQJXAm6PaaPBKouWrvp5ocVfo+S0wuUIIUjAxVebmm gcF9ZKntvzUjz+cKKOA70NZAbuhGJ2z9a21v6rBHCvzUa9ueDh Q19HERaY6/aGRKs0DBjuVXOdnmpoFz3vyyYVRtvI4ewKrZbGyfza+DU6bD5r z3GOLYhxFIEuqEr57a+YTZG8VMG+G68WdFVjh/oN541cN4h2fXccMptdzli52AqwKtClOY1c/6nv8/V/yMBOw7gPIw1l5AddxjRYr2v7hOyAu0mPymudXDG8z7sP49QNp4 w9GCDUarOSPFx/8A/wiCVvw/JMLQvhFB9qq1Fyo8Li8n19+4mdwQPrMe/2VJa0rEsgu0tJWJ2fw6jA6QH9FR7dZowmhTqmpK+scRqx67uEJ LZpGW5tdyY8C1OVSnylPr142wk2IKboQHlqsEtP41WuDgRJl6o uKefdQY06h2/jH31g5BtxvPADzUyocSzkPaeeo6K5c2Pw4AwmZsmbd9GOqmgpb vO7puVBnc2ynZQSsU7xUVFFmuV1woCYzB0CkqppC5r2wekG0XW cSG88HOosI9D8BAwPUrY4qpAnXxGbmtKyBH1GA0QGvG3QKg1cS UFvw2kE3oOBHFa2aoS2/ulEF0B86nzY4z/uz25zbuhEGOENLG8wkltKrCb1VFXVx//feliv8cEEXCBK0XSoV4eI3fOq4aZP92wwSpN2wmdEHfyl2Gjlv +X1jJZc25tRSw+9btXSsbsdT66Erll365O/kJnOvweNnzJ8z0tuspROrrc6n/YfPHT1hEtgNSOg6aXH0vi8TqYaz287PemMK7DeSdmH+GFZmHjj +HmMxMw0p+/dnMphl9IpXnprqUvDtL1BSd37dB9hXImrlg6M/3RpXztkHjZy6aNGUMLA6pPOAW4Q/8MTkejAnVmPtEzNhzvwZwz1aRwGgS5zKrymB8Zq7hE9cunLBUG eg7FrPdZ4W05CZXE86hE1aDOZU6aXeDTSQd4fY+cOP/lLmM2XRkrnD3G5syUjahYY6nMuwGbXgvnvG+bdMLehCeDDmasx KrsPMvMbOjAZLGgmRJVywYhU578GV82OcoPOjs6Au481PjsOHt N/TrxvhWSMSkk6xk4IO71JGL3nw/qmDWoZ2XZSRwG7Y3KGHfir1mHjP/YsiFDvf/SzBduzCe+aO8jbvbAjUWS7QeUTgNhAwfI7wqVPGtSX8+COPTZk +q1dypK151lytFGmAI6ERzIVNlyvgcEL3P68WWpQTOOa/trM86afEDy7gYIMfYPDKpE1bwINQiQVwgQisrDViE6WZJGzTp7 2S83ZcDKTUV8XayqMb1v2aocVwr8X/exOstG5O/emVj9PGfLRhmadBK09Vnfv+g+/imwPmPf/CojBLgbZk+yuv7q3BHOd9vGGJ4eW3lJ3m6xue/iyJEg9/7eu1Yb3bfASY40tpZy/bG3rlliS5qZtpaVam3DXYy1Knq8ByyIxiYWBAl5oTJqTJ/H7te+cU3vdt/N+sjlMZbkoQdBMicHcSOH7k4Pebf+iQt1vdA0pRVGrl6crIpHB nDo5bcjmhn+ClTxgLmmJGqRRKrJqLS/oplVuKtsOQTegyde1LDZ98vje3eOebT5z2dNKUlAOzQ0pO4z2e wIilFxiwFy3UE6EPvPfSNH7CJyNNvw5tJXZDhxhxp9ySlMDNkl wI3QZuuvFKryITWHp69+qGfriYtB0Uzq8m1AVc5BQa0INkVAXn EhWY+cil45z1x0M9uBNdgggMOAKGnfpbfWrUjXKBgKTUFNhMHC 6v67cAoofL9DRaISFQyW68RK/fEuyDiHGLkKVvfz32+oljlzLyS2s0AVFDh85YNL69nsDY+gvff h0HnKR+MYMkHK1Vyaszjv+8HUxttRn94Cyvvp54T9dnZkF89iE +A2H5F1WfmV7GCqmS83+elYsjH1nS5WKUPih0FAUicFcSAG7tW 8qXVkMBdx5Lw7mihj6IW4q6/c1w1iw0T9FAXlqjm0LSh9H3W1S42CVm1sqYLsx6YF6Obi+p/K3rntzaKojIf/bLaxdH9WA3wN6ITssyTuw5WQ5vYeuuHN6bRJj7jRofbnurPYbe yHA7r9WWHPzkvZ38GhqM9F/0xprxnaznvp1CobQQgTufgIFeuFW3NoPhYNtusB4C7jNu4DTvQ yAgCei8ADuEgNda3KJ+60Ox+iAqsf+SJxYo/jqeXK4AGRPbeASED5s4Y/pQj1Z/eR+kwUdB18VvfverON1GTJg0fs/2BP+F66YY7AfeV+n96/Ew9fE7TgBPtsjOa9CQmfffO8r13/Sx/Os4kACIQI8JGGgKDHQnDU/2OD5wISGkKdjHh74KfiF2PwUwdIEebwK8OUmDkQZu4X5K9bZES 9pGLXkhakm/p0Xaxz7xVewT/Z7OHZOAwG7MC9+MuWPEQYIgAv8hAh31wq0aoITWlpRSTYDOPkO D2U7Xhw7npz/pJkEBLm0zoPj5Ty2pdxRCN+2pZQpty+QncHnr3FndDFr+kwB+E SUltLH5DyFHoiICiAAi8B8jYNBI36oBytzLS11RbkoI4GYeYFU F2DsWvo2iZaU2nPCq0xH8gKNlrbbe+js9fPxM2dZpsq3HrdpCN 3MWfBOQ6maVuZf3f4w7EhcRQAQQgf8OAQNNAQxQt+ZgsIqJkhU XmQlFLANWVLQs1G7TFvw6CrhsG2qBlgFGp0YqXpno1IVuQMGPK 1oGGS06Q0AK5RTtODjqv8McSYoIIAKIwH+NgIFegPNb2/5uIjeOwwY3qRicFPKb/cEdAOEn3PJPtxsgB/cHbNsZkD9u+eOvufG1/TUtuweCSPhr2vYWxIWiJiXlMHzwTYiKbkEEEAFEABHoCQF9vaA 7vtVZkrZhwWqaU1I0SQpZLVjZxY8hwABCfyTB7+2hc3q3jGv0V ZZu3KATXzfkaB1KgHP8BFl+SIFxhEis1FA0QdoEB/Ykt+gaRAARQAQQgT4hcKsGKLDDq9e8afWHjzuZmHJqDdwCqkVb GJieeC3Rsqes/lypFotUm+nJmBmK1xWYiWmdkvaaO/MWjWZ9Ag5FggggAojAXUugzw1QgNSgh5ZVNqo0ArEWF9IsDu1P 8D0TvBUKvn+CfxFFi63pxvsn2mxKcOty/k0V0BilM1vxMfDH8ABECKLVEiINIaqWq0Fyd23xoIwhAogAInA HEDBmgDJ0dfdSUFM7W48p46ouXHY0l2gbG+GusuCVqfzUJ37eb MsBH6vOv22QQMvAouUKaHPizVjggOUPwC7VpMSiqlnrPXOK2Pr m3q1tkCg6gQggAogAImCUgIFe0HNrc3BzjpsIwLMQ9dyj9Sqtg sVZkSnFEVqWoFgBOKDgAdF2oAWDAxanmY5/uvMdLubvEsCoOIIVmylYol5Dh695SOfIuAk50S2IACKACCACPS FgOKqAYwDdH0vpvUOyJ5Hx1zAMo9VqBRKLwa+vKVFqaZEZjQth Ew8VhoDXE+ATHrTqD6BCeC1y448/o9MKLTpGdz2MBJwHEdIi0xKVduhbzxJmZiA5kGiPBUQXIgKIAC KACPSWQEdlcWO1Nul3T68iA7170GSDAN0QDOM6Ibbs4rWSM/GuYnNWrQQ6pMUA1Wpz0psGZSSd1qlPLf5u3gAFPdmYQICbmJWq tC6TRzuPGcowYOgjAEmDIAA/GfhejESNTiECiAAigAj0ioCBAQqs1mZVcoVcTvss8MGwqp7Hxr uqYdApDJqmI557+GJFdbICAqQAACAASURBVEV2oQPQFioVxjKg 4ecVBmzzdUpA/1Wp+mnpZkbpKwx4TAgEItMaWms+yC9szUqQRNstcDoujgNt0XO B0ZWIACKACCACPSFgoCkwslnK+ExfMmn4qBF+jdev9yQSeA3o1 Ov0hP4BRmBD3nvu4quflueX2gtNMK0Wo8HGUDolwTu3ddHz/+h0RsvcWV5F8N5s+As8AD+AZX1CsgLoiQCvmHfXQIXDJwo0BEE QbQdoYKGDij4RAUQAEeg7Ah2VBRH04KqwstPbtuzKqSq49WQIs WjExy/YxEaW0xolRzBCE94Pwfst2pwTvK9bwwrA3w1XNv9rm3sD3KgEr 6Gm1PZjYoZ/9DyI9tZlQzEgAogAIoAI9IQA7OK3/yMX2qX8VEUR7hH29dcV2LCexAKu0XXtdT4D0Me/0d/HCU4kjH754ZLBIUmbtpqwpIQQQt8C2EKc5XcL5IO+GaplYKEbU vDxMjiuAGYxjIl68UHP8cOBFQrsXKuzOIG0QNClrjvoocDoMkQ AEUAEEIGbJkDSKXk10mb7aW6K7P2Ydy/i0WkIoAXa7uFbcwZnMNDWe00Z6TA0LP37XWWn401wkQlHCAkCv lRVZ2PSmZr4O3WqAvqnCTCqYNUYp2Y17hNHjnl0oamVBOgJAQE/9INOYYDPXoiLLkUEEAFEABHoIQGDGUMkp7EZsWzOhHGWjTucex iJ7jKgFkiShMoBTl3VdfrhJ2zHwbQogrCwsxn26qqQB+dm7zhW dOQCxtBgwymSJQTADQFeUcTHAhQEDdzfGMcQHMVpMZHAZ8aowM VTJc72ICKgJEDglUWLttB906mNXkmLLkYEEAFEABHoIYGOngrQ dDuMjmD+ic8b5CgxUd3EwgpdOw6UAwuUA2zPdZt4wGlROr+3tb vz0LUrYp6+ry6rsPp6Zm1SdkNRhaaxqWV4gWNiK0trb1fH6EFO gwfZD/LBoYLg9Q0fWuO/cQjOAIXUwwyjyxABRAARQARunQD5/o9HQSwZX30DPh/wu5kIdcMLcCevHgS8hmibGNXyBXx3jw52ixoELtN5ONpS0o1Hw FfdiET3tU1V8KOUFs0BlMTNyIfuQQQQAUQAEegVAYPu+K2+BU8/9Ta7UJui0B3wAwg4zxX+w4cOMrdpC6AWeJUB3Ba33XfN0RQjEA no6svbfvz7fKXPw+8/PdzmDvCGcJrySzv+3B3fNPz5Nxa71d049hWjsVUXlb8dtx6x4l TFZ//ecTS+2HnFu0+Psu2nfsntSaULMH3yEyvPOvzzT3sz7B/44MXRdv3ymPQOVM+LW1uXsP37P69I7n3vmaGW6BHqpDoYgsHnz 53WdvGK5Q9MmT6rk3tv6bROPbRpC0NVAc7weoJXFLc/cJrKi38dNZ02JG/7Zfel9w+R0JiZuahnknBUfUGl0NvLsh/aFk6Vv+fLPRYPrp3iROof95RQf8rWUxn+jet6z4qRJe7eXTP8/qmeIrA/JdwarT/C7UmlPyTXj1NbfvK7H9P9FsyJdHN2sTftD0XRA1B6dbvHxc1pi o78fFoye8lgW5GpWQ8f8P7GeefFf/zIwd/+2NpBLuCXvh2S6hTAv25B4ihp7rWraaWNWlbTWFuWfjm9ztrNw 0QjVygpy2ErZ277+pLTq+udwSqOHrttOGX6T699VDBz44fzXW/1PVEGJUFXndh8kJ7zqSOIWf/Y4ELjJ/pVNuNJ3hlne82Krj755eb8qR8uhiO1fnskbk8q/V0EVOGuT7cpFm6YHtlvg+4egNKv25jeY9JV7jll5u9fXXB98R1 3iz5/VLtK97/4m+FDAJDdFl1xB9CipfG/bs4KXbV84QghEIdriv/wWqH/k+8+E2ICpeOaLn20lhv0ikvvVvvhZgFzHn+JCHfpbeVjFCU5Nd aBvl0MRhhZyqVq50lBElBE+sedwuwQ503L1mkC/40fesRKPytU0eF9hc7LgvvXHmGYSg/qwB1HXFsWd1HqtTTMuj8GE7rcGoIypKBXt5naG4+J4YV6Z5TZh +PZsNftevuodhnpXftjR71AtG4se7erDFZ2+ftf8qNmD7WHegI ETdm1Qsx7iHvr+EFTlgC+D3bvnaYAEYndYqJcejuWpQq3rXvnn xqwF0rngVPkXqmyjQq0BBfpH3d2h5E4b0q2zuK/U85zmpITmzf9maro6PNqEbAnrNrlhaq8mtjkFBNg0fHp6NMcG6 RipLyMJ8g2Jv72zqv/d03eSYaN39U/Z7VVCdebXIfx3Zd+CgagjKfTWrd7XNxUZWKRwD/CuddPuPH07/KzcHjd7q8v3dp3NDuqND5HgcW0yUhVpeSqXab7mbdUeW1tej7l OtPHlJFlHN++/Zhs8luvjLaBP3JUddLxoyePnquOeuF/DwWb6j0jwMWReHD7ofpJLzwaZqKpSjq8Y3/x8GefiVKBs7vixff/77EQqIk4ZcHpHXvPXEpRDHnhvYfDQJJMQ1Zao+P4ttTBRbQ07e j+uMzc9LQylWXIPa8+O9WmJKFEHLjEEeo2td6x0YtdRR3i1JfN FGabbsg4feD0lauXc4mhq15fPdaO6VTgNkr8QYcbx0jq088dOX SmxHOUd9nZ8yV+azc8FsoPzDBOU93xp0cDGtvRY2RZJ7fvSAt5 +rkRWPbJ7buyB695MlR6afe2A3lBT7++wINX5B1SHKk99sEbv+ dQIiGH+T244bWxVEnilQyHe+4JB4aErrnxYumFjhDsQQ+TbcpN lVmHB9sY7W0axu/arq0xLFwD+ccaTaV9ebENV/7v7a8u1rOkSEB6LnzztXmORX9/uc9k+dr5HiIhW5ZbqpLzu2UayDNOkNOhODoy12/V6cbc83t3xQkXrx1bt3/rthNFDovfenmWG8RuEPNUkFOmMfvk7p3745WxL761PEDMNGRcl 9oNDbESMMqairKCy8dT7ResHA+RGL0d8D70+/az+Q2UViUnIle/+8RwG7zzBwpm0LA4DGKeYlff9tyZtn80jDytwOMdf+DgiVNxmQ 1E6Jq2vmG7eoG+dCBg2BMYMAYowsTaFCsql9KYPXQ/gycgWWYTPsi6xRfNKfLTpRJP7PJv3xSolel53MSH+T4m15y54+ uTLg+vmlse/5mCvrE3CXg06q8dPZGSeCyBWr6ClKUdO5mcdiK+Nsr35I//V0XXJZaZr9LZsrQVx747aLFyzdsLyktoV3NMXXZh/8FTJ0pZy/R/ttZ7DJsxJdSGKdrzyRbFwtcenyWuP/7Wy8eCwx1FVFl6Ieu9nB/mUNU3jjGNwcUObNmFHfpxTotRx59ukQ0qK+Al/OObUzb3rX5y/CTvt17648i1GVZYUppxgfUrjeGNs5ycNAJ1ZnKleYDH1CdeHVP BuOr0BGCalylt95PWqnDH5zfoMY1pR4/EJ5w4Vxpge+rXLRWFVy5X+Im3fXFGrspPqQuZaQcbLIMUkxZ7U LJJ/9v8Mnvm+/3m9493JNkGFrcOCAKGBEMU7bm1q/+GMS8MsE85m1pecqWIEdid/ntbTsz0ySGtVQLeayz+dnF2KFxQZXqSyrTRlhmH2tWBSbZXrzk 98e1Wf2XcH8fN5831FqmSjpwqs3/JCRQ/RzVrBA7hvha4gTyh4uLMmq6Y69dYVp5z/sSVhKNH07Bw652XQobOnpS1YWdSpRaoCoOYQQ0Ew7fDX2+jFz7 1xL3MzkYRfMG9qiixyiIkiM2+kmYZEhYYuzgwludh7HaMqjz5/e/ymU+/PddEnvzLpnNRK4fZ4J0/UKCjkXzqdMfiMC3t8GiEkqmH9tyo2/qPhpGnlZFe+e2n7LBVq14b7fnGK2djfFv7hu2KEX0xJNBRWYA5 qTe2hTK8/O45I/Jd9NrG9Q8G6uxNoCt4rUocEAEfRD6oy5LLGZJmfOc/9dgkR6FNcATfmdeWHfz5stPU8KZzxxrHr3ko1EwPCGkXM3NWhB h3AiYioU3YtLnDbAVgP13T8PueWjFcInQdGsCbvumayyeS8vNK 5Zilp48teD+IifuoOWOsMYthDz352EP3zQi1IZm6uF+PsFNmBZ nhnDwrvsIiItSeZBX5aY1OMd5mIBL9Y2MXCzvGaed4QzYCY+rj vtucN3TFTF8zXGjr42EusnJwiexMYL0sGrvRKSjKhy2pN4uaNy 3cycLGO9C+laHAKqD9Tz6a4+3oWViHTVs43l1MWto6D1u6crqX qdDOPXjWU2uXhUrMAmJ9wODHSIpWmNp2VIx10c4tGUPuHQM66I wsv0Qyfoav2BiKdtz0smIsZltrl6gZy+4ZZUWLQu999KEVy6a1 0xOY0fj14zQo3B6mYmPeoQ5YK6vYkVOChIW7f0gKmjUY+ItVee cyBREjPCFcVUmS1GvaWFfCsJ442nXDXL/GEpLAcfOmR9iSDmPmrXho8dhwf09rEyt3W6HxnNI1J384RE6b7 GNuN/KRx2d4AZc/VZ5UTClTv/nggMbD3aJtyp9RUKz04pZdDcPG+ZjirDRhxwUuAtTlrh4oXOxk UByYkSy3q9v6j4Zh5Jwi6bdtFbGzY2xJujZPahHoZ9UP8xTbVY m75Iu+XtAdd2Urv0sy3ZIN0trDBTa7fNCUAMeE7zDPlv4wRtWk F9KBi5dPD7bGGnJzKa8hus58+fm4WoH87D9JDkvXLgmRdKRFlV 8vEQeFOUDLBS3LydGAKCb4S9iKpFJJZIgNXytJx5GzozTHN7y0 fl+RRmdspqqTcyn3GLcWNwlTl3Ci2HlMOHQUqkquFQuDolyFmL o4sdoiyNcKpql3bPxiOO5oFyc40Sabtvz0nly36SMcoDysvLyc 9YHNcmcC84D40MmNVFVqHu09zJs3a7UP7X6ijNBjGvNzlN5zFo 3yECsK8pXes+YMcxOritIanYf5g/6esRT9nYJj3ZvO/3nVafpgiIgqO3/Zds4Ud5FxFPrc9IQzFjPUTZi2Ji1b7RFrxFFhPP52Ge5YuD1Pp X154dbR0yPNa07+esluEpxYxDWkHk3FQkZ5gSrCypJP1g9bMcE RN1pPsG6Zt5NYU5VVZxkdy5s+qZqscnHIEFeh0Zxqqy6cqHCN9 dUrZrouPZsKWr58mDBt++FCqjVio7cz0oRj+bYjwkAF5hQ58RX WEf4S3FiVaF+D2heH0Zj167b+o2EYOdeQdCTLYUKENVByVWl5l HOIU4unskO9RV+7JwAePVBn2v66v+GuuEJTcjnHZMTs8FbfHKs oAK6Dwf7A5gTGG4l1jtG6zry8IEMq9h29dOXCGAcRqygraWr3p laq6nqayj0KGGo5hlbkgftiQL8do+syc1ifcH7EQjc3sY7j1n7 80jSL3PhcOVyFCAw1WamNDpE+4GmllU0atrnwWoXYzdUcB4uOz h9OVTmFupKa6oyrhYxbqCPJKMpzspKLW48zEq8YXiyVNsvaxcm BR6NVNrohO6nWzN3JBOpJbcX5k00x82OsCa7ZqMB65cs25SUbu ZGWpSc1ugyHLXvH0O4n1gg9rjn/Wq3DEOCpB0eJNfbRAaAI1KXXKqwi/YEi7iRFjK69crrMPsoT+Imo0pN7ZdMeAKsjuW65FTW0vQurs5i hZTxJ5jg0GM4daB+Mxy+VqtucywaF2/NUOtQBGCVVcfZ4hcswHzPgyUretztJ5RrjYYKxjckHr/msXBJgghnNr7SuMvVGcRhj3i5X2tqMIs47HJpGwWKEk5cFo6cH mBjNqbL48qVqExe7loWetEKmouUFyfX2UaERM2f7SI9vuyTlqz Mw0hqpwOryaxfLRO5uFsAWu+fH3zNI/2AwUjZSJdpDb18cRmPWSKsLE1qfO3V5WuujUZp6PbXD0yovuFx kHuQBBuuq0oQEmV2ouxmmbZSq+MXANEXfAdME2mf/Tvqmrxfg8W1aV3EnIQAd8Mun1LNeeji0rbFTlyZXW4SFAPMQpi lPKsNsTKsuH7qIhYWBhlwtlTWrpHVpFxJqXMdPc9PLCWgYE+tY y/yT27JZ75Hu8aWS8BVgfMvJ8pNrxAHa7DO7Lx8/fMXsgY3rRptZ23qNnT9EtwJYW51drW3Kjz/8Z5PKbcysEeLKBm1Tw4mdPx+pp+lSNWeSvu+PGhsmW6Yhr+37M 98hMlqbW6+h+ePwELlc2yRvf3F90KSJknZxjjVTtMmmFama2ca i4ibGxzR//2/Zg1c/FQ50YnOBocD7q6xGTopunUzIauQawxs5afaVarshwfom/RYoXJP+T6zakJ4m92qpWfgyYEVSFcUXmYYsBmlRZSm5SrNgdda 502orqZEUof2/REbJq2qrs+IPn+dmLY+FIGl5N9yip05oKy7jeYEzDgqvVdvERE LHc4fAGIsfoJ49whZeSRXvevvDrJkf6xduz1PpUAfGWoo5ZXFW LW2ibSiITyvAnMzBcEIlr7h29lzNsAfnwdXmRvNbHzDY4uqN4j DGXD9brDw3VWoRasHR2rrkXXvUs9cs9wKu61ojJKvNG9WcojSv Wm7H5l65WkiEThlJX68wC1huK3Ycs2TUPxu278uLhRZdo6Cqra gGWlG1++dtYTEhdoTQMxL0qLoTr2NxGI253i8MT6pjreBzR5tj afUaTPdohLJ0+6d1Ml5WpVSZpx//fW9+dXYlY6bJOLov2SZ6fKxF+fFvNx62euz9B0PaLA0dy39gfz dcVyFwdnZSqTW6v2FDhvr5B97FiDhl7u4NH369/bKUU2RfvnQ9tw6z83K35ipO7Y43n7F4hKOQU6Tv23tZSnqNnjM 9ytHCXJMdd+bM5Zw60n/y/AmBlu0mjNH1SecyCN+osbNnjfasObDtuuPchcMcSLr89O4T+ZR NxNQ5ox2qEs6cSpMqG5jQRfMjW6aig0kaKQnVIq8R8xdPDrUXE ZgyNz5T4xgzd+VC/8rr5c5jZy2cP86PzbpU7jBq4dLZg11FDWmtx26maiMXB1qT2nZ x4nqyjQl3Jwrjz5y+XtGkMh+8bNlYdziJS5O/z4jAYwMkN2y5hIkVZ3ijuuhiJjlsxkhvC4N+ePufCBMDetrS49 vjLecsHu7AFh7465JkzuKRTkJN/sFdF2oxpyHTJke72xlJES6KU+dfunA5ucpi2JJlYz348REYCnf HTW8dsfG8gAa/8ODOdL97FkQY2rCNxw9Qt4ym2IacKx0Kt+epdKgDMEpOWXL1wr XkKknsvOmhorIrF5JSC7V+05ZMbpknZ1wez4arWTeKwwhz/bEf15yx4+8L+ZlnD54CQ+Apj9473AnO8jYa84RwB0Vy3KWEtEK ZOGDirAnhzmKWtY2ePNZPQuIiO09R2v59aZYjR/sDl4WRghgfSObFVwTMf/jecdYZO/bXDFk8fZCENKwS+uIZFIdRwcb5MmnnW587H21G66PhbiPp+LQS msIryXUW0fOWT7XPTWryi508c1KMr7UQjDLyTp2XecYOH2TX22 nud3Hz2Ja1grzcc3EX2vSC7gB/aYwPmNgA3jsErhux5sN+2tjjzuYL+msy0smBb1loaV4J5uULXH 29CRzVUKMwc7SF9Y5TV+ZLrfxc7+juyr8jMN1UISWdHc3AjvOy cpnYxckMKCZOXVPabO3xrzyzVPG2N74TrvnfIvfelXdv6gYcgN yOVHogkybrh+e+YJ/Z9ERwj3cj6DRWuv7i9+/+UDzshdeXhVsyxce254TfO6VlwwKmIe3QviynaXOHAius4uqGZ 35xeO3zVYGt8x86jfM2guJomiNJg95Op6INqB/Axh6vvvE8UAnAII9zcIAB9u0jzURgvxu+U6P3uqEBxQV4niUuD q1ZJm39fXuffVxk7cSbJUDATVz8XXsfxe29498RmLR0ddQxEtm 4O7XxcmybXnB7IWBUyfksnyUv8KsK+i/cnlR6IL+2OjVb5bbYrdsmuwdxYaTdyCc3+qWcPr7j1ytaeRPpN 3mYboSgrY77bbts3COL/PglSKrCy8UWgxcAO1e34XaCAi/baT+g6Va6gXWBkAQ7tsIsQ20KlAaHkyKoWYHiAKcRuoFVGwZub jmtUklpG3KOHxfOvzeqvxYe355Uel6KbGN2ksw+lp+x0ScBzG6 NnH5/ZLu4qKJdH3+XFPHqIqh+OU1Vwr4j9Izn7gvoahhzp4HqEzj/8UjEQFWArTzAaAL8x+Fgp1dSLCL5bV/BUp2+qkH/cUhI/LucAF13dtOrP+dbh0976NHFofxs5H4ItyeVHgtOS1OP7jpSTLF u5/cfDRo7KbKfdkLCRbZe7szZj546a+ft6+IyaOL8NWvdutwu/w4D1WOid/eF4EVB0C8BPBPQNQGsUAT+1aJw/hRG4KzrPesGpK/i7i50lDtEABFABHpBAPgqPnzvFWB7AuMJeBv4YDFS56rgDVD91 L3qhYjoUkQAEUAEEIF/nQAugAoCurSBaxtjoFtbRPLnwP/I/vSvlw8SABFABBCBO4EAcGpDRwXHcgTHYDpfBQEHGuzAnf90J5Q LkgERQAQQgTuIAAuGEtBTAb0TYMYsRpAmZkJ4ggXKAgVEABFAB BABRABjoJ0JJwjdqgowEwpMljURgt1lwZJ4hmXadsxBqBABRAA RQAQGLAEB1BNgaQXLL6OA82ZJMdAUJBhfkKyWUQxYMCjjiAAig AggAq0ExEIBcGLDKbNgVztgdQIzoLRa3FwI1AbOgLMoIAKIACK ACAx4AvBFRrpl2gzHsKwWbIPC4WJSQBMkTnKGm2sOeGAIACKAC CACA4+AlmHBdCeG4ViGARsEgv/BJuRCQkSKcPAuUDSqGHg1AuUYEUAEEAEDAhoWzJDlLU9gChSLg 1lPBNWs5hhCKMLFpmhUYQAMnUAEEAFEYOAR4Gg4mmBYsPsTP0M W4whFk5JSs5hAJBSjUcXAqxEox4gAIoAIGCGAt6gIsOUT2AaKZ Qiwx6ZKwao1Ygz5KozwQqcQAUQAERhwBFj4bgoCOCXgvh7AAsX ipEajqq2oVjSrbR1EmP2AI4IyjAggAogAItCBAJweq9vrCWzrA VwVBEZK1ZpaebO4tq6+zsphMCKGCCACiAAiMNAJsDS0OvHvp+B nzXIcUa1QUThuaWlhgrzaA716oPwjAogAIqAjAF9iBP7ARoHA/gQNUJxW6+zuEOTrhjNYPaKECCACiAAiMOAJwB3I4S6B8N0UYAc ouMGsj42Vj6uDrbO1oAcvvx3wABEARAARQATufgIceEUFw+EMS +g2khXgZEiwp8ReoqEZmtbe/QBQDhEBRAARQAS6I0AICOioYICmgC+oIMAugWBXKJwQmtlKxCa EtLv70e+IACKACCACdz0B8FYjuKWsAEyZFYBxBViPRzbUNlEa2 sqUtrbG8+56ACiDiAAigAggAt0RAL5s8G4juLgCx8E/QGeQzQ3NarlSXVtr7WyGje8uAvQ7IoAIIAKIwN1OAJqfgDubd2 6TDAZeWEGaD3VkNCoVcHFbmd/t2Uf5QwQQAUQAEeieAAfeU4Gx/JIKgsEw8Oo7cuxjb9E0zSlLcab2Qmb3UaArEAFEABFABO5uAkB JCFm4+k4LXpGK825tW6cIjjDF1KWcIhnLBPoDBUQAEUAEEIEBT QDoBhCgwuDgPlCmBPiHoXAOvBMPrKpACysGdOVAmUcEEAFEQEc AWJy0BEsLMLD7E5wwC95UwdJNLEOx0IWBVAWqJ4gAIoAIIAKYq QATgaUVOHRUgDlQWrC0AmObMUYNJkOBJRaIECKACCACiAAi4CY 2tSaFJIEJgP1JIMBwcEg3YEwjxmkwjv4vAuI05Rd/3/ji02/uKNDw5jUUEAFEABFABG6JACEkbUWkj1hoLwSrKjAGvs6IrsVwc 4wArzbqB1XBqasysynvCE8zOO2qbwJH1RdUCr29LIGHRZW/58s9Fg8+98n9aF/cvqGLYkEEEAFEoIHjnAjcnCDUBMvROAGsThgjxwg5dHUL+q5Tz mmqUuKuFMikmceP5diPnpwh0dSVVVL20VMXTg23uyVDF6dM/+m1jwpmbvxwvitWdWLzQXrOp45IT6C6jQggAohAnxEgcFbKYjU cq2E5E7D3E1itzYEPAownKH538j4ITFP6vj8vm4xbOHO+KL34y JmQhQ8sj7HAMW312U2vr19XsHb9U0Otjb/Gm1GU5NRYB/qC8UKnATcLmPP4S0S4C1BytSmXqp0nBUn6bsTSabLoB0QAEUAE BgwB0KY2cwx4v5GIwE1wTIwTJC40xUiwIRTNMZ3uLFtVVfXqa+ soCqiTjkEkEn380YfOzs4tP1Alez/5qWrBO6uDJTiVdz1X677IR2d8EjqNWDJx97qD+68tj5lka0RXU IXb1r1XfN+m13w7JtL+u9gtJgqe4RS5V6psowItjcTVdQzoV0Q AEUAEEIHOCRAkJmQIgoWLtoUYbgENUGBHKE4LXrgNz3USgCa4Z 8mS/QcOuLq66F9SUVE5Z/bsG3oCtN5pOw+Uua0dBDv6dG1aWpPDqKC2plxo5WaNYWoVBd/aSlUnHT968ui56qgX/vdQsClwnDRkpTU6jvczvzFGoKVpR/fHZeamp5WpLEPuefXZKXb1iQe3H6qf9MKjYabqkoQSceASR509 i1MWnN6x98ylFMWQF957OEwvlk4yhU4jAogAIoAIGCVgTQoYnA VbPwGjkwlOiOBkWUbJsSqOVfM2qE7DjBnT7WxtG2QNbVeAY3AG nNe7h5YVlKt1L2TF2MacxBpJSLhDmyOBUcqUmMjRzVLQnLnj8x 2K4YvnhonkCjDIUZdd2LH5+z2lLJP+z9Y/D6fLgEVMU7Rn/edJvksef/Hdt5e7seLgUDL10J4jhw4mUH6uYiBtdXoh6z3YnV8Noq049t1B 8ew1b29495mFg5Ce6LQc0Q+IACKACHRLIFBoMspMMsrcMtDM3E okFJA4wakaMC2YL9sE+vtd3A+2on366adkDQ06MxT4bGhoAGfA eb27BNahsx97eWWIKTAPKQuvlgoDY9zaFvZxypK0WmHgSF+y7O DPl52mBThOaQAAIABJREFUhjedO9Y4fs1DoWa4ifuoOWOsMYth Dz352EP3zQi1IZm6uF+PsFNmBZnhnDwrvsIiItTRMWbmrAgx7s TbnFhFflqjU4w3b92iay6fSMrPK5Vjlp4+tsjJ3UVBop8QAUQA EeiOgLdYZGkqFoqFYCEesAKpwAtTgTMbZ7W4Vokxqq5vB4amxY sW1dXVg+1pweeiRYv0TE+6WwlJ8ITxIfYi0HxrSuLzyfCxfkBr 6AIrSzqRLxk7P8q04nxcrUB+9p8kh6Vrl4RIeFcDVZ2cS7nHuI HhAgxMXcKJYucx4dD/rSq5ViwMinIFhiaq/HqJOCgMDlTUxYnVFkG+VvzdpOPI2VGa4xteWr+vCK2u0CFEn4g AIoAI3CQBDc4VazVpalWBhpJpWQb4LcBO5BzLcAyFabtfV6EzQ 5WWlhmYnjoIxCnzLxS6zF4UBqY+8YGTp+zar5q8eukgsbwgQyr 2Hb105cIYBxGrKCtpYsAiwKzURodIH2A6opVNGra58FqF2M3VH OdUxecPp6qcQl1JjbS6MCFN5R7lKsI4dXlaMeMW6kgyivK8chn rOG7txy9Ns8iNz5UDHYgCIoAIIAKIwE0TkHKMJcs60mAgwWkJz ISE6ypA00phuAgjW3r0XcSuM0O99fY7BqandjfR0pRD5yzvXzv LE1qfGI206OqJE7m+T7810csEpxuBKlBLZc0qaV3ahYQa1/HT3MBM2uxqbVN+/OE/m1RuY2aNEFc2aJsaTuz8+Ug9TZeqOZP0fX/U+4XhSXWsVf7Jbdm0OZZWr8Gu7fszEyilS4Lln6wbbWZt6zV2/hDbzt3zXeQM/YQIIAKIACLQQgA4JCQ44U/gFjibB2bN0iyJAU1BCDiaYZXNGGbVLSpgdProow/BqMLolVRV3JZNW8+XKoW2bi4bU3cJRWKxuVPI2BmzHx1tppvUS jrEjA0++PfWjz9NHzVt8fzZLtBcpbX29rQpYxnXyfcOdhThXJO nt711NeEwZtUiwYGPKsfFTpg3OcSifMcRl0APx+AZYwOIxLLja SYBoxaMlSQ1Zf3817e/VoZ4RK5aeWO+lVH50ElEABFABBCB7gg4YQKwmYdMwA8jWFwL3p yqzXgXjCfgFrNNDQeLQqdMn9VdJOh3RAARQAQQgbuWwPEjB4t/3tgI5sWyuIbj1BxHgbfggelPvMpo6fPftblHGUMEEAFEABHoGY F6YOoBzgPw/juwqSzHga3IScKU1cipRplMqtJgWGjP4kFXIQKIACKACNy1BBi GE8GFEGDTJ/iGbRxjySalMreouriqrkahdvZG1qe7tuxRxhABRAAR6CEBM7CB B44z4J13QF3guBAXkueu5KTnVWu1NEdzzt49jAddhgggAogAIn DXEpAQApplwfoJVgBeWEHgDEeeuFpA0pwZLiTAGgsUEAFEABFA BAY8ATMOo0mSIgQs3IAc5wiWkFNakUCIcV3uFzjgwSEAiAAigA gMHAICkQkhNhUIRCKMIKG3giAlhBlH4QICvDq1072TJJbdr7cY OBBRThEBRAAR+E8TSL5+VSe/r3+g0YyYWFqzGjVHU0IWZ3GBltOSJhoCvKqCEWnpzt9XYTQudB IRQAQQAUTgriQgEoGpsbQGpwUkSQIDlJYhwZbhuIjBRRxyVdyV RY4yhQggAohAbwkAI5OZGLwpFWMpFmwtC2dAcRiYPwu2TWIEAu TW7i1PdD0igAggAnchAYIUCgSkSGhCCymaphiGJYE5ilECYxQH X3SEAiKACCACiAAiAN4ChOE4KcTAW41oMaGlSUrLNato8AoKBg 0qUP1ABBABRAARAHqCEGACgmDgYm3OxJQzIYgambZWztWrsOau 3pd6m+Bx6tKzm99ZvfK533PV7Y5vPX2Oqjz/47pHl61cHyftnzdacMq8/T/tL4DvEmTVcoW200EaeA34se//98z9D350pg7srdJN6DkHbc2ln15/9JH1Fxs7TbolLT1Ru0n9Jn7uuRj/buRgF+QfXlp5/+sHKrp/UctNSIpuQQT+ywTMLTlrB9banpVYc1a2nKMzqdWSWgxuHCjsvG 27kWOOqs0tE/n5WvXDSyE4Zc729dskT7zxf4+Q8PijlmMjvDmqsbZJ7GBv0vLi JCOXtDvFNib98e0Fz5VvfX0vjZtbtI2f+jA7dOWp38/ZLXzA1wSjcn9a/Vr+Az98OMHWUD5GevXPv6pGP/zmV4+BCQVgNYth0JNKn4nhhXpnOHX+3u+OWS18ddNSsVk37xbX E7UtBra5NKPMYlCQTaczprtMvvXH9mJgVG1OH9aW3uSxG2kZ2f Vtv6T4LH7z+1USE9JYIXQTAfoZEbirCbDWdriFBbA+gR0DOQHJ iYRgER4H35qKCyjw7opuAtec/PXaFzYereyPbhhdcfirf+hxI52BGPrHvExgTBC3+Y3H7pmjC3M X3f/SzmKw82GPAtd45dtPzjlNH+FqIrawgK/H0IU+zA5dffafqlFLR9hDhiLX6AAiL67QyAto6crD67/KCR/jDZSccT2B6UtlwKGT/HLNqVs2nHGfO8ZTYm5+I4NGr24nKqety4g7fmTv5pdXPvnpjr+ +ff/Fx1Y9v+l4+c2NMNuLgfVtbelNHo1m/MZJuvSfdzYkh86O9bBEeqIbVujngUmA8/ZmHRwYO1sc/FlZYqamBCvAQcMPOrg02+1IATcftGTt22/Mc+tWqXTAy8iLUnMbuzS2MPXXz1W6jg6xBC25/jGMiSo5+stxatSTH2/5668/f35/vgcYFjgGOoKXbfckcE0pBxK40GFu8JV8+uGms9MxVVZ2+e/kkOnBrb15saOvDSOrVxkYujT5e3fkuU4Mt+qiH6snVUcOHdNt+ 96csTeOjRoM3zneTdATlVMXHfjk7d/KPcdMGmKjZNxnPPjYmjfefcy35OQvu/KhGa3Xob0YfYZXJ0fP8tiDmoZpy84cKXYcE2mDvHO9LmJ0wwAh gItFnIAAm3iAVRRghTZQEQTF4MDFjbFgy48eQDDxGD7ETdxFQ2 csDirvl2df2lYNdj7vPHDyrIsVdjEh1lAovWNwB5X/16aLg+ZNCHa1lZiTtefPMIP9RLa+bmY9FENdHJ+H+Q3zMvI+WK PZAU3ooa8++ilR3p3FvzU3dPmJ44KJQ+3a8keYWJmCiWYGhg2q/NLVBpfYQZKuJW+VqgOHzuFpyq/mC4IGu3TUhYZ33BCVU2Vtfe/XukkrJ3uKlfkJ1ZKwCKBpOK2iWYs5elt3r3QMIzcQwyhew/t6dMYgcmN39aSmYWxj1vVay0GB1t12jYwlgc4hAgOBAMuAl+DB MQQ4YFmw+o4lqxpwnMAkIszWtGsCnLr8yp7f9tROf/OZKFN1RcK+33cUjHr95aHKq7u3/nXe5OFP14abgBi45tyjf2w/du66PPbNz5+KssAxRpqeJHOeEgAOWwNdn7R/5+m0rKTkYpVVxAPvrpttW3ipSBxyvxMcKaj0joExqurKxYI6ST ODWQnkyX/vI2bMEH9yxDHIETaMnLoy+eT+PccKfcb7FR8/VRT06jdrQzXph//6Y9f55jFvrV81yERbnZyjcZ/nb8ZIUw5u3XpQOuvjd4ETQS87YbI9657dkqkRCTkscPU370/SFF25mOK04oFoCQksX1cP7j984GRlzBufrQ4H6skgg+rcQ1cdx s0xv4GPY2lOZGnZ0RLENmQl1ttGh9m1tcMGHGbZ17ZBNmvPwVA SkPuS87v/OXzkVJqMiHjFC+KHTJrzDny3XbvoxYXeHXWHnqhU0clzNYJoR5 AjkE4hEbDQQ0TXnd/yp2zsC/+b6QJFNJKigcCz3UVGxehhbcFoWcqezVtP5Mg0lFIuiHnuk+dH 2bbrUhjPI6gWBlXIXdSjmuauLUooJdznmeTu/XTzjvjG0CfWvzzJUWA0QoOyNsbkRrmjI0TgbiEA5sQCGzlcoA1 zBD8IazMCrNtWaImmrkYVdN3lf7bv2/PPJU2gu1CadGDX4cP7zteY1Bz+6rM/465cLbaIcOM77dqy/Zv+MVn46sZvPnnl3jALTFV85vevvvi7iKVT/v7hp73JUpCIOv/vtz9ICLj/2Tc/2bjKkxGHRTuLqKrkPNZvmCeMRP8YSmli5x4aDpwAdO3Zrefd75 3l2pAnk/CDCqYhO62eVKVeK8fE3rOee//9l5b5VP3z0aaMQSuee2iEjx1oxcAQJTe5ztIHu/DDJ5tPXruazQX4SHD97JhQJVekMz7d9veXK4aMW/38VOArYVnCZlCIPckp0n778Hf5qOVLIkVNchpS65BBcEZTfD7T dphPSzvN1xRWKdXaB/Fqjw9AoSXs/fXHb7/dXsAQlcd+/XV/iowx5BBBJrZBBtHpczAiCVN34fsNR0TTnnpv/SpvofvwVlXMacrO7T+TeD2j3qBA9UUFvQUOI1gtg1HlVzObBYo z3274/hQ+b+PXL4x3BIrCSIrGCg4zJkZPawtVfvizLzMjntr43Q+fPR3 rMXz1EyPb6wljkUOghpI4MT2saRhVkZinYerOHyt0njjVH6tLv l6lNRKhs8iwrI0waS1i9C8icFcRABuNc9CsAvQF+ANNH0tYill na8LTTigCqy06DaT98AULBotxl6HB1kLbqDmLR9kLhCaYafRDL z42ylLoPnIQb36nq+IOJeRkFTVhVj7+oPOMm3qNXzTRFpOMWv3 CmtWr5kXakkzNme/3srMWhJjjXFP6+VLLwZEOJNuUk9zgPNwPGvv1j6E8Asepb36we oxFwe6fcoatmOCESfMraVsfOyCtwHrQED+msM58yJI50S4SW98 A9uSXe8g5M/0tHMY+8+w8X+A8VhVfKwWzuxj/pS+umeEktA0b7CzE9LODqZT244fbFvzxdeqIhyaCVpKR5hRaTp kXaKIt/ufbOJfZ0Y0nDzRMfXV1JBg3dMwg0B2VV1KF4V7thmRUXQnjGXR j8ICbuAyZt3LFeGutKPKhZ1Y/unJOhA1myMHZ+QZkoh0HQ0k4ecLmX8rHLB5uR2qrs+slwW32FN wk8P6Pvvvhbd3IQL9E24kq9lv63g9bXhxsTtemJDZ6zBiuuBxf ahfqa6Ub8himaLTgjIvRs9rC1p395g/ZqMn+Zjhbd+mPM1w0X/43gvHIgSvLEJ2DsIc1DWNkWalS62nPvvTonOGh/i5mOKhKhJEIQdekQ2XGDJl0+rygHxCB/zwBsNUTmKFJcCywkrC0lgCdSwHLWoowK2G759Qgo5rSy4XikEj Y48RoaWamJuT+VdOCLJmyhGLLmHA73vBLOo9bNER98N3Vb+3MV +tM/VTltSyN1zDPll43U3PpUKHrxGjYe1QWxheKQoYAE4aq8GqlJDQ AOiraHbcKQVef/uqzIyU5uz5+7eWX399VKXR2NteZKajKpGzab5SfGbxUW3H6cJn 7mAD+iy5QVSl5dPDyR+aG22KyzCzKN5YfuYDBQGt2CKuwMZ6NJ 3++5DJnGJSKKj55wX7xLC8RVXLqTLWg8fi2a04rX70/whImaJBBTlmSqbB3k+gbTajKVJnPaJ8OvhFtVVKGynt0i6PCOI cbUrXjYCgJJ0vYl+Y0dbANjkEClFuEnqMCF9m52Rjq/fai4ibOvi6moLPQkHqxxmX8hDETAtjMo9dlOle8YYpGBe5cjO5 rC1MXfzDHfix0LnPyzPNlNjFBcErDjdBZ5EYlAbf1qKZxyqJr5 SbBsdBax8qLcxUuo4fa1RurkwZlbchEX1p0jAjcTQSAPQZjtHi znFM0gd40odESZgJcS4OFeEyTysBgoZ91quJKssprqIcY4xhan n2lxml4AOgE0rWpmYx/NN9Q0YpGxnnKq1+/M0eSeT6riW90GFl6oswpxh84KujmRjWryLtcJvZ0B0MKZcGpvY lKl0h3oboy9WIe4xHhLGTkpZnpCQWtxwUyKBInT/nrl4JxH/zwzScff7z+wxenOxFWri1+V7o++arMbVQQ7wdhGzMTqkzc7Fvc 7rRcqqSbcpNlTsOCgCMZ2J2v1joP89X1XPWzw9DVF46WOAz1AT YtqujwDunsR0fbEmxTbkqdScCElU8sG+4kYuUlhY0awwxqa3Pq SFtzfQ8p6KbnO40L1nPNQBDAUXGt3nlEOK8OgXnHGIf6yrz4Vs iq0sRWDkWJVxI7SNKUe77AIsQLYiy6FF9vH+FljlENdUrAnGMo 2qhD3oioQCxp0iVu6uMzPGyDR3qxWceu8+sTDfPe0GhM4JL4Y/lGxei+toBVgJdLRV4eErrm6t9fbU4hg8I6TOBqNp7H5iZjktTX K+p7UNPqK/MTCjifEb6gN8HUXT1e4Td/spvGWITVxZV0u8psyMRgSh/YsrkNfWfHsPRRQATudAKEVos1yLhGGaaUE83NeEMdGCOAdxxhA gGn5QzmduplB7bJNaxVzuFf01m/cZ5xxZbRj9oIME6aA1rnYCrj2La4g3svmj/6fx9MMLOx850MlhjwzSdVlVGpbcyJ2/NTo8pjwoJxJuVSbaP08B/f7quj6SIVZ5q8Y0uVHZMuVZOXd/yU4xQzVJtVp9bCYxsfX/eaallm3CXV2KdXDdeNWzBtTXYta2JtqmtxG9MvVjrEhtvyabEq WTMrL8qubHJgMy9cyhNEzh6juVZpGQU9HZi6NKEYs51WGffPOW zI9AC97MyaZV0k1TRVVFemnd97ilvw8BgoO60Cmk1VL1Uo62uS zsRXOfrJ/5+984CL4sof+MzsbKEtvawsIE0EBUREosTYQY09amKLCZaz3Bk TY/nHqNEk90msSTRGjXrWBGMUQQmoSFBQURBBFBCkd1hYlu2zU/5vFkxElmKLJr53XDK38+bN731nb377fu9XNq/Jn7Sz9QTBtoQKaZ5qCy2y6nq269gxrX8hA0VVkFJl/VqgflEGXlNNBjjU9eiDptXSFixknSmSIdEgeiYB/rTuYUnEI8egJZVKlWlmzL4TeZXZFZSx9vaZE4R1UOhA7q2jG7+ 5G/LFl2CXukWgB/9qKypDVKfElA5fFe4PVKnAL1jEHAe6YnioDf3I3MEdRQYErhI2 VSlVtm3FGKDr5NtystLMUysl5AU///C/PsF+Nhxe90DwK+ThRjYanuMwFwOS1PmMHm3e+Tetroc/crdR6GuJqCVZ5/Zf6jbvoxF2SHXCo9/JY2t2xGd7Lju0ddifX+a2TMY7PSQvoy2N3bohyvKDbQv9jAnDx 52ERj7yvOD/hAReHAFUo2DqKhGa4vCNEJKga8pxEKMNNjBw8G7sMOyBVitwR+/u9r0njPAxyv1mH893YjfwLgLhuDWEHMtSh749g1+Rs+vg1t0Vv t0DFy/0MW82yeCWbt2tSmjSacx7/R14KCNzdbezrMLshy+ewTm1tnzEoLBpY3zxGxVx2UY9h7wT6mb cdNXB1gUcT+tXvm3xxttmgZMXLQ0PbNYFAByjbpBRaH3C0QhOk F+gL1JmM3LaYL2/DrAO2QaEuJ349edNm4qGTnhn6gx7nq7wQDHXZwwwcSGMqqpcrl LeKLJZMNHHnCpM/GM6lmD/JKC35dWo7ft07y1YyIqpH80+eETvyEN7123MHDJ+5rS3HOSXbw jjHp0grdNqVU3sr/mWpslPLA18c/zDu9zsGW3ZjRKL1+eIW+xCHKEBDn5mpUeiWyB7YzdKf8toZmKi Kx7R+8yfkgAPArl7TwduTZVw8sIFVduLtN1ce4cNctG7Z+FqJd/ODDi5Nbst/CEX0KR/igqi149sPXC1pKJSaurmW3/adPb4ANturw8WRUScS5UMH233yNzBHQ0+OF6m9FKuATGIwk6+L TPGdtOlXbZ3931zZnh/Ju6jRvs3HtmoQDjmhufIyGrbfoX8wJZNaVe+aSlfHLfUnNuwrG DQuOlzN0y0A8+aafsspgf1JzfsOrR1d+WfX2asDZNW5jKUA9bE jLUbDr4K7R0/9DDgISTwchMA2Z+A6UnexO5pq5VYfS0a9oYzDwPJoRCZDlv+7y 9Hjnqz7QxaV8FjCGm13NjBmrXzMOqKvHqLHsCg1Paql+ATsqmi HhfZG7Nqi6y/V4S4ebLb4c+wEfk/Llh5b+a+TSP1YRVk9cVDqe6zxz3qpkoUHlq2nbd623TnZ3v7tj NhKB2NcUGe+TbtEVHbnAcWmfh1C76tHLPlh3leer9nQqnjdhb8 3XaY1p8Y/LbQ0ozIX++Kxk4dCCyX8qvrw3fbb/xxibeByJfOhn+88wypVDHGJp3syj3emH/2ZkiSwUE1Sba1d/ykY8PrIIFnSaDjKngX4mLo6nKk5D5aUw0UBqrV0GCvAhiRWNdI GiE6XFU8JCbKsxTp9QRoqJGj18uqJ4B4uNCxWU+wx9Zez1pPgF F5jsF+xuU39Ek8GE1R/AXt0JEuj5p+EG1Rwl2PWW86PW89wT4RjkE98Yio7LNr0zg2Q/69IFAdvf1oJthoIsqi1y/9KqELCQ3bDPTwB22/LWTV77t/vO83c0aIiF2/qQuuFJoGD33UC6DDQZ/0JIqbPC89AURCH+iJDo6fVHJ4HSTwlxLgVBSiDQ2oUg02tFEdC K/AcS6wCjEMB7xiYPDqkzwL416Ldu3nCk2Aj23a2cxu48ezHroPG qNTKrU6aXZMDHfa+0GPbF88yd2e6po/RG13FFw06tPvHS+ev/7TnktyGd37vTkDWraI2r3kcU8QBcfWb08L2DizOYKyMuVENDnx k/e9H7XYPe64sD8kAAk8OwJoowIjEVDgCDMypnkkhyRxOytjULJC oiAUhEG/mWd383/oSBwjc32gNscqaMrEVnMka+O/XLorzzJg/KKls/yaXZ9eKIQHonYkBG7tGzbdt6MeT3cO5du4OpPxa2fH27p7Ojr2 Cntn5WqnrmYIfrpbw6shAUigiwQ4YPkAsj/hIKwCbHMiHBrDbYVGPA7aoKVk6idKEdfFO7+K3XC7kZ9FjHwVZ 97BnLnisf+3c2wHHeApSAASeOEEMI0a02kQQotoCYYCCaE4uJq gdBjK4yBW+s1f2CABSAASgARecQJcpQzsYDMUjWAgmoKD4DgOQ vDK6tV1UoKHwM2KV/zrAacPCUACkABLANUowYYEzuFjXD6Dc0E2ckytIZwtUZAIgia7 6gIFWUICkAAkAAn8gwngPGMOl09hGAlK4OF8kDQQ+/1OU2WDzlmI2wnhquIf/Ojh1CABSAAS6CoBrrkV18iUKzBCBTwSBAthHLywnmhUkSJzrgb k94ANEoAEIAFI4JUngJkKMRNTjNAyhBqhQQFlDOcZYcAodb9Bj VBwW/uV/4JAAJAAJAAJgF0KkPoJx3UgVJskcA4HxbgYqdXx+SAQj6MmdBA RJAAJQAKQACTA4YACRnycD7a1eTQHpUDuJ0W9RqMkeXwTI36bf BTtAAP5o88cOFP4fMIwdLXXDqyZP+/rq7LnEBH4DAcnqpP3rpgza83Zyg5zt7eD8Dl9zGibFCD/I2yQACQACTwNAbb2Hd+IKzAGcRQIyAEF0sXy+ByNUsvl8niCrm VrI6sSjl62nvyu20OpGGhlWXa5aU8vy+YUr08qIqMpiNp93nzy 6m/e5huDAgx1eeU8dzfzZ7Pd3nrwp9qXoaTpEQdvu05ZuyfcTIA/1VBPiurBdYy2USJHjU15pLQs9+qFW8KJc0eaPu2g8HpIABJ4tQ mgOjWuE9CEBmxXcEFOaprCRW6WKhmhJbSclpyYHRMiay5FVofM mcFWgEAYnSTn+q3S+pL4I1H1fUYHIIV3Cmmf6csXj3Ts6grlob sxyqz9mxLF67Y4m7EJDJUZOz9Ymz/xhx3TxE+ngPS3aD14x3Ps5CxZFvnZpsw3d0530hfGe6GNKPxp+ bJTtXoZzAPfX79yhCObkpCW51747WbRnaTcbvM3LgwUomR13JY j/HkfDW1VWuOFig5vDglAAi8tAZ1cSqvljEZNaxUUAiLxSNzG3kR pxlM0EVp555YUWnr9eKbPzKEg5zijKY7ZsTe/z8JFw12bog+KR7+3YJpYm/bFnA0HT4YNWvoE6d+U2VHJdJ8vW2qhoSY9p36wHuvj+Az0BHge rQdv7wFR8uLsaksfz44WMrryxLgSuwlsoc8X3tT34u76rtg23p HDNbN1sDVrzq5NlJzaHGm1dNW8AdiyL9JrdIGm8qvH0nq+swrq iRf+xKAAkMDfggCjU6sUCpRmY+9ohCJp4PXEUAxNC0y4Qlujzu ZAVsRf4AwLApUZGHXu4c8PSYbPGeHMVxWk1Zj19gOveEanUOoQ u+4t1Uw7G671eW1FagHHq++fNaIFTsH9HFuSnT/eUG17txm8bRdQp+n+wWUrImqQDnUALctNrxP27GHxbMxihuTo6 megkuwFyaCpAz3d3N2c7Fr0BMjqnXsynjO4nzVG69S0UGSONaX/ktx99pi/IgV6V0WH/SABSOBlJmBhYWUENieAdR2oC5R1j8W1GgrVV8EDFVM7EV2T/1uq7eBxbB5Vovji5VpOgB0ovKYuulaEeYL6nKQkaf9P0jeWbxy jr0rHEFWpMWdiz16sCvx02yJf0BNUF8o48+vvd3IzMkvU5n7vb vhkLKhPx2hKk05FxsYl3JFifqtc9DsgjKbixukjp+tGrf1PHyN NZVr00ROFIWtWBqlSTx3+OUkwd+sHvvp+pPT26X2H4/OkWkIl5wR+uOWjEKtWL3pDg+svNCAJ1XA3Q+ow0vOhotgGuumK 08ow8QRBftTWfSdSZL0Wfr1yuB3H4ICgmve5Y7+cv5wuf23t9i V9wLgGmLDSPFmjpTeiU7KKUmf9au42cNqSuUMd9eX7SEl2EeYy UYDSTYXF5q+HGeWdvmj7zodti2g82V3hVZAAJPDPJwCUBI9nRF OMltKSDI0yFOZgYkSrEJ2GQUExvA6btiQpx6q/a/O7HIyBgJ+tFEJUpOYoOYrEXZv2JKATNu9cPoQtH80o7hz571F5 yMyp/rwmOQkwgow6AAAgAElEQVTUEqIpOL7+yzTPWcvWbtkc7kzxewc 48EDhtSt7NsXxwpZ8/nV4d644mH1Pk5Lrkb9En468pu0h5jZknD0ZGxudVCuojd2x7af kG6klpn6gfidoREXstu9y/JZs3r13279fcwpetHBgaz1haHD2wraS2FMliUd3fHu8mCZvH99 7ICqzARjj2nYDAhOVt+5rKUnS+SKHYaEeiCQzvVpnuKeu/Mw3kYLJqzd/v2XV9N6snmjLpEPenZ0kVfiARes2frZ0pPDeb9u/iq5oMSBiOKKWKkhtaeJN8VsD1efiBJPDTLJiIvZ89fWhq7WdWx k7uy88DwlAAv90Aur6Mq1GSjNaDkJzGRIDlqivVvad/7a7r5sFwhZy66Dpqm5kcX1dmq1UfPe3P9+7/+O+JmTd7Vsyp9HBiuspZda93MybdxZ0JZG7kkVjA2QXzzaGrl7 kDwr/1CbuiaLfnORjgjJNd5PKhH39bXFGnrbvYMWgKcGgMnLNvXozb7 1ZB7cJnjSpLx8VBXlbcK36jJsSYsPhChCjgPc/XhAi5IoH9jQHmoeWXPr+mDRkhIcxSkuuHUtkAh4pzmx4cFBC3I AkXCOXIW8Ns0LMQhYtX7oofIK/Fai3bUBghJLmZjVYhC1bMX9ccC8PkTFq5WqNGexJVif/lpaXW9yEmLt6WAMsbZl0ALsrp3jiwWOH+nl4DZyx9G1HpCK7Bo RUgoY7DJs9hDgXEZPnMXem7dUo9cj+xXsO1vWdMmsUlhBxpQ7q iq7AhX0ggVeagAa4OhEaSqdGSa2AQU0wDHO0waeFdlsZ7jJ7vH 1HbBhVaY7CxtGseemBChzcREYoQjdmXa0VDRk6aKgnnXMuXQrK YIBGlCYk1nBkFyJu2s9ZPcsPOApRtdd+K+o2LID92a8qSini+f QT8xhpWvQd+9C+lihCVGXcIxz9HmxUaMuuF/F9/Nn1CUI25ORofWaFh3kJqfK0EmGgL1uYjZKkxOTZvMFuLjPynKR yy0Cv1kXm2hvcoCSszFU3c7Uu/Z1bXIANd2NUxTcrBN6vsWYyWl6SrxC9HmRdb2Bq7Bt78Fv9NDE bFq37tUADTHttmXRE+7HOcUysTXkOPmDNo28cq/5zVq0InxzaS5vwS+2QUOZsNDlskAjn8Pg8IyH/xe+xPNbkYGdIABL46wmANzmKgRrAGMpBuSgDHGYwRIfwScIC0Q zw7NDBVVeXJ8GtTFq/aOiGjGtM6L9GO1l5D3Shc8+nN7C6gm7Kvy0ReA6ds/CdYHseLS8tapTdv17OdwZVuBlVYULULZXIX8zVlKacLzD1cWE/LL6WUm/j5wJiKRolKpqovJGpdgly4oM9d1J+70atfbAncLoi67JyKI8AV p+AKMDrZTwXJzOyNvX4jn23ca/eLZ5TD6Aq85MKDQyubDIkSX29ov7uLal9oAdrAFPKNLTCYLeqg rRCxnWAmzGrq1IvVLpPHOGoNdSzpqSKdBi5eudn48xyknJBpeq 2TGSPJvJlSIJ8sF3U3rGhrwwtzUoh3ng/tFtrXzGy6sKxgtem+MhSsnBvsQChFRU1lkE9X3TVVkMzgJ9BAp DAy0UAeD2Bn/WgVAWKoQz4D0LjGrVGJWuUK5W2Zvxaafvi0qoGFdLK25IhqlNi SoevCvc3QxGBX7CIOQ50xfBQG1oN3rXq+gaFqr42IzGlWjxyjK iiQSdriD22K1pCksVqxijzxP4qYVOVUmWbGbPvRF5ldgVlrL19 5gRhHRQ6QJeZWkub58Ueuku7D3ZOLhEGzLcE++4NeWnVAm8i+/zJSjNPrZSQF/z8w//6BPvZcHjdA4FeebiRjSWVSpVpm8GHuRiQpM5n9Gjz7CqdLC/59AGZ2mnopMECQ916+CN3G4W+lohaknVu/6Vu8z4aYYdUJzw6tWNrdsRney47tHWYsaW124i3BwBsbZmMd3p IXkZbGrt1Q5TlB9sW+hkTho+BsvyjMarK+1Khu6MJWX8n9tfco I8X9mutA8ja3w9n+s9ZYYXmgkq4Igwha25ctxi7WO9xABskAAl AAh0RYBjgJssB7rEMwu4062tXWNkaWdij0jpU3dDRpTSI2FM1q fSrBlnGka0HrpZUVEpN3XzrT5vOHh9g2+31waKIiHOpkuGj7ey DR/SOPLR33cbMIeNnTnsL7ELLXN3tLKsw++GLZ3BOrS0fMShs2hhf Xqb0Um5NlXDywgVV24u03Vx7hw1yAZ5SRKECd/Tubt97wggfo9xv9vF8J3YDKwkQvF1DyLEsdeiMsd10aZft3X3f nBnen4n7qNH+jUc2KhCOuXtPB27bwRlZbVtJ/MBeSalbd6sSmnQa815/Bx7KGBQYT/niuKXm3IZlBYPGTZ+7YaIdcDlihG2mNj2oP7lh16Gtuyt9uwcu XuhjDtQz1oZJqyBvlMOl5Iy1G86u+9o5/vPpgMDzMzs2RGTTIm+fvqGzZs/vzvqXPdTAvvv/rnjMXOMAHJhdhvbT/fDDYbWPaOriQa33/Tt63vAcJAAJvLoEtAgtoFEOWFIACwoDckAxqEYdSZG6xpLriur CO/VTRo56sy0eM6E5QuT/uGDlvZn7No0EYRWGGiWJX7fg28oxW36Y58Ua/BlCqeOa6P03n7wxhLRabuxgzYZXMOqKvHqLHmITWpoR+etd0di pA4EtSn51ffhu+40/LvHuWl6SJ5cFYUilijE2aY5ze4pxDF/KkCSDt0TMt3ds+EoDnzI0SaM46xUNGyQACUACjxDITE9t/sTNo0dbOBfiYsxOfcdlOAIQgMegJMPogD0K5zmTWq1GIeGaW7S 95s9PeI7BfsblN4ra9ZPi2Az594JAdfT2o5nANk+URa9f+lWC5 FGDfEe3MHAO5VmK9HoCNNTI0UtsQlb9vvvH+34zZ4SIWDWkLrh SaBo81PW56wlWANzkeekJ/eh/ZlZp79gAIcMfoRjUE4bJwE8hAUigcwI4iIVgGBLU18aAJQpFGQ zXqcppeb6ZKR/D2exB7TfjXot27ecK2QC8dhouGvXp944Xz1//ac8luYzu/d6cAayz0rNsRMGx9dvTAjbOZLfgGU1lyolocuIn7z9BGpFnKRU cCxKABCCBfxIB8Ntbv0uBspsVCAMMFDjdeIXPY/i8bqRG1fFUOUbmHeiJ5mtxa9+w6b4dj/M0Z1G+jaszGb92drytu6ejY6+wd1audhJAO8vTMIXXQgKQACTQ mgAXQ2mQiJwBRbUREKsNfDNx4DSLoTwU/HH/CiPOUz4Rrnjs/+0c+5SDwMshAUgAEoAEOiBAApsTwwCjEBdELICsTyDKAjcfynC saErBIPIOroSnIAFIABKABF4RAjqGplCGwBCN3hAFlhE4xvcA0 VkUBfSE+StCAU4TEoAEIAFIoAMCwG0fBVlQGRCijbDZloABiqE qMZ47ZuGFoFYIktjBxfAUJAAJQAKQwKtAgMuhMQQDu9lgx0JHM UqUxkE5Zkzgj+Kwyuar8AWAc4QEIAFIoHMCICgCpH4CEcFqlAE VjpSgYCrKe4NBBDS7dQFzjnZOEPaABCABSOAfT6AJYzisnyymR RkN6wzF4DRYWFBakMAVOEXBBglAApAAJAAJ0Gx5VJTPLiNokDe OJnFcp9M0cwEqBDZIABKABCABSIDHgKUEouVQYFsbpTlGYOOC0 DeVSlFXU9hFQCAD+JkDZwpbVEwXL+pqN13ttQNr5s/7+qrsOaiuZzg4UZ28d8WcWWvOVkK7XVefLewHCUACfw8CJAi7A 6kCaYRPcfgUiKsg8QsXz8jry4jGHFljcY++yzqfB1mVcPSy9eR 33VqKALFX0Mqy7HLTnl6WT5fiGmRMjdp93nzy6m/e5huD2hV1eeU8dzfzZ5McpPXgT2Vto6TpEQdvu05ZuyfcTIA/1VCd84Y9IAFIABL4iwmAH+oamuEzqDmwQTGYGtSt2LV1Rcm9WC tri6CQmV2Qhqy5FFkdAkowsEqB0Umyky/ERe1bOWfx1hM/7/ri4wXhH31zoaK5cmcXRmvVhVFm7d+UKB4/yNnMxISHKDN3frB887mqZ/OjvfXgT/V2J8siP9uU2Wvsa07Cl0JP0Mri5IgDceVgpfhHIyW3og5899nC tya999npYrAV1dxoee6548d2r1247IebTeyqjayO+2rz70+b1L HVY4T/AxKABP72BGia4TIoD+FQGEpwQAIoFFu/YdP09z/vM3CG2N270/nR0uvHM31GeYMqO4ym+OyW9UcqnAcN72eposSj31uw9NMNC9xK Lx48WfBEtilldlQy3advSzU71KTn1A/WfzrB8ekWKg+m1Hrw9iZKyYuz8tuUp2vdW1eeGFdiN4gt1fqCG 6Orz4k/9v2mtSu+PlFuZfFHtkeiInbTlnTXqf9Z998lPrKbR4/eVuolJUpObY7kDp8+b/5g8lZ6jQ6h668eS+v5zuutKla94EnB20MCkMBLQAAFEXgIn6Yw SteE0HUgzMLS1qVJWVlTGVdbE9uZgGRF/AXOsCBQsIJR5x7+/JBk+JwRznxVQVqNWW8/8IpndAqlDrHrbvEkr3dtRWoBx6vvg/LaCCJwCu7n2JKAvDPJOjvfZnBDFxD3Dy5bEVEDish20GhZbnqd sGcPi2djFuvgTp2eYrQqjte496f2NsNd+rNFXPWNLIvedEQ9ar KvGYqZ+wSLEW11RRO7NFPnnoznDO5njdE6NS0UmWNN6b8kd589 xqnjjMKdigE7QAKQwD+NAPCO1WGMEmNUDKMGZasRHd5QlyJvzK N0WlMLz06mq8n/LdV28Dg2vSxRfPFyLSfADlRfUxddK8I8JzvxSEnS/p+kbyzfOEZflZMhqlJjzsSevVgV+Om2Rb5snTayPuPMr7/fyc3ILFGb+7274ZOxYh5YnpQmnYqMjUu4I8X8Vrnod0AYTcWN0 0dO141a+58+RprKtOijJwpD1qwMUqWeOvxzkmDu1g989f1I6e3 T+w7H50m1hErOCfxwy0chreu8GRpcf6EBSaiGuxlSh5GeoLz2g 2agm644rQwTTxDkR23ddyJF1mvh1yuH24EIeAMDMsr8c8d+OX8 5Xf7a2u1L+oBxDTD5416Pf4CZOvUwpWpjMqW2QT3ZUntsU909c arEca6fBTsLjAcseQjJYythkJLsIsxlogClmwqLzV8PM8o7fdH 2nQ9dOqyo/vhCwSsgAUjg709AA3JAIQgIqgCJyEGuQBrDMFl9LoeDcXk2Clm 7VYuaJ64tScqx6u/a/C6nKQbUvtBRCFGRmqPkKBJ3bdqTgE7YvHP5EDugKBjFnSP/PSoPmTnVn9ckJ9nqrJqC4+u/TPOctWztls3hzhS/d4ADD6EkV/ZsiuOFLfn86/DuXHEw+54mJdcjf4k+HXlN20PMbcg4ezI2NjqpVlAbu2PbT8k3 UktM/UAFVtCAnWXbdzl+Szbv3rvt3685BS9aOLC1njA0OHthW0nsqZL Eozu+PV5Mk7eP7z0QldkAfoa37QYEJipv3deCeqTnixyGhXogk sz0ap3hnrryM99ECiav3vz9llXTe7N6oi2Tp/9GMarC1Eqjnn0cWpYGqrzzN+R2fTxAuXPQKDUoiG7UzdGU1SMY jqilClJbmnhT/NZA9bk4weQwk6yYiD1ffX3oau2z2RJ6+gnBESABSODFEwBFtRm 2VgVGcXCaywWVjTAdAdSHkaS+Lj39SocC6qpuZHF9XYz0nfjub 3++d//HfU3Iutu3ZE6jgxXXU8qse7mZN5uedCWRu5JFYwNkF882hq5e5 G+CULWJe6LoNyf5mKBM092kMmFff1uckaftO1gxaEowqG1dc6/ezFtv1sFtgidN6stHRUHeFlyrPuOmhNhwuALEKOD9jxeECLnig T3NwXuQllz6/pg0ZISHMUpLrh1LZAIeKa9teHDw+jQgCdfIZchbw6wQs5BFy5c uCp/gb4Ub6oYjlDQ3q8EibNmK+eOCe3mIjFErV2vMwIA42DBO/i0tL7e4CTF39bAGWNoy6RB3F08S5TcLaLf+zi0J5InqW7lK3LG HTbPm0FbnSTjd+zmx6h13GDZ7CHEuIibPY+5M26tR6pH9i/ccrOs7ZdYoLCHiSh3UFV1EDrtBAq8AARRkCOQzOB/lcTEODvKRY3V1Dfl5+Wmpd0vLqjsCwKhKcxQ2jmbNdg5U4OAmM gK5pBqzrtaKhgwdNNSTzjmXLgU5Q0AjShMSaziyCxE37eesnuU nxMD7+dpvRd2GBbA/+1VFKUU8n35iHiNNi75jH9rXEkWIqox7hKPfg40Kbdn1Ir6PP7 s+QciGnBytz6zwMC8hVZ5WIgz0ZUvrUZKUmDybN9jNZUaek1Ru GeglbOXX1N7gBiVhZa66mat16e/c4gJsuBujKr5ZIfB+jTWT0fKSfIXo9SDregNTY9/Mg9/qp4nZsGjdrwUa4GzUlklHtLt6jpTcvasQBbqz5j226SRF9Yi5q GWLm6i4mU95Dms2RiEcq/5zVq0InxzaS5vwS+2QUOZsNDlskAjn8Pg8IyH/xe+9dHXSsB8kAAk8ZwI0zgXrCQbj0GBhAUqngn9dvHwv7kL2/YJ6kJ68o7vr6vIkuJVJ6xcK3ZBxjQn912gnK++BLnTu+fQGVlf QTfm3JQLPoXMWvhNsz6PlpUWNsvvXy/nOYrCkUBUmRN1SifzFXE1pyvkCUx8X9sPiayn1Nn4uIJaiUaKi icobmWqXICc+CC4n5fdu1NoHewKnK7IuK4fyCGD1CYgCvF7Gc3 EyI2tTj+/Ydxv36t3iOfVgDsr8pEIDgyubDElSX6+ov3tLah/owRrAlDINrTDYraogrZBxHcBuIVOS1AuV7hNHOGoN9awpqSIdR q7e+dk4s5ykXFBrvC2TNo5WDEmAnL/Nrb3jR54Qo7h/s9rU29vqgR8BTepA8uCWbQtNwcUUasC0EJtW+/Rk1YVjBa9N8ZGlZOHeYgHIQF9RYxnUs7Wi7eibAM9BApDAP50A qHyHsvHa4NUFXreguDaC5+Y38LiopQVPDpyXOmi0Cpi9W3lVMk R1Skzp8FXh/sAwLvALFjHHga4YHmpDq8G7Vl3foFDV12YkplSLR44RVTToZA2 xx3ZFS0iyWM0YZZ7YXyVsqlKqbDNj9p3Iq8yuoIy1t8+cIKyDQ gfoMlNrafO82EN3affBzsklwoD5lhyEachLqxZ4E9nnT1aaeWq lhLzg5x/+1yfYz4bD6x4I9MrDjWwsqVSqTNsMPszFgCR1PqNHm2dX6WR5y acPyNROQycNFhjq1sMfudso9LVE1JKsc/svdZv30Qg7pDrh0akdW7MjPttz2aGtw4wtrd1GgCAUDtKWyXin h+RltKWxWzdEWX6wbaGfMWH4GCjLRxtYoBXS4tF/eo3x7T2tkTvpGTU6T4eauMNZvRd/Gdi8bdFyKVn7++FM/zkrrNBcBcGIMISsuXHdYuxivScCbJAAJAAJ6AkwYCnR/MYBqchRoC9wM2Mc4yAURcuVf0RqGYJF67RaVZNKv2qQZRzZeuB qSUWl1NTNt/606ezxAbbdXh8siog4lyoZPtrOPnhE78hDe9dtzBwyfua0t8Au tMzV3c6yCrMfvngG59Ta8hGDwqaN8eVlSi/l1lQJJy9cULW9SNvNtXfYIBdgSiEKFbijd3f73hNG+BjlfrOP5 zuxG1hJgODtGkKOZalDZ4ztpku7bO/u++bM8P5M3EeN9m88slGBcMzdezpw2w7OyGrbSuIH9kpK3bpbl dCk05j3+jvwUMagwHjKF8ctNec2LCsYNG763A0T7Xhgs1rYZmr Tg/qTG3Yd2rq70rd74OKFPqxzEtaGSasXP8rhUnLG2g0UFAEVRQwf P/RUGE35lTPnMgvzU5NVFPeXHTty3f3HTH5DzOd1nzB/1K3vItZ9Vuzv5PH2F0v7tAr/APvx/7viMXONA3BsdhnaT/fDD4fVPqKpiwe19gcw9PzhZ5AAJPDqEAAWJ5AFCmxgMiBtIDgC C4v+fc2BUxTITsHloB8u/37kqDfb4jATmiNE/o8LVt6buW/TSBBWYahRkvh1C76tHLPlh3lerMGfIZQ6rgl4mT5NYwhptdzYw ZoNr2DUFXn1Fj3EJrQ0I/LXu6KxUweCn9Pyq+vDd9tv/HGJ93MvDc6QShVjbMJ9uim1h4MhSQZnvVpZdu0ct3dtlz9naJJ GcVCwBDZIABJ4dQlkpqc2T97No0dbChfiYpqivgU7EmBlgbG5y MF2BYppCAq8PnQ6mgCerx00nmOwn3H5jaJ2PWo5NkP+vSBQHb3 9aCawzRNl0euXfpXwtCkjUJ6lSK8nQEONHL3EJmTV77t/vO83c0aIiFVD6oIrhabBQ12fu55gBcBNnpee0I/eoic6OO7g+XTtFKilDvVE11DBXpDAK0yABuWLKBLUqOAwbMFUD KFxDkZzMFZ9gCVGh2SMey3atZ8rZAPw2mm4aNSn3ztePH/9pz2X5DK693tzBrDOSs+yEQXH1m9PC9g4kw0cYzSVKSeiyYmfv O/d4rf0LG8Fx4IEIAFI4BUlACzhGMoA/1Jgf6JoChihcBwHViiGAyxQQH102DhG5h3oieZLcWvfsOm+HQ7 zVCdRvo2rMxm/dna8rbuno2OvsHdWrnYSdKzjnuqG8GJIABKABF45AigFFg8gZy CrJEjwb7BJITADP88RDkhN/ncog8cVj/2/nWNfuecGJwwJQAKQwF9IgKZJEEwBKqaytVKBBxRYVViAADjQQB GLThYVf6GY8FaQACQACUACL44AzWoHsCfBLiFQsJLQITihM2bX E8A16sWJBe8MCUACkAAk8PIQABHZIJSXVRfAAoWAhQWC19Y5cc CGBWicZ7wF/fJMG0oCCUACkAAk0HUCYEeCohAK2J5A8DO4DKgK1vWJ0rFx2zh UFV0nCXtCApAAJPCPJUAiIKsH8OBHwOoCqAoa5IDiYBjrDEUCf 6jmXH//2MnDiUECkAAkAAl0hQBQEMBVttnXiaYxCmSWA5YnsLgAYXgE8W QlsbtyX9gHEoAEIAFI4G9DAATcYcDWxP6BoF2wmY2CVOQ4BmK3 2VwfcGP7b/MgoaCQACQACTw/AhjG46A4F6wlwPY2xm5YYHfu3r1fUFBUVJR/P7+LNwYZwM8cOFOo6WL3x+umq712YM38eV9flT0HzfUMByeqk/eumDNrzdnKv0FNIEbbpNA9B56P92hhb0gAEvibEECBigBpATkY h8OGVdAYG639mOmnyaqEo5etJ7/r9lAyDVpZll1u2tPL8jHHegQboymI2n3efPLqb97mG4PaFXV55 Tx3N/Nns93eevCnCvCmpOkRB2+7Tlm7J9xMAOLcX8LGaBslctTYlEdK y3KvXrglnDh3pOlLKCcUCRKABF5CAmBDm61nBApTANMTcHriYD iX21KVuWvikjWXIqtD5sywYZUCo5PkXL9VWl8SfySqvs/oAKTwTiHtM3354pGObI6mx2yMMmv/pkTxui3OZmBwRpmx84O1+RN/2DFN/HQKSC9G68EfU7LW3cmyyM82Zb65c7oTKO/3EjRaWXz1TILi9dmjxH88SqLwp+XLTtXqpTMPfH/9yhGO7Dlannvht5tFd5Jyu83fuDBQiJLVcVuO8Od9NLRVJZKXY FJQBEgAEniBBEBGDzYIj9LH4AEDFLBECfiPkWqPll4/nukzcyiossNoimN27M3vs3DRcNem6IPi0e8tmCbWpn0xZ8PBk2 GDlj5BAj9ldlQy3efLlmp2qEnPqR+sx/o4PgM9AYi3Hry9R0DJi7OrLX08O1rI6MoT40rsJrClWl9wY3T1 uZfiEtJuJl4p8V079iGVr74Xd9d3xbbxjhyuma2DrVlz3nSi5N TmSKulq+YNwJZ9kV6jCzSVXz2W1vOdVVBPvOAnCW8PCbxkBEDe J7CsAFF4bCSePpMsxhfw//jrTFqyIv4CZ1gQKFjBqHMPf35IMnzOCGe+qiCtxqy3H3jFMzoF KKVn193iSV7v2orUAo5X3z8rugmcgvs5tiQg70yyzs63GdzQBc T9g8tWRNSAhLsdNFqWm14n7NnD4tmYxTq4U6enGK2K4zXu/am9zXCX/mwR15bGyG9fkAyaOtDTzd3Nya5FT4B87bkn4zmD+1ljtE5NC0X mWFP6L8ndZ49xeqxlZadSwQ6QACTwtycAKlLoQJkKNoksxmCgC h6K8/ldLvWgyf8t1XbwODa9LFF88XItJ8AO1KxTF10rwjwnO/FISdL+n6RvLN84Rl99kyGqUmPOxJ69WBX46bZFvqAnQtZnnPn1 9zu5GZklanO/dzd8MlbMA8uT0qRTkbFxCXekmN8qF/0ah9FU3Dh95HTdqLX/6WOkqUyLPnqiMGTNyiBV6qnDPycJ5m79wFffj5TePr3vcHyeVE uo5JzAD7d8FNK6npuhwfUXGpCEaribIXUY6QnKaz9oBrrpitPK MPEEQX7U1n0nUmS9Fn69crgdx+CAjDL/3LFfzl9Ol7+2dvuSPmBcA0z+uNfjH2CmTj1MqdqYTKltUE+21F 5zo6U3olOyilJn/WruNnDakrlDHfX1pUhJdhHmMlGA0k2Fxeavhxnlnb5o+86HLk9 gKnx8UeEVkAAk8HcigJJsxVQaJJIFry0MeMcyGBfsVjz463gq2 pKkHKv+rs3vcpoC2oYG1ZCIitQcJUeRuGvTngR0wuady4fYsTs NijtH/ntUHjJzqj+vSU6yCxhNwfH1X6Z5zlq2dsvmcGeK3zvAgYdQkit 7NsXxwpZ8/nV4d644mH1Pk5Lrkb9En468pu0h5jZknD0ZGxudVCuojd2x7af kG6klpn6gAitoREXstu9y/JZs3r13279fcwpetHBgaz1haHD2wraS2FMliUd3fHu8mCZvH99 7ICqzAbg1te0GBCYqb93Xgrqj54schoV6IJLM9Gqd4Z668jPfR Aomr978/ZZV03uzeqItk46Bd+UsoypMrbq+9XkAACAASURBVDTq2cfhz6U BqcIHLFq38bOlI4X3ftv+VXRFi4sWhiNqqYLUlibeFL81UH0uT jA5zCQrJmLPV18fulr7N/Dj6goP2AcSgASeAQEta38CqoIiwa6FjgZ/wBfqz9bhHXRVN7K4vi5G+k5897c/37v/474mZN3tWzKn0cGK6yll1r3czJtNT7qSyF3JorEBsotnG0NXL/I3QajaxD1R9JuTfExQpuluUpmwr78tzsjT9h2sGDQlGNS2rrlX b+atN+vgNsGTJvXlo6IgbwuuVZ9xU0JsOFwBYhTw/scLQoRc8cCe5kDz0JJL3x+ThozwMEZpybVjiUzAI+W1DQ+OGJS Ea+Qy5K1hVohZyKLlSxeFT/C3wg0KjFDS3KwGi7BlK+aPC+7lITJGrVytMUNTQ8jq5N/S8nKLmxBzVw9rgKUtkw5xd/EkUX6zgHbr7/zQ0pAnHjx2qJ+H18AZS992RCqya5pDK3GHYbOHEOciYvI85s60 vRqlHtm/eM/Bur5TZo3CEiKu1EFd0UXksBsk8M8nwOYgZ6PtKJCNHFRJpSgKR OT92ToCwKhKcxQ2jmbNdg5U4OAmMgIxfI1ZV2tFQ4YOGupJ55x LlzbnBiFKExJrOLILETft56ye5Qcchajaa78VdRsWwP7sVxWlF PF8+ol5jDQt+o59aF+QBp2oyrhHOPo92KjQll0v4vv4s+sThGz IydH6zAoP8xJS5WklwkBftrQeJUmJybN5g91cZuQ5SeWWgV7CV l6r7Q1uUBJwG6LqZq7Wpb9zyya/4W6MqvhmhcD7NdZMRstL8hWi14Os6w1MDSg8h8Fv9dPEbFi07t cCDWDelklHtLt6jpTcvasQBbqz5r02jWNibcpz8AFrIX3jWPWf s2pF+OTQXtqEX2qHhDJno8lhg0Q4h8fnGQn5L37vpc0E4AeQAC TwYgiAVxawGYG1BUgsC/5AkYoOt3AfFlJXlyfBrUxav1DohoxrTOi/RjtZeQ90oXPPpzewuoJuyr8tEXgOnbPwnWB7Hi0vLWqU3b9ezn cWgyWFqjAh6pZK5C/makpTzheY+riwHxZfS6m38XMBsRSNEhVNVN7IVLsEOfGBUiPl9 27U2gd7Aqcrsi4rh/IIYPUJiAK8XsZzcTIja1OP79h3G/fq3eI59UBmZX5SoYHBlU2GJKmvV9TfvSW1D/RgDWBKmYZWGOxWVZBWyLgOYLeQKUnqhUr3iSMctYZ61pRUkQ4j V+/8bJxZTlIuqDXelons0VLmDEmQD8Lk2jt+5GvDKO7frDb19rYy5 EdAS7NSiDfeD+3W+iRZdeFYwWtTfGQpWbi3WIDQiooay6CerRX ti/l6wrtCApDAy0EAuMqCkhUUyCar39oGpigc/LNFto6jeWlVgwpp5VXJENUpMaXDV4X7m6GIwC9YxBwHumJ4qA2 tBu9adX2DQlVfm5GYUi0eOUZU0aCTNcQe2xUtIcliNWOUeWJ/lbCpSqmyzYzZdyKvMruCMtbePnOCsA4KHaDLTK2lzfNiD92l3Q c7J5cIA+Zbgsjyhry0aoE3kX3+ZKWZp1ZKyAt+/uF/fYL9bDi87oFArzzcyMaSSqXKtM3gw1wMSFLnM3q0eXaVTpaXfP qATO00dNJggaFuPfyRu41CX0tELck6t/9St3kfjbBDqhMendqxNTvisz2XHdo6zNjS2m3E2wMAtrZMxjs9 JC+jLY3duiHK8oNtC/2MCcPHQFk+2sACrZAWj/7Ta4xRVd6XCt0dTcj6O7G/5gZ9vLBfax1A1v5+ONN/zgorNFdBMCIMIWtuXLcYu1jviQAbJAAJQAIsAaAhwHIC7LCCwh XN9fDwrqZ+onVarapJpV81yDKObD1wtaSiUmrq5lt/2nT2+ADbbq8PFkVEnEuVDB9tZx88onfkob3rNmYOGT9z2ltgF1 rm6m5nWYXZD188g3NqbfmIQWHTxvjyMqWXcmuqhJMXLqjaXqTt 5to7bJALMKUQhQrc0bu7fe8JI3yMcr/Zx/Od2A2sJEDwdg0hx7LUoTPGdtOlXbZ3931zZnh/Ju6jRvs3HtmoQDjm7j0duG0HZ2S1bSXxA3slpW7drUpo0mnMe/0deChjUGA85YvjlppzG5YVDBo3fe6GiXbAtYgRtpna9KD+5IZd h7burvTtHrh4oQ/rnIS1YdLqxY9yuJScsXbDAd/2jh/6CjOa8itnzmUW5qcmqyjuLzt25Lr7j5n8hiNTcGbHhohsWuTt0 zd01uz53R8xTIH9+P9d8Zi5xgE4NrsM7af74YfDah/R1MWDWvsDwP+zQAKQwKtNgAFb2QgKNjBBwnFWZaA6NHBgvz+Yf PLBhyNHvdkWkZnQHCHyf1yw8t7MfZtGgrAKQ42SxK9b8G3lmC0/zPNiDf4ModRxTfR+mk/eGEJaLTd2sGbDKxh1RV69RQ+xCS3NiPz1rmjs1IHg57T86vrw3 fYbf1zi3WWn3ycVhyGVKsbYpDme7UkHafc6hiRBmpVmtu0dt3t xV08wNEmjOEgVCRskAAm8ugQy01ObJ+/m0aMthQtxMbKY74CPK/CVxQUgpQeDYRTwmP2ztb3mz094jsF+xuU3itTtdeLYDPn3gkB1 9PajmcA2T5RFr1/6VYLkUYN8e1e38znKsxTp9QRoqJGjl9iErPp994/3/WbOCBGxakhdcKXQNHio63PXE6wAuMnz0hP60Vv0RAfH7VDq+sc gPB/qia7jgj0hgVeUwGtDfIKHePUPce8T6NwrwNmjt0uXDVCIca9Fu/ZzhWwAXjsNF4369HvHi+ev/7TnklxG935vzgDWWelZNqLg2PrtaQEbZ7JePYymMuVENDnxk/efII3Is5QKjgUJQAKQwD+JgKVXd1KjUZdX12dVVhY3ykDGQEqp Q3EM5WJs/sAOG8fIvAM90Xwpbu0bNt23w2Ge6iTKt3F1JuPXzo63dfd0dOw V9s7K1U6CTgR/qjvCiyEBSAASeNUI3P/f78DiJKtuul/QoEQYsZsp/sV/NyQlJSVfTlaqn08BimfKmCse+387xz7TIeFgkAAkAAlAAq0I3I +72ahiKpSkk49ZyEhHoSkfv3T5d3dPz7Hjx93JyIK0IAFIABKA BCABhVqXq6DEfkb9p5hbCGnVNQkuV6pvZ2bmZN/t1fs5Go4gekgAEoAEIIG/C4ESDSpyNgkbaWdhhUt/r71zSY3JFWqtjiZI5Pbt23+XaUA5IQFIABKABJ4fgUYzbh9Pcx MdVntBfitFS6BGOKEmQJZYkBGKz4fpqJ8feTgyJAAJQAJ/GwIiZ1OdgJ96Q1FaprRHjcR2xmxiD+ATBRIHgvTkf5t5QEEhAU gAEoAEnhsBK0vBnXpdZYW2B4W7WPJQDo0TWo1Gq8FBlm/YIAFIABKABCABkFaWoOukWmMSsedxaZpWERReVFDANxJ0d/PA8WccLgeBQwKQACQACbyEBPz7BjVLJW+SGRSvrkmNKLU8kpFy EEIHEnugGMhFrpApcrLAnnZXDVAgA/iZA2cKn08Yhq722oE18+d9fVXWVXEMztTwh89wcKI6ee+KObPW nK2ENYEMw4afQgKQwN+VAKXQGmMMjdNlhLZcQ8pIGsd4ICMUxp BUWsqNqeO6EN1GViUcvWw9+V23h0xWtLIsu9y0p5fl06WyZjQF UbvPm09e/c3bfGNQu6Iur5zn7mb+bFY7rQd/qgBvSpoecfC265S1e8LNBPhTDfV3/SZBuSEBSOAfTKCbuTFCUhqtrklJykktDVKZsknIQYpZLqdrL2S y5lJkdcicGTasUmB0kpzrt0rrS+KPRNX3GR2AFN4ppH2mL1880 vEJvKkYZdb+TYnidVuczcDgjDJj5wdr8yf+sGOa+OkUkP55th7 8qR4xWRb52abMN3dOdwLl/V5oA0mw0uIv3b5382JypXjiJ+veZUt46xspuRUTnXTrxqVMje+ cz/5vYvfmdIq0PPfCbzeL7iTldpu/cWGgECWr47Yc4c/7aGirSiQvdFLw5pAAJPDCCVjbiDQaFa7WMBxNk0rboCYxmqSAE xT4A2mgOpWPll4/nukzyhtU2WE0xWe3rD9S4TxoeD9LFSUe/d6CpZ9uWOBWevHgyYInsk0ps6OS6T59W6rZoSY9p36w/tMJjs9AT4B5tR68vYlS8uKs/Dbl6Vr31pUnxpXYDWJLtb7YRlbFbd1dETB17vIvP59kknPy2K2 mZpsdURG7aUu669T/rPvvEh/ZzaNHbyv1khIlpzZHcodPnzd/MHkrvUaH0PVXj6X1fOd1qCde7JOEd4cEXjYCfCsL3N4BtRdxHO y5DjaohQnIQ45QGrbkdhdqHJEV8Rc4w4JAwQpGnXv480OS4XNG OPNVBWk1Zr39wCue0SmUOsSuu8WTvN61FakFHK++f1Z0EzgF93 NsSUD+tCDbDG5oQOL+wWUrImpAlvYOGi3LTa8T9uxh0bVVWAcj PeUpsirpzB1czLLmO3g7cjnc5iTmZFn0piPqUZN9zVDM3CdYjG irK5rY/RR17sl4zuB+1hitU9NCkTnWlP5LcvfZY5y4TykJvBwSgAT+YQR ADWYjDkLyuISpKWlpQzjY45u+Xv3dzsOFxaUYt7OXnyb/t1TbwePY9LJE8cXLtZwAO1BlTV10rQjznOzEIyVJ+3+SvrF84x h99U2GqEqNORN79mJV4KfbFvmy9djI+owzv/5+Jzcjs0Rt7vfuhk/GinlgeVKadCoyNi7hjhTzW+Wi3wFhNBU3Th85XTdq7X/6GAErS/TRE4Uha1YGqVJPHf45STB36we++n6k9PbpfYfj86RaQiXnBH64 5aOQ1vXcDA2uv9CAJFTD3Qypw0jPB0Ycw910xWllmHiCID9q67 4TKbJeC79eOdwOMDUwIKPMP3fsl/OX0+Wvrd2+pA8Y1wATVponahyH0FXrA0y4bL30qqxK4aBFvqzk qrsnTpU4zvWzYE1RGM8EWAJJHqtESEl2EeYyUYDSTYXF5q+HGe Wdvmj7zocuT2AqfCJ54UWQACTwtyFQqdAgGGZCIcBTVovSqJEA u3bl0rz3J00YG0qpdR3PQ1uSlGPV37X5XQ4ivBHw85RCiIrUHC VHkbhr054EdMLmncuH2LE7DYo7R/57VB4yc6o/r0lOMuC9pSk4vv7LNM9Zy9Zu2RzuTPF7BzjwEEpyZc+mOF7Yks +/Du/OFQez72lScj3yl+jTkde0PcTchoyzJ2Njo5NqBbWxO7b9lHwjt cTUD1RgBQ3YWbZ9l+O3ZPPuvdv+/ZpT8KKFA1vrCUODsxe2lcSeKkk8uuPb48U0efv43gNRmQ3gZ3j bbkBgovLWfS2oO3q+yGFYqAciyUyv1hnuqSs/802kYPLqzd9vWTWd3UQwwKRj4B2fRbkWLj09bXiotujMsfLhq//F6iJElXf+htyujwcodw4apQYF0Y26OZqyCyUMR9RSBaktTbwpf mug+lycYHKYSVZMxJ6vvj50tRb6cXWMG56FBF4lAqicqGrQ1Ci 1OooSMBSPJrHGJmX8xXh3d+s1az7oEIWu6kYW19fFSN+J7/7253v3f9zXhKy7fUvmNDpYcT2lzLqXm3mz6UlXErkrWTQ2QHbx bGPo6kX+JghVm7gnin5zko8JyjTdTSoT9vW3xRl52r6DFYOmBI Pa1jX36s289WYd3CZ40qS+fFQU5G3BteozbkqIDQdECBoFvP/xghAhVzywpzl4D9KSS98fk4aM8DBGacm1Y4lMwCPltQ0PDl6fB iThGrkMeWuYFWIWsmj50kXhE/ytcEPdcISS5mY1WIQtWzF/XHAvD5ExauVqjRkYEEfI6uTf0vJyi5sQc1cPa4ClLZMOcXftJK Mtu3AgRjB7xbSeYP8IqM/qW7lK3LGHTbNRSVudJ+F07+fEqnfcYdjsIcS5iJg8j7kzba9Gq Uf2L95zsK7vlFmjsISIK3VQV3QNOewFCbwCBJyNuL44aq4lKxR EhRKsLGhMqVAbCYyvXkm7eTO5IwKMqjRHYeNo1mzIRwUObiIjF KEbs67WioYMHTTUk845ly6l9UMQpQmJNRzZhYib9nNWz/IDjkJU7bXfiroNC2B/9quKUop4Pv3EPEaaFn3HPrSvJYoQVRn3CEe/BxsV2rLrRXwff3Z9gpANOTlan1nhYV5CqjytRBjoy5bWoyQpMX k2b7Cby4w8J6ncMtBLqP8h/aC1N7hBSViZq27mal36O7e4ABvuxqiKb1YIvF9jzWS0vCRfIXo 9yLrewNTYN/Pgt/ppYjYsWvdrgQYYidoyeVjaJzoGngXnD53jjF/wpht4FPqmkxTVI+Yii2ZNQVTczKc8hzUboxCOVf85q1aETw7tp U34pXZIKHM2mhw2SIRzeHyekZDfmfnxiUSEF0ECkMDfkYAWx4x MuJ7GvL4YKtSQEpUOKy+tuXevtLSstqiovKMp6eryJLiVSesXC t2QcY0J/ddoJyvvgS507vn0BlZX0E35tyUCz6FzFr4TbM+j5aVFjbL718v 5zmKwpFAVJkTdUon8xVxNacr5AlMfF/bD4msp9TZ+LiCWolGioonKG5lqlyAnPsJQpPzejVr7YE/wo5msy8qhPAJYfQKiAK+X8VyczMja1OM79t3GvXq3eE49mIMyP 6nQwODKJkOS1Ncr6u/ektoHerAGMKVMQysMdqsqSCtkXAe4GbO6KvVCpfvEEY5aQz1rS qpIh5Grd342ziwnKRfUGm/LpI2jFUMS5IOww/aO/3xCgEDs0STzSe8N0zsmM025KQVKhiZ1NIK27MtrCi6mUAOmhdi 02qcnqy4cK3htio8sJQv3FgsQWlFRYxnUs7Wi7eibAM9BApDAP 52AgkZAGLeGjwmNuZ4muCeO4iiKKFRKC3NTlZroaPq0Cpi9W3l VMkR1Skzp8FXh/sAwLvALFjHHga4YHmpDq8G7Vl3foFDV12YkplSLR44RVTToZA2 xx3ZFS0iyWM0YZZ7YXyVsqlKqbDNj9p3Iq8yuoIy1t8+cIKyDQ gfoMlNrafO82EN3affBzsklwoD5lhyEachLqxZ4E9nnT1aaeWq lhLzg5x/+1yfYz4bD6x4I9MrDjWwsqVSqTNsMPszFgCR1PqNHm2dX6WR5y acPyNROQycNFhjq1sMfudso9LVE1JKsc/svdZv30Qg7pDrh0akdW7MjPttz2aGtw4wtrd1GvD0AYGvLZLzT Q/Iy2tLYrRuiLD/YttDPmDB8rLcwNTcQTvjrZ2tOqrwCiWNl3V2cbZn716r7zPVCu fae1sid9IwanadDTdzhrN6Lvwxs3rZouZKs/f1wpv+cFVZoroJgRBhC1ty4bjF2sd4TATZIABKABFgCBPjhCqw UCEeHoBSPY4EhuLmlmS2fl51T1ABKqHbQaJ1Wq2pS6VcNsowjW w9cLamolJq6+dafNp09PsC22+uDRRER51Ilw0fb2QeP6B15aO+ 6jZlDxs+c9hbYhZa5uttZVmH2wxfP4JxaWz5iUNi0Mb68TOml3 Joq4eSFC6q2F2m7ufYOG+QCPKWIQgXu6N3dvveEET5Gud/s4/lO7AZ+OYPg7RpCjmWpQ2eM7aZLu2zv7vvmzPD+TNxHjfZvPLJR gXDM3Xs6cNsOzshq20riB/ZKSt26W5XQpNOY9/o78FDGoMB4yhfHLTXnNiwrGDRu+twNE+14YLNa2GZq04P6kxt2 Hdq6u9K3e+DihT7m4Fc91oZJK3MZyuFScsbaDQd82zv+4+EA/icv6IbMnaWoKC7Mvxp57liN8agvvgXbQAjSfcL8Ube+i1j3WbG/k8fbXyzt0yr8A+zH/++Kx8w1DsCx2WVoP90PPxxW+4imLh7U2h+gg68BPAUJQAKvAAG SoHQ0I0N0KhQDGaAwjIMu+tfb5+NTCJLGuNi2L/87ctSbbTmYCc0RIv/HBSvvzdy3aSQIqzDUKEn8ugXfVo7Z8sM8L9bgzxBKHdcEvEyfp jGEtFpu7GDNhlcw6oq8eoseYhNamhH5613R2KkDgS1KfnV9+G7 7jT8u8W69rHiau7ZzLUMqVYyxCffpptTO4AioGQKC55vZtnfcf C1DanSYgGf4MbQ3fMulNEmjOOf5zKDjW8OzkAAk8NIRMJgu8EJ cTPWPWxUUpUQoDofD4/BAjDYWG3eVICmQCQrBOnyB8ByD/YzLbxSp25ssx2bIvxcEqqO3H80EtnmiLHr90q8SJFR73bv2Ocq zFOn1BGiokaOX2ISs+n33j/f9Zs4IEbFqSF1wpdA0eKjrc9cTrAC4yfPSE/rRW/REB8fN0FD8yfQEOzAG9UQzRPhPSAASaJ9AHaGrp0gKQU1QjinC CBAG19HsegIojc4Syxr3WrRrP1fIBuC103DRqE+/d7x4/vpPey7JZXTv9+YMYJ2VnmUjCo6t354WsHGmfitXU5lyIpqc+Mn 73g+lLnyWt4NjQQKQACTwChIgEZqHYEBP8BCUoWk1w+CTJ7x57 vxFhVLVabQ2x8i8Az3RTBO39g2b7vv8yKJ8G1dnMn7t7Hhbd09 Hx15h76xc7STocDn0/ISBI0MCkAAk8I8kYMrlYKwvJUZQtJZhVEBV9O/v9cYbr+Xk5p8+HfPyz5krHvt/O7uQKf3lnwmUEBKABCCBl5UA2MsmaeBkA2xQCAHyyYIs5PHxiV pC6+XVY+nS+S+r2FAuSAASgAQggb+OAIjxUoMoLxQkggKmfhBs zWDGJiZgR7WsrDIh4dxfJwi8EyQACUACkMDLSoCkGZRhKJDRg6 Q1JKUGqwpjU2ONRodimMDY9GUVG8oFCUACkAAk8NcR0CLA5sT+ VwWSaTAMm5uvrLAS53FIkuQ8yAfx14kD7wQJQAKQACTw8hFQkC TwFgL1USmwtgB/wFnWxs4CaI+SonIQavHyCQwlggQgAUgAEvirCWiBhyyDgLxPIC EFSPpDAA+o+IvXZY0KkgKb3cz7c/5qgeD9IAFIABKABF42AmqK1jEIB2X4KAb0BYi8wxtkclBVm8OH 6eJetocF5YEEIAFI4MUQkNEMKHVnxKAchgZrCxok9kBBLiFQ8w F4znYWrv2HyCD/9ZkDZwo1z2UOutprB9bMn/f1VdmDhNzP8DbPcHCiOnnvijmz1pyt/BvUBGK0TQrwGwE2SAASgAS6QgAEVQATlI4mlTSlpEgtTeEM0BG P1ciqhKOXrSe/6/ZQMg1aWZZdbtrTy/Lp1iYgt3bU7vPmk1d/8zbfGNSuqMsr57m7mT+bPZTWgz9VgDclTY84eNt1yto94WYC/KmGeiz0j9GZ0TZK5KixKY+UluVevXBLOHHuSOjh9hgAYVdI4FU mwGOj7hhQQEfFriIQHENBNmoGAVsXXW1kzaXI6pA5M2xYpcDoJ DnXb5XWl8QfiarvMzoAKbxTSPtMX754pL7czmM2Rpm1f1OieN0 WZzMwOKPM2PnB2vyJP+yYJn46BaQXo/XgjylZ6+5kWeRnmzLf3DndCZT3e6ENJMFKi790+97Ni8mV4omf rHuXLeGtb0ThT8uXnarVH5sHvr9+5QhHtjAeLc+98NvNojtJud 3mb1wYKETJ6rgtR/jzPhraqhJJ8xjwn5AAJPDKEmBAXAXwgEJAgAUJsnpTCEZpSLCw AI3Sdm5JoaXXj2f6jPIGlRFAtc6zW9YfqXAeNLyfpYoSj35vwd JPNyxwK7148GTBE9mmlNlRyXSfvi3V7FCTnlM/WP/pBMdnoCfA4249eHvPn5IXZ+W3KU/XureuPDGuxG4QW6r1xTayKm7r7oqAqXOXf/n5JJOck8duNT1YIKrvxd31XbHt22++3bX/p/+tn+wJqoAA/VFyanMkd/j0efMHk7fSa3QIXX/1WFrPd16HeuLFPkl4d0jgZSOgJikC2J3YPwqUqCApNvs4ThMUp daZ8o06E5esiL/AGRYEClYw6tzDnx+SDJ8zwpmvKkirMevtB17xjE6h1CF23S2e5 PWurUgt4Hj1fVBeG0EETsH9HFsSkHcmWWfn2wxu6ALi/sFlKyJqwNZNB42W5abXCXv2sHg2ZrEO7tTJKbIq6cwdXMyy5jt 4O3I53AdJzBn57QuSQVMHerq5uznZmT2or6HOPRnPGdzPGqN1a looMsea0n9J7j57jFNzIe5ObgdPQwKQwKtDQE1RKpL909BAP4C YbQaft2D2jh37uBz8802bOwGhyf8t1XbwODa9LFF88XItJ8AO/FpVF10rwjwnO/FISdL+n6RvLN84Rl99kyGqUmPOxJ69WBX46bZFvuzvWrI+48yv v9/JzcgsUZv7vbvhk7FiHlielCadioyNS7gjxfxWueh3QBhNxY3TR 07XjVr7nz5GwMoSffREYcialUGq1FOHf04SzN36ga++Hym9fXr f4fg8qZZQyTmBH275KKR1PTdDg+svNCAJ1XA3Q+ow0vOBEcdwN 11xWhkmniDIj9q670SKrNfCr1cOt+MYHJBR5p879sv5y+ny19Z uX9IHjGuACSvNEzWOQ+iq9QEmXLCS0FVlVQoHLfJtlpyW3ohOy SpKnfWrudvAaUvmDnXU15ciJdlFmMtEAUo3FRabvx5mlHf6ou0 7H7o8ganwieSFF0ECkMDfhoCWotlaQ/qdbH0UHoIF9fN1dRGDtcbVpEsdz0NbkpRj1d+1+V1Os9fSOgoh KlJzlBxF4q5NexLQCZt3Lh9ix+40KO4c+e9RecjMqf68JjkJ0k 0hmoLj679M85y1bO2WzeHOFL93gAMPoSRX9myK44Ut+fzr8O5c cTD7niYl1yN/iT4deU3bQ8xtyDh7MjY2OqlWUBu7Y9tPyTdSS0z9QAVW0IiK2G 3f5fgt2bx777Z/v+YUvGjhwNZ6wtDg7IVtJbGnShKP7vj2eDFN3j6+90BUZgMwxr XtBgQmKm/d14K6o+eLHIaFeiCSzPRqneGeuvIz30QKJq/e/P2WVdPZTQQDTDoG3vFZlGvh0tPThodqi84cKx+++l+sLmIbqcI HHjhIuAAAIABJREFULFq38bOlI4X3ftv+VXRFi2ERwxG1VEFqS xNvit8aqD4XJ5gcZpIVE7Hnq68PXa3t3PrYsTjwLCQACfxzCAD/JwQ4QTE0UBngGHhDYTdTb3z22UqGpH+N+KXDieqqbmRxfV2arV R897c/37v/474mZN3tWzKn0cGK6yll1r3czJtNT7qSyF3JorEBsotnG0NXL/I3QajaxD1R9JuTQAFopuluUpmwr78tzsjT9h2sGDQlGNS2rrlX b+atN+vgNsGTJvXlo6IgbwuuVZ9xU0JsOFwBYhTw/scLQoRc8cCe5uCdSEsufX9MGjLCwxilJdeOJTIBj5TXNjw4UJE GJOEauQx5a5gVYhayaPnSReET/K1wQ91whJLmZjVYhC1bMX9ccC8PkTFq5WqNGRgQR8jq5N/S8nKLmxBzVw9rgKUtkw5xd+0koy27cCBGMHvFtJ5sZW1944kHj x3q5+E1cMbStx2RiuwaQv8x7jBs9hDiXERMnsfcmbZXo9Qj+xf vOVjXd8qsUVhCxJU6qCta+MF/QQKQAApSySI0BjaxaYai2H9gNTW1OTl3p0wZxwGlLDpojKo0R2 HjaNbcCRU4uImMQGraxqyrtaIhQwcN9aRzzqVLQcw3aERpQmIN R3Yh4qb9nNWz/ICjEFV77beibsMC2J/9qqKUIp5PPzGPkaZF37EP7WuJIkRVxj3C0e/BRoW27HoR38efXZ8gZENOjtZnVniYl5AqTysRBvqypfUoSUpMn s0b7OYyI89JKrcM9BI+eFfqZWhvcIOSsDJX3czVuvR3bnEBNty NURXfrBB4v8aayWh5Sb5C9HqQdb2BqbFv5v9n70zgmjq2x3+zE raEHULCDiLIIiKiUqoiglqte62itcXlob5nra1La9Vi2/evu63WrWjdaHFFUMQFKYoLm8iigCBL2MISCCEkIcnNvf+5AayY 4APRX22d+ejnc7mZOffMd+5nzp2ZM3NGzRjanhC5dOPZ0nYwit NkolazPwl4Flw7dpXy/pL3HEFTaCaKvqkB3codjIXUiWIybMHa1eHTQwbJk083jA7BL8W jQYFsKoWuQ9dl6vzVay+a+sM7kAAk8BcRUGBgWRsMKDAwHdKx7 46c//DJ3bQ0Ltf8+++/fJFWysZiAdVEv3uHgjXn3MND/jXBxsRtpB1WdC27mbAVWGtJnoDhMmZBxIf+lnRMXFneInqSXq1 jywVDCmlZctwDKdubS2uvTLtWauBuR9ysuJfWZOZlB/ZStAikmKI2I1dm52ejg+AqVPw4o8HS3wV8NKON+YUqZx/CnoBdgOlVdDsbQ7Qh89SeqDyqq0en51RXHSQlqWVahEtatWnS1 NTW9OiB0NLXmZgAk4jasTat2filWWW4wwhHPcJWZV6vdZoazJF ry1nP46NW49bt/WayYWFqEYg1rslEw9EKR8EZ8Z3q93T9ZwsBAoknU1nTPg5SOyb jrUVppZJuW2QwYX6a4t1PQqy7Oxmg/OvRpcNnuovS8qluXAaCtdXUG/sN7G5oX/QmwN8gAUjgn05AAY4fx7B2TIXiwFpgwHGWmpNTvHjJbB6vjMcr nzp5To8EMGmzFOnmVYkr6tISKseuDfc2JCEML382fgrYirEhZp gM9LWypuY2aVNDTkpaHXfcRHZNs1LUnBi9L16AohUyXDf3zGE+ s5UvkZrnJkSdKa4tqFHpyfMunlGY+oWMUOZmNmCs4sRjjzCnUb a3eUyfxcYUBG8uzqpjuCkKrp2rNXSRCxXi0t/3/zrY38uMQrf3BXbl2YS28GolUgMN4UF2WjRpdJ8wgVXAV4qKb18 4IpLZjJk2iqEt2wBv5FEL09MYkQnyrx6+ab1oVbAFUpf8fNWi1 +9JKnBZeWxHkJ6xqWPw7BEAmyaT922e0ReXVybuiIwz/nRnhJeeQvv10xkmYtm/9Ow3689JXX0V0VX2drbm+JN7dYMXugKjW/tEyHTi6KNNDxPPFvl9ETG0uw1AG/44nuu9YLUJqahNgbPJCFqfkW40aZnaEwEmSAASgAQIAgqwVgGW o0mIEgSuQAivUKr3YA9bW/t2uVTe3jGn3QMpTCmXS1ul6lGDKOfEjiN3eTW1QgNHz6YLBvPf 9zG3fmcUOybmaqZg7AQLS/9gj9hjhzZuzh39ftgHM8AqtMjBycKYT7Ycu2wu5fyG6uDA0A8m etJzhTeL6vnM6RFL+LvK5dYOHqGBdsBTSlHWRuW42Vt6TAl21y 3aHUX3nGoNvpzB5u16hZicLwuZO8lamXXL0snzvbDwYfiVVS2W 7z63UIFQWE4DrWiawnFRg6YmXmCtpNLR3oSHoTYTPx5mRSfhWh Wmpn13yrj9auTK0sDJcxZGTrUArkU4U6Nqc/yGoZH7ju04UOtp77sswp0FOJM1mHSbMiJRaCoxbupIBXx7un7a MID/uevK0QvntdVUlJXcjb0aXa83/rsf3fWR9icX90TGFGBsN/chIfPmL7ZX76f4M4H1+F/vOIettwKOzXZjhir37z8uc2fPWhbY3R+gh1cA3oYEIIG3hIA+R pwsC7oPMFMBFixAF0aaOD6IxaILWyV3UrOPHj40bvx7miwMmSx EUfLLkjWPw6K2jgPbKrQllSBp45Ifaydu37/IlZjwxxUSJU1f7af58glXCOvEelamxPYKXFZT3GQ0gKuPCXNiz z5iT5o1EsxFie9uCj9gufmX5W7dhxUv/8weS+KoRIrr6XftU+gx38v9gKPAdblza0RP1x2ScbRdSWaAs7v 6nnAMxUhUSv8ape+PhSUgAUjgjSQgbhVp6nX9SsLP/wHxs8HI4s9dZuQ/Uu4olKpqXo1S9UIfGDrH30uvOqNcpim44w7FbPS/l/jK4nedzAVz84qq+E0rfkgWEL65/UgkujFbbSdAIulyXLn6KP+PA7888QqbG8AmzJCs9E6Zgf8Yh9d uJwgFqPqvy06opXdtoev5uoMkifpydoJ4CBnaiX68jrAoJPCWE LBEVZYqjI1ilihmpcTYSowKjiRPuZlJAz3Iiz2gEL1BS/cdpjGJDXg9JCp7/Nc/c25cS//t4E2xCPP4eMEIwlnpVSZFafSmXVk+m8PUS7nttWln4tGpX33i9 szRha/ycVAWJAAJQAJvIQFbJdYZFlXtKwO+yqkkOkUilyPtOJn2P7p1i i7rBXaigybV1DN0jufrI0vSMXOwRZM2zE8yd3LhcAaFfrhmnQ0 Dzqe8PuJQMiQACbx9BNyUKnCKLNGzdvWuVLC5gkTMXZP6fBr5X 4GPxp305d5Jf8WT4TMhAUgAEnhbCGwiE3P6RLzUrkTtfUSjtwU SrCckAAlAAm83gap2+XMA+h7a6O0mCGsPCUACkMBbSEAd2ggmS AASgAQgAUigi4DmegSV2GgBEyQACUACkAAk0DMBOKromQ38BRK ABCABSEBNgJp0JekpCgG/GmKBBCABSAASeMsJ/PHHH88RoLrYOnbc0jfUT4Km4i1/QWD1IQFIABJAkAH2Tk/tgkQsAdd/HiTU8XdvKIHzry8euVjW3pu8fc6jbLh3ZP3iRVvuil7DGsorFK 6ou31o9YJ56y/VvvA8lD7XHxaABCABSOCNIfDULvT97GmUn3zylun0jxyfOUwDk 1QVVBsMdDXuu7hnkYCzteMOXGNNX7d7to4eiF3RWFxNd3Jk/Y9d5L2E2l14vzZ4q4TZMUfzHGZuOBhuyKD2S1QvlYfZIAFIABL 4Kwn0tW9H62/G1gUsmGtGFMSVgsL0B5VNvKQTcU2DJ/ggZQ/LMPc5ny8bpw6308eES/IPb03hbtxuawiE45KcvZ9uKJm6f88H3L4qqeXB3YVrydD7W2hV 7Ddbc9/bO8cGhPf7ixMmLr4StffEHxUkpwkrN/1rmFGnRqjgQUJ86oOMm7ntngu++XKqfcdxipi46Prl++UPU4us F2+O8GWS0Lor20/oLFo1plskkr+4UvDxkAAk8MYR+LOz0zf4nyc8IZgw/VSu+3g3EGUHROu8tH3TiRrbwLFDjaUq7oSPl6z4OnKJY+WNo+d KX2puSlIQdxsbPKQzmh1Jf+CsTzd9PYXzCuwEoN5deE/NoBJX5JdohKfrnltZnXKFZxFIhGr9qxMuLsmudwj7fte60eQna dn1nVNhiprErduzHWb9Z+N/l7uL7p88mUdMNYKAH7zz22JpY+csWjwKfZBdr0SwprvRWQM/fAfaib+6JeHzIYE3lABYw+7Q7Jm1iraODuUFGqM1SdcpQX4gYA UuKzr+7THB2AXBtjrS0qx6Qw8v0MXjyjaJErGwN3qZ7l1ek1lK cR3SFV4bQRg2/kM5nQeQv0CpXv2kIVxbKcWToytXx9Q/s36jJRcmKspuZA4cYPRqpsW0PKH3t0iGrqMC3SwMjcyMTN0nT3 BQD+XQqvitJ2Tjp3saksgsd38uIq+raSWMiKzoXBJl1FBTMqaU YUw2i9yaffq2/fyJNrTePxHmhAQggbeKwEutVbSXXM40HzWZMDKKihu3Gig+FiD Kmqz8XjnZZboNHRWkHv5N+O7nmyeqo2/iCn5mwsXESzf4vl/vXOpJxGNDm3Iunv3jYVFOLk/G8voo8qtJXDoYnlSmno9NvJL8UEj2WmunXgHB22syLpy40Dh+w 38G67bXZsWfPFMWsH6NnzTz/PHfUxkLd3zqqc6HCvMuRB1PKhbKFVIxxfez7asCusdz0yZcXVC LJqrmRzlCq3EuILx2V9KSTVmRVUXmTmGUxO2IOpMmGhSxZc1YC 4pWgbik5Gr06Wu3ssXDN+xaPhjI1cLk6bNe8gITl1z7PZm9MnK 8ndpSSB+dOc/jLPQyImpBpuuDmyidiISBCgrKyXZTGSSstayC9U6obvGFG+Yff tZR6iUfDotBApDAW0KgD7Mocl5qockwh46+HFOBwKuYUoUoajI LJZS2lH1bDyaTpmzb+/loC2Kloe3hif+eFAeEzfKmt4pRIjpre+mpTd9nucxbuWH7tnBb lY6HjxUdUQnuHNx6hR66/Nst4fY0rj/RT6OC9NjT8Rdi78kHcGnNOZfOJSbGpzYwGhL37PztdkYmz8ALR GAFCcyz7Pyp0Gv5tgOHdv57uI3/0oiR3e2ENuFEQU1NLFW8lJN7fjxVgaF5pw4dicttBp/hmtmAworaB0/kIO7otXKroBBnRJCbXafUnlNZfXF3LGP6um0/b187x4OwE5pM+vWS4dKKW7//vG3LTycvJuz7amtSAzF0kBZfyxBbDHYG4c5BUslAQHRda44B0c xkKiITtqHyypT73BkjZVevMKaH6ucnxBz8Ycuxu+rCMEECkAAk oJ0AWdWVGLovjg+k5Gfk0zztdNVydJxmf3vo8BdD9NHGvAcimw n+belpVaaDHFkdU09KXuy+2+xJPqIbl1pC1i311kdUDSkH47D3 prnrk/DWR6lVzCHe5lRcnBV1tCZwpj+IbV3/uMnQTT2tQzXznzZtiA6J7edmRDMZPHlmgBmFxkB0fT75YkkAk8 YdOZAF+kFMcPPnaGFAsLMeCRPci07BfZ4Lr61dOOg+tWhC07Ub PSPIBDEMWPr5iqXhU7xNqNqyURGVsCi/2Sh05erFk/0HObP1SCYOpmQtAqkIWnf7clZxUUUrwnJwNgVYNJlob5He36Ua e02NWPvdT4e2TDZry0p+AgIUKuoeFEmonAFmHZNK8rpiAcV+qA 3RslSroPmjFVdjEoqdF4aZ342TjRtWcfBo45CZ88aTk2PuNEJb 0XvyMCck8A8n8Kdd0GN0XJNFXam2pvZFtcellYVtZhzDjmEIiW HlyNYlIVhL/t0G9ugxgWNcsMKr2UIQixUkRWVySj1FdD3mvuWCdfO8gKOQquH e5XLrIB/is19anlZOdx/KpePCrPiHliFDjEmIgp/zWMHx6lqokFell+u4exPjEwRtLiyUu88LD3VlqqqzeExfTyK0n kqQllBs9i6xuIyLC1OrjX1dmd28VnsSrlUTQmf+/SK53TDbTnupPRv4kL9fw3AbTkyTYWJeSRv7HT/TJi1VI3rmUTOGtidELt14trQdbBHRZPIi2r35jURnGekCFCQ9O 2+OjsVANhhrKQXlTQiLbdRhKRQ190tULkEdk1EIxWTYgrWrw6e HDJInn24YHYJfikeDAtlUCl2HrsvUeQPWXnpTa5gHEoAEXj+BP +1CdW3HNdnWye7pvxcpoGwsFlBN9Lt3KFhzzj085F8TbEzcRtp hRdeymwlbgbWW5AkYLmMWRHzob0nHxJXlLaIn6dU6tlwwpJCWJ cc9kLK9ubT2yrRrpQbudsTNintpTWZedmAvRYtAiilqM3Jldn4 2OgiuQsWPMxos/V2A0xXamF+ocvYh7AnYBZheRbezMUQbMk/ticqjunp0ek511UFSklqmRbikVZsmTU1tTY8eCC19nYkJMImoH WvTmo1fmlWGO4xw1CNsVeb1WqepwRy5tpz1PD5qNW7d3m8mGxa mFoFY45pMNBytcFSBdm077OlaWwvh0vJc1buL3idWHTBUiYEA2 h3Z2ktvpKlGfBBg1m2WEeVfjy4dPtNdlJZPdeMyEKytpt7Yb2B 3Q6vtOfAeJAAJvC0EnrULHde99lXCpGDau5tXJa6oS0uoHLs23 BtMjDO8/Nn4KWArxoaYYTLQ18qamtukTQ05KWl13HET2TXNSlFzYvS+eAG KVshw3dwzh/nMVr5Eap6bEHWmuLagRqUnz7t4RmHqFzJCmZvZgLGKE489wpxG 2d7mMX0WG1MQvLk4q47hpii4dq7W0EUuVIhLf9//62B/LzMK3d4X2JVnE9rCq5VIDTSEB9lp0aTRfcIEVgFfKSq+feGISG YzZtoohrZsA7yRRy1MT2NEJsi/evim9aJVwRZIXfLzVYtevyepwGXlsR1BesamjsGzRwBsmkzet3 lGX1xembgjMs74050RXnoK7dfAWHYmXMLLLae5ulvrksFCeXp8 GnvhYn+1866OpYsp8jA7p17pYlV/5Xi+x7LvfTuWLTqLog1/HM/1XrDahFTUpsDZZAStz0g3mrRM7YkAEyQACUAC2gn0uofAlHK5t FWqHjWIck7sOHKXV1MrNHD0bLpgMP99H3Prd0axY2KuZgrGTrC w9A/2iD12aOPm3NHvh30wA6xCixycLIz5ZMuxy+ZSzm+oDg4M/WCiJz1XeLOons+cHrGEv6tcbu3gERpoBzylFGVtVI6bvaXHlGB 33aLdUXTPqdbgixls3q5XiMn5spC5k6yVWbcsnTzfCwsfhl9Z1 WL57nMLFQiF5TTQiqYpHBc1aGriBdZKKh3tTXgYajPx42FWdBK uVWFq2nenjNuvRq4sDZw8Z2HkVAs6WKxmalRtjt8wNHLfsR0Ha j3tfZdFuLNAJ07WYNJtuoxEoanEuKkjFfDt6fppA6KNGecOHbx VrzvA32+AjeuoyeHD1evWINHtpywe/+CnmI3fVHjbOM/+bsXgbts/wHr8r3ecw9ZbAcdmuzFDlfv3H5e5s2ctC+zuD6D9VYF3IQFI4O 0lQGoVtTyt/fUrCePGv6cJw5DJQhQlvyxZ8zgsaus4sK1CW1IJkjYu+bF24vb 9i1yJCX9cIVHS9EFn2p+EK4R1Yj0rU2J7BS6rKW4yGsDVx4Q5s WcfsSfNGgnmosR3N4UfsNz8y3K37sOK/jy1h7I4KpHievq0/lWpB+EIjqI4lfBqJdj1cN1T2V7fxzEUI1GJWOowQQKQACQgbhV pQtBqCLR3+5qFETrH30uvOqMcuNloTxSz0f9e4iuL33UyF8zNK 6riN634IVmg0p65t3dJdGO22k6ARNLluHL1Uf4fB3554hU2N4B NmCFZ6Z0yA/8xDq/dThAKUPVfl51QS++0Ey+47i22HvORyNBO9AgH/gAJQAI9Euj1BBSiN2jpvsM05gtO/6Cyx3/9M+fGtfTfDt4UizCPjxeMIJyVXmVSlEZv2pXlszmM2G6Gt9emn YlHp371iduL/XxfpQZQFiQACUACbx+B3psKhKLLeoGd6EBHNfUMneP5+jCSdMw cbNGkDfOTzJ1cOJxBoR+uWWfDgPMpr484lAwJQAKQAHD//3tBoHEnfbl30t9LZ6gtJAAJQAJ/dwK9Xqv4u1cU6g8JQAKQACTwsgSgqXhZcrAcJAAJQAJvDQFoKt 6apoYVhQQgAUjgZQlAU/Gy5GA5SAASgATeGgLQVLw1TQ0rCglAApDAyxKApuJlycFykAAk AAm8NQSgqXhrmhpWFBKABCCBlyXwMqYCnAB+8cjFsvaXfeYLyy kb7h1Zv3jRlruirgO5X5i9bz++QuGKutuHVi+Yt/5S7d8gJhAub21TvgaefaMPc0MCkMDflkDft+Ch/OSTt0ynf+T4zGEamKSqoNpgoKtx38U9Sw5vL407cI01fd3u2Tp 6IHZFY3E13cmR9WoOB+kuvF8bvFXC7JijeQ4zNxwMN2RQ+yXqd b04uLxFICbpGdBRYVXR3esPmFMXjjN4XQ+DciEBSOCfTqCvfTt afzO2LmDBXDOiIK4UFKY/qGziJZ2Iaxo8wQcpe1iGuc/5fNk4DnFGUx8TLsk/vDWFu3G7rSEQjkty9n66oWTq/j0fcPuqpJYHdxeuJUPvb6FVsd9szX1v7xwbEN7vL06YuPhK1N4 Tf1SQnCas3PSvYUadGinKfvt85fkGtXYs3082rQnmEIHxMHHR9 cv3yx+mFlkv3hzhyyShdVe2n9BZtGpMt0gkf3Gl4OMhAUjgjSP Qt84OE6afynUf7wai7ODtFZe2bzpRYxs4dqixVMWd8PGSFV9HL nGsvHH0XOlLzU1JCuJuY4OHdEazI+kPnPXppq+ncF6BnQDUuwv vqRlU4or8Eo3wdN1zK6tTrvAsAolQrX91wsUl2fUOYd/vWjea/CQtu/7pVJjs8ZVHnqt3/rj7x32Hf/t103QXEAUEBPzgnd8WSxs7Z9HiUeiD7HolgjXdjc4a+OE70E78 1S0Jnw8JvOkE+tTfoTVJ1ylBfiBgBS4rOv7tMcHYBcG2OtLSrH pDDy/QxePKNokSsbA3epnuXV6TWUpxHdIVXhtBGDb+QzmdB5D3F6OGc G0CFU+OrlwdU4+8kAkmKspuZA4cYPRqpsW06dHreyRD11GBbha GRmZGpu6TJzh0DuVwcd51QeCskS6OTo42FoZd8TVkReeSKKOGm pIxpQxjslnk1uzTt+3nT7TpCMTd66fCjJAAJPD2EehLp95ecjn TfNRk4nhZRcWNWw0UHwvwtSorv1dOdpluQ0cFqYd/E777+eaJ6uibIJRnZsLFxEs3+L5f71zqSXzXok05F8/+8bAoJ5cnY3l9FPnVJC4dDE8qU8/HJl5Jfigke621U6+A4O01GRdOXGgcv+E/g3Xba7PiT54pC1i/xk+aef7476mMhTs+9VTnQ4V5F6KOJxUL5QqpmOL72fZVAd3juW kTri6oRRNV86McodU4FxBeuytpyaasyKoic6cwSuJ2RJ1JEw2K 2LJmrAVFq0BcUnI1+vS1W9ni4Rt2LR8M5Gph8vRZL3mBiUuu/Z7MXhk5noitTSRMmBGfll+eOe8sy3HkB8sXjuGo40uhgoJyst1 UBglrLatgvROqW3zhhvmHn3WVesnHw2KQACTwVhB44Rd0dwJyX mqhyTCHjr4cU+EI+DxVIYqazEIJpS1l39aDyaQp2/Z+PtqCWGloe3jivyfFAWGzvOmtYhQHfVV76alN32e5zFu5Yfu2 cFuVjoePFR1RCe4c3HqFHrr82y3h9jSuP9FPo4L02NPxF2LvyQ dwac05l84lJsanNjAaEvfs/O12RibPwAtEYAVJUZO486dCr+XbDhza+e/hNv5LI0Z2txPahBMFNTWxVPFSTu758VQFhuadOnQkLrcZzOVoZ gMKK2ofPJGDuKPXyq2CQpwRQW52nVJ7TmX1xd2xjOnrtv28fe0 cD8JOaDLp1yuGSytu/f7zti0/nbyYsO+rrUkNnfNPqJQ6YunGzd+sGMd8fHnXD/E1nffJVEQmbEPllSn3uTNGyq5eYUwP1c9PiDn4w5Zjd7sK90sj WBgSgAT+qQR6byqU/Ix8mqedrpqEjtPsbw8d/mKIPtqY90BkM8G/LT2tynSQI6tjlKLkxe67zZ7kI7pxqSVk3VJvfUTVkHIwDntvmr s+CW99lFrFHOJtTsXFWVFHawJn+oPY1vWPmwzd1NM6VDP/adOG6JDYfm5GNJPBk2cGmFFoDETX55MvlgQwadyRA1nA8mCCmz 9HCwOCnfVImOBedAru81x4be3CEa2a0HTtRs8IMkEMA5Z+vmJp +BRvE6pWhRGVsCi/2Sh05erFk/0HObP1SCYOpmRtVUPQutuXs4qLKloRloOzKcCiyaS/7xTV2GtqxNrvfjq0ZbJZW1byk84AhXTuqEljvJxdR85dMZuD1B TUK9QPoloFzR+tuBqTUOy8MMz8bpxs3LCKg0cbh8ycN56cHHOn 8W/g89tfYLA8JAAJvCyBXpsKXFpZ2GbGMewoQGJYObJ1SQjWkn+3g T16TOAYF6zwarYQU+uhqExOqaeIrsfct1ywbp4XcBRSNdy7XG4 d5EN89kvL08rp7kO5dFyYFf/QMmSIMQlR8HMeKzheXQsV8qr0ch13b2J8gqDNhYVy93nhoa5MV XUWj+nrSYTWUwnSEorN3iUWl3FxYWq1sa8rs5vXak/CtWpC6My/XyS3G2bb6QKsPRv4kL9fw3AbTkyTYWJeSRv7HT/TJi1VI3rmUTOGtidELt14trQdbGnQZPKyTdZVjkRnGekCFCQ9O 2+OjsVA9vNBYyn6pgZ0K3cwFlInismwBWtXh08PGSRPPt0wOgS/FI8GBbKpFLoOXZep8wasvfSXCCwPCUACr4tAr02FsrFYQDXR79 6hYM059/CQf02wMXEbaYcVXctuJmwF1lqSJ2C4jFkQ8aG/JR0TV5a3iJ6kV+vYcsGQQlqWHPf+b9bgAAAgAElEQVRAyvbm0t or066VGrjbETcr7qU1mXnZgb0ULQIppqjNyJXZ+dnoILgKFT/OaLD0dwFOV2hjfqHK2YewJ2AXYHoV3c7GEG3IPLUnKo/q6tHpOdUFSlKSWqZFuKRVmyZNTW1Njx4ILX2diQkwiagda9Oaj V+aVYY7jHDUI2xV5vVap6nBHLm2nPU8Pmo1bt3ebyYbFqYWgVj jmkw0HK1wVIF2bZPr6Vrba4BLy3NV7y56//lVB0yYn6Z495MQ6+7rUSj/enTp8JnuorR8qhuXgWBtNfXGfgO7G1ptz4H3IAFI4O0l0OtlbU zaLEW6eVXiirq0hMqxa8O9DUkIw8ufjZ8CtmJsiBkmA32trKm5 TdrUkJOSVscdN5Fd06wUNSdG74sXoGiFDNfNPXOYz2zlS6TmuQ lRZ4prC2pUevK8i2cUpn4hI5S5mQ0Yqzjx2CPMaZTtbR7TZ7Ex BcGbi7PqGG6Kgmvnag1d5EKFuPT3/b8O9vcyo9DtfYFdeTahLbxaidRAQ3iQnRZNGt0nTGAV8JWi4ts XjohkNmOmjWJoyzbAG3nUwvQ0RmSC/KuHb1ovWhVsgdQlP1+16PV7kgpcVh7bEaRnbOoYPHsEwKbJ5H2 bZ/TF5ZWJOyLjjD/dGeGlp9B+DYxlZ8IlvNxymqu7tS4ZLJSnx6exFy72Vzvv4tLaJ 0KmE0cfbXqYeLbI74uIod1tANrwx/Fc7wWrTUhFbQqcTUbQ+ox0o0nL1J4IMEECkAAkoJ1Ar3sITCmX S1ul6lGDKOfEjiN3eTW1QgNHz6YLBvPf9zG3fmcUOybmaqZg7A QLS/9gj9hjhzZuzh39ftgHM8AqtMjBycKYT7Ycu2wu5fyG6uDA0A8m etJzhTeL6vnM6RFL+LvK5dYOHqGBdsBTSlHWRuW42Vt6TAl21y 3aHUX3nGoNRhJg83a9QkzOl4XMnWStzLpl6eT5Xlj4MPzKqhbL d59bqEAoLKeBVjRN4bioQVMTL7BWUulob8LDUJuJHw+zopNwrQ pT0747Zdx+NXJlaeDkOQsjp1oA1yKcqVG1OX7D0Mh9x3YcqPW0 910W4c4CnThZg0m36TIShaYS46aOVMC3p+unDYg2Zpw7dPBWve 4Af78BNq6jJocPN1CPDsGG9It7ImMKMLab+5CQefMX26v3U/yZwHr8r3ecw9ZbAcdmuzFDlfv3H5e5s2ctC+zuD9CtDPwDEoAE IAGE1CpqeYrh+pWEcePf06RiyGQhipJflqx5HBa1dRzYVqEtqQ RJG5f8WDtx+/5FrsSEP66QKGn6aj/Nl0+4Qlgn1rMyJbZX4LKa4iajAVx9TJgTe/YRe9KskWAuSnx3U/gBy82/LHd7fqr+5Z/aQ0kclUhxPf2ufQo95HrZ2ziK4lRqB9uerl9W9tNyOIZiJCqlf 43Sby2gAEgAEngzCIhbRZqKaDUE2rt9zcIInePvpVedUd7pZqO Zg2I2+t9LfGXxu07mgrl5RVX8phU/JAtUmhn7codEN2ar7QRIJF2OK1cf5f9x4JcnXmFzA9iEGZKV3i kz8B/j8NrtBKEAVf912Qm19E478YLrvqDTmpdEhnZCKxh4ExKABF5Io NcTUIjeoKX7DtOYxAa8HhKVPf7rnzk3rqX/dvCmWIR5fLxgBOGs9CqTojR6064sn81hhFcP3l6bdiYenfrVJ2 7PHF34Kh8HZUECkAAkAAkAAr03FQhFl/UCO9FBk2rqGTrH8/WRJemYOdiiSRvmJ5k7uXA4g0I/XLPOhgHnU14fcSgZEoAEIIE+mYo3AReNO+nLvZPeBE2gDpAAJA AJvD0Eer1W8fYggTWFBCABSAAS6E4Amgr4RkACkAAkAAn8DwLQ VPwPQPBnSAASgAQgAWgq4DsACUACkAAk8D8IUHG869yh/5ET/gwJQAKQACTwVhDQtAtwVPFWNDysJCQACUAC/SFAbW1q7U95WBYSgAQgAUjgH0ZA0y6QmabMp/96WVtwAvjFIxfL2nuZvW/ZlA33jqxfvGjLXdFrmBh7hcIVdbcPrV4wb/2lWhgTqG8tDHNDApDAm07gWbvQcd2H3dqdlUP5ySdvmU7/yPGZwzQwSVVBtcFAV+O+i3sWGTgZNe7ANdb0dbtn6+iB2BWNxd V0J0fWqzkcpLvwfm3wVgmzY47mOczccDDckEHtl6hX9MbgcgFP oGvL0YcTiq+IKBQDCUACzxLoa9eC1t+MrQsAIRgIo4ArBQW3r1 +Ji1qzYNmOM7/v++6LJeGrdl+v6YjQ2VfQuCT/8NYU7vuBtob6+nREkrv308+3XeW/mo/27sL71bujVbHfbM0dNGm4DfPNsBMgBsXWJcuP8LA/q4UKHsQd+embiBnTPv7mQoW8qykwcdHVU9EHNkSs3H+/lRi1oXVXftj2R38PdexrS8P8kAAk8Hcj0DdTgQnTT+W6j3cDUX bw9opL2zedqLENHDvUWKriTvh4yYqvI5c4Vt44eq70peamJAVx t7HBQzqj2ZH0B876dNPXUzj9G6h0tUd34T21kkpckV+iEZ6ue2 5ldcoVnkUgEar1TUi4OO/0LxeyhFbDQJTxDoUUNYlbt2c7zPrPxv8udxfdP3kyT6L+QcE7v y2WNnbOosWj0AfZ9UoEa7obnTXww3e6Rax6EyoFdYAEIIE3jEC f+ju0Juk6JcgPBKzAZUXHvz0mGLsg2FZHWppVb+jhBbp4XNkmU SIW9kYv073LazJLKa5DusJrIwjDxn8op/MA8v5S0xCuTaDiydGVq2PqkRcywURF2Y3MgQOMXs20mDY9+nAP BMz7NcPCh0k293br1Aitit96QjZ+uqchicxy9+ci8rqaVmJoJi s6l0QZNdSUjCllGJPNIrdmn75tP3+iDa0PD4RZIQFI4O0jgKJo Xzr19pLLmeajJhPHyyoqbtxqoPhYgA9ZWfm9crLLdBs6Kkg9/Jvw3c83T1RH3wShPDMTLiZeusH3/XrnUk/ikxdtyrl49o+HRTm5PBnL66PIryZx6WB4Upl6PjbxSvJDIdlrr Z16BQRvr8m4cOJC4/gN/xms216bFX/yTFnA+jV+0szzx39PZSzc8amnOh8qzLsQdTypWChXSMUU38+2r wroHs9Nm3B1QS2aqJof5QitxrmA8NpdSUs2ZUVWFZk7hVEStyP qTJpoUMSWNWMtKFoF4pKSq9Gnr93KFg/fsGv5YCBXC5Onz+r7BdaSeeK0dMJCu8QVJOdpnTZW+ujMeR5no ZcRUQsyHczkISidiISBCgrKyXZTGSSstayC9U6obvGFG+YffvZ 8RO6+qwFLQAKQwD+TQENDQ0LC5fyH+cJm4Qu/oLtXX85LLTQZ5tDRl2MqHAGfpypEUZNZKKG0pezbejCZNGXb3s 9HWwBDgbc9PPHfk+KAsFne9FYxioN+q7301Kbvs1zmrdywfVu4 rUrHw8eKjqgEdw5uvUIPXf7tlnB7Gtef6KdRQXrs6fgLsffkA7 i05pxL5xIT41MbGA2Je3b+djsjk2fgBSKwggTmWXb+VOi1fNuB Qzv/PdzGf2nEyO52QptwoqCmJpYqXsrJPT+eqsDQvFOHjsTlNoPPcM 1sQGFF7YMnchB39Fq5VVCIMyLIza5Tas+prL64O5Yxfd22n7ev neNB2AlNJv15v+S8S7/cc/xoqkNbfoGUO9ROVy1MWnwtQ2wx2BmEOwdJJQMB0XWtOep4qmQq IhO2ofLKlPvcGSNlV68wpofq5yfEHPxhy7G7Da9mSag/FYJlIQFI4E0hcP/+/ZkzZ4WEjv89JubRo4Ky8vLemwolPyOf5tnZI+k4zf720OEvhui jjXkPRDYT/NvS06pMBzmyOkYpSl7svtvsST6iG5daQtYt9dZHVA0pB+Ow96a 565Pw1kepVcwh3uZUXJwVdbQmcKY/iG1d/7jJ0E09rUM18582bYgOie3nZkQzGTx5ZoAZhcZAdH0++WJJAJP GHTmQBfpBTHDz52hhQDCYoccE96JTcJ/nwmtrFw66Ty2a0HTtRs8IMkEMA5Z+vmJp+BRvE6q2bFREJSzKb zYKXbl68WT/Qc5sPZKJgylZi0AqWDC+fTmruKiiFWE5OJsCLJpM+vFW4OLc6G jRhPDRFpTW4vuNpoNdmGrboKh7UCShcgaYdUwqyeuKBRT7oTaE eadaBc0frbgak1DsvDDM/G6cbNywioNHG4fMnDeenBxzpxHain60BywKCfyzCITNm98oEOj p6RUVFeXm5paUlPTaVODSysI2M45hRwESw8qRrUtCsJb8uw3s0 WMCx7hghVezhZial6IyOaWeIroec99ywbp5Xkwy6J/vXS63DvIhPvul5WnldPehXDouzIp/aBkyxJiEKPg5jxUcr66FCnlVermOuzcxPkHQ5sJCufu88FBXpq o6i8f09SRC66kEaQnFZu8Si8u4uDC12tjXtaOz7GqwnoRr1YTQ mX+/SG43zLbTBVh7Nlxacb+G4TacmCbDxLySNvY7fqZNWqpG9MyjZg xtT4hcuvFsaTtwNtJk0o9XS14W/+v1vMQfIubPC1uyu0jBj//2u7gq4HimFJQ3ISy2UYelUNTcL1G5BHVMRiEUk2EL1q4Onx4yS J58umF0CH4pHg0KZFMpdB26LlPnjVh76QcTWBQSgAReGQEajVZ RUVFdXa2nr29hbW1ibtlrU6FsLBZQTfS7dyhYc849PORfE2xM3 EbaYUXXspsJW4G1luQJGC5jFkR86G9Jx8SV5S2iJ+nVOrZcMKS QliXHPZCyvbm09sq0a6UG7nbEzYp7aU1mXnZgL0WLQIopajNyZ XZ+NjoIrkLFjzMaLP1dgNMV2phfqHL2IewJ2AWYXkW3szFEGzJ P7YnKo7p6dHpOdbGSlKSWaREuadWmSVNTW9OjB0JLX2diAkwia sfatGbjl2aV4Q4jHPUIW5V5vdZpajBHri1nPY+PWo1bt/ebyYaFqUUg1rgmEw1HKxxVoF3bDnu67qydjlPY7t9jfjt58uTh jUHGVLdVB6Mip9gALhiqxBBSZ6O2l95IU434IMCsWyOj/OvRpcNnuovS8qluXAaCtdXUG/sN7G5oX9kbBwVBApDA35BAe3s7Q1eXZWlGYjDAMYFgn1uvl7Ux KZj27uZViSvq0hIqx64N9wYT4wwvfzZ+CtiKsSFmmAz0tbKm5j ZpU0NOSlodd9xEdk2zUtScGL0vXoCiFTJcN/fMYT6zlS+RmucmRJ0pri2oUenJ8y6eUZj6hYxQ5mY2YKzixGOP MKdRtrd5TJ/FxhQEby7OqmO4KQqunas1dJELFeLS3/f/Otjfy4xCt/cFduXZhLbwaiVSAw3hQXZaNGl0nzCBVcBXiopvXzgiktmMmTaK oS3bAG/kUQvT0xiRCfKvHr5pvWhVsAVSl/x81aLX70kqcFl5bEeQnrGpYzDYhEJBNJm8b/OMvri8MnFHZJzxpzsjvPSAW5O2a2AsNRLakJMr1Pe06vKT1bF0 MUUeZufUK12s6q8cz/dY9r1vx7JFZ1GwB+N4rveC1SakojYFziYjaH1GutGkZWpPBJgg AUgAEiAIYHp0HEPa2yQ4hU7RpasovfeAwpRyubRVqh41iHJO7D hyl1dTKzRw9Gy6YDD/fR9z63dGsWNirmYKxk6wsPQP9og9dmjj5tzR74d9MAOsQoscnC yM+WTLscvmUs5vqA4ODP1goic9V3izqJ7PnB6xhL+rXG7t4BEa aAc6PUVZG5XjZm/pMSXYXbdodxTdc6o1+GIGm7frFWJyvixk7iRrZdYtSyfP98LCh +FXVrVYvvvcQgVCYTkNtKJpCsdFDZqaeIG1kkpHexMehtpM/HiYFZ2Ea1WYmvbdKeP2q5ErSwMnz1kYOdWCDharmRpVm+M3DI3 cd2zHgVpPe99lEe4s8FVP1mDSreMnUWgqMW7qSAV8e7rW8grjk moe3W2o41OnLbr9lMXjH/wUs/GbCm8b59nfrRjcbfsHWI//9Y5z2Hor4NhsN2aocv/+4zJ39qxlgd39AbQ8Cd6CBCCBt4gASa4AzkhkDKHTwCYI1NDGn CRqET4FkHT18rjx72nyMGSyEEXJL0vWPA6L2joObKvQllSCpI1 LfqyduH3/Ildiwh9XSJQ0fdCZ9ifhCmGdWM/KlNhegctqipuMBnD1MWFO7NlH7EmzRoK5KPHdTeEHLDf/styt+7CiP0/toSyOSqS4nj6tf1XqQTiCoyhOJbxaCXY9XPdUttf3cQzFSFTK6 6lBr7WAGSEBSODNICBuFWkqcv1KwqdffqmQK3WZ+gYGenKMbDo AzED0MtE5/l561Rnlsp7yU8xG/3uJryx+18lcMDevqIrftOKH5P4eGUGiG7PVdgIkki7HlauP8v8 48MsTr7C5AWzCDMlK75QZ+I9xeO12glCAqv+67IRaeqedeMF1T +x7fZ9Ehnai17BgRkjgrSVAoVIpKE4hU30GGUwdyyK3q3o/Ra03aOm+wzQmsQGvh0Rlj//6Z86Na+m/HbwpFmEeHy8YQTgrvcqkKI3etCvLZ3MYmJACG/Vq087Eo1O/+sTtmaMLX+XjoCxIABKABN5CAjj4Kibpq+ikBin28SD7WkV770 0FQtFlvcBOdNCkmnqGzvF8fWRJOmYOtmjShvlJ5k4uHM6g0A/XrLNhwPmU10ccSoYEIIG3jwBGp9IYNKsBNtZeA3Tb64P9PftgK t4EXDTupC/3TnoTNIE6QAKQACTwTyVgxDZTtLe3tUhqymqixZSpHw/u9VrFPxUJrBckAAlAApBAdwI4lUbWocvJJGpNw+BBbkxxNTQV8 B2BBCABSAAS6EbAwJxtam7qR24zlqtGjhr+R1Lq32wCCrYnJAA JQAKQwOsm4GGOmd5Nk5EMfcYENTY1XM8thKOK180cyocEIAFI4 G9GIDD3qh0D44kolmxO1oPi4koxHFX8zZoQqgsJQAKQwOsm8ER lhjm6TRwXaMhxfnjznpxEgqbidTOH8iEBSAAS+JsRSLHy9Ne1m f7hLAoJ14mhmFAVcALqb9aEUF1IABKABF47gfxiDr1SBynF5Rk MeYVYLH8ZUwFOAL945GJZ+2vRVtlw78j6xYu23BV1Hcj9Ch/zCoUr6m4fWr1g3vpLtX+DmEC4vLVN+Rp4vsKmgaIgAUjgzSGgr 2wruZ8tbTqBSY+RpM3hc4P6PgGF8pNP3jKd/pHjM4dpYJKqgmqDga7GfRf3LBy8vTTuwDXW9HW7Z4Pz0YmjZKv pTo6sV3M4SHfh/drgrRJmxxzNc5i54WC4IYPaL1Gv6N3A5QKeQNeWo99l+nF5i0B M0jOgo8KqorvXHzCnLhxn8IoeBsVAApDAP5wAmami6WI1OdmGJ uwZU0Zj5L4c7KFmg9bfjK0LWDDXjDAKuFJQmP6gsomXdCKuafA EH6TsYRnmPufzZeM4xBlNfUy4JP/w1hTuxu22hkA4LsnZ++mGkqn793zA7Z8BUqvRXXgfNeueHa2K/WZr7nt759iA8H5vQgIxKHasPiD5V1Qkp0sdRdlvn68836D+k+X 7yaY1wRwiMB4mLrp++X75w9Qi68WbI3yZJLTuyvYTOotWjekWi eRNqBTUARKABP46Am4jA7yZoj/u0+dOcyA7KipLmvvW2WHC9FO57uPdQJQdvL3i0vZNJ2psA8cON ZaquBM+XrLi68gljpU3jp4rfam5KUlB3G1s8JDOaHYk/YGzPt309RTOK7ATAHh34T21gEpckV+iEZ6ue25ldcoVnkUgEar 1TUi4OO/0LxeyhFbDQJTxLoVkj6888ly988fdP+47/Nuvm6a7qH9S8M5vi6WNnbNo8Sj0QXa9EsGa7kZnDfzwHWgn3oS WhDpAAm8QAYp1XrWJs7NdU2ulUoY3Nwn61N+hNUnXKUF+IGAFL is6/u0xwdgFwbY60tKsekMPL9DF48o2iRKxsDd6me5dXpNZSnEd0hV eG0EYNv5DOZ0HkPcXoYZwbQIVT46uXB1Tj7yQCSYqym5kDhxg9 GqmxbTp0Yd7IGDerxkWPkyyubfbU42A9bguCJw10sXRydHGwrA rvoas6FwSZdRQUzKmlGFMNovcmn36tv38iTYdgbj78FSYFRKAB P7ZBIpKH3o627LtTO9ntdXzG3kKy7506u0llzPNR00mjpdVVNy 41UDxsQBfq7Lye+Vkl+k2dFSQevg34bufb56ojr6JK/iZCRcTL93g+369c6kn8V2LNuVcPPvHw6KcXJ6M5fVR5FeTuHQw PKlMPR+beCX5oZDstdZOvQKCt9dkXDhxoXH8hv8M1m2vzYo/eaYsYP0aP2nm+eO/pzIW7vjUU50PFeZdiDqeVCyUK6Riiu9n21cFdI/npk24uqAWTVTNj3KEVuNcnoaU05pNWZFVReZOYZTE7Yg6kyYaF LFlzVgLilaBuKTkavTpa7eyxcM37Fo+GMjVwoTQ5iUT1pJ54rR 0wkK7xBUk52lPbSwmzIhPyy/PnHeW5Tjyg+ULx3DU8aVQQUE52W4qg4S1llWw3gnVLb5ww/zDz+xeYqrwJfWFxSABSOBvQoD3sIzHJHm6eFN1jB7zsMEDZC/8gu5eKzkvtdBkmENHX46pcBBNT6lCFDWZhRJKW8q+rQeTSVO27 f18tAWx0tD28MR/T4oDwmZ501vFKAi9h7SXntr0fZbLvJUbtm8Lt1XpePhY0RGV4M 7BrVfoocu/3RJuT+P6E/00KkiPPR1/IfaefACX1pxz6VxiYnxqA6Mhcc/O325nZPIMvEAEVpAUNYk7fyr0Wr7twKGd/x5u4780YmR3O6FNOFFQUxNLFS/l5J4fT1VgaN6pQ0ficpuBW5NmNqCwovbBEzmIO3qt3CooxBkR5 GbXKbXnVFZf3B3LmL5u28/b187xIOyEJpP+vDZy3qVf7jl+NNWhLb9Ayh1qp9slDJVSRyzdu PmbFeOYjy/v+iG+ptNFi0xFZMI2VF6Zcp87Y6Ts6hXG9FD9/ISYgz9sOXa34W/gx9UfWrAsJAAJ9IGAApXJZJKKavJAt2EDXKnVJfW9NxVKfkY+z bOzR9Jxmv3tocNfDNFHG/MeiGwm+Lelp1WZDnJkdYxSlLzYfbfZk3xENy61hKxb6q2PqBpS DsZh701z1yfhrY9Sq5hDvM2puDgr6mhN4Ex/ENu6/nGToZt6Wodq5j9t2hAdEtvPzYhmMnjyzAAzCo2B6Pp88sWSACa NO3IgC1geTHDz52hhQDCYoccE96JTcJ/nwmtrF45o1YSmazd6RpAJYhiw9PMVS8OneJtQtSqMqIRF+c1Go StXL57sP8iZrUcycTAla6sagtbdvpxVXFTRirAcnE0BFk0mfWi 457Pi4tzoaNGE8NEWlNbi+42mg12YTxcq6NxRk8Z4ObuOnLtiN gepKahXqEtTrYLmj1ZcjUkodl4YZn43TjZuWMXBo41DZs4bT06 OudMIbcXzkOHfkMBbS0DXkE6jkO8/qswrqyOxyNMm2/TaVODSysI2M45hRwESw8qRrUtCsJb8uw3s0WMCx7hghVezhZga raIyOaWeIroec99ywbp5XsBRSNVw73K5dZAP8dkvLU8rp7sP5d JxYVb8Q8uQIcYkRMHPeazgeHVNosir0st13L2J8QmCNhcWyt3n hYe6MlXVWTymrycRWk8lSEsoNnuXWFzGxYWp1ca+rn92loQOPQ nXqgmhM/9+kdxumG2nC7D2bLi04n4Nw204MU2GiXklbex3/EybtFSN6JlHzRjanhC5dOPZ0nawpUGTST/eQnlZ/K/X8xJ/iJg/L2zJ7iIFP/7b7+KqOkzCn2Ip+qYGdCt3MBZSJ4rJsAVrV4dPDxkkTz7dMDoE vxSPBgWyqRS6Dl2XqfNGrL30gwksCglAAq+MANNQT9SOO3ONFX K6DqpIvlrTa1OhbCwWUE30u3coWHPOPTzkXxNsTNxG2mFF17Kb CVuBtZbkCRguYxZEfOhvScfEleUtoifp1Tq2XDCkkJYlxz2Qsr 25tPbKtGulBu52xM2Ke2lNZl52YC9Fi0CKKWozcmV2fjY6CK5C xY8zGiz9XYDTFdqYX6hy9iHsCdgFmF5Ft7MxRBsyT+2JyqO6en R6TnWxkpSklmkRLmnVpklTU1vTowdCS19nYgJMImrH2rRm45dm leEOIxz1CFuVeb3WaWowR64tZz2Pj1qNW7f3m8mGhalFINa4Jh MNRyscVaBd2+R6uu6snY5T2O7fY347efLk4Y1BxlS3VQejIqfY PLfqgAnz0xTvfhJi3X09CuVfjy4dPtNdlJZPdeMyEKytpt7Yb2 B3Q/vK3jgoCBKABP6GBIRt5FKBQo8kyn9U9ghsxauT9npZG5M2S5Fu XpW4oi4toXLs2nBvQxLC8PJn46eArRgbYobJQF8ra2pukzY15K Sk1XHHTWTXNCtFzYnR++IFKFohw3VzzxzmM1v5Eql5bkLUmeLa ghqVnjzv4hmFqV/ICGVuZgPGKk489ghzGmV7m8f0WWxMQfDm4qw6hpui4Nq5WkMXu VAhLv19/6+D/b3MKHR7X2BXnk1oC69WIjXQEB5kp0WTRvcJE1gFfKWo+PaFIyK ZzZhpoxjasg3wRh61MD2NEZkg/+rhm9aLVgVbIHXJz1ctev2epAKXlcd2BOkZmzoGzx4BsGkyed/mGX1xeWXijsg44093RnjpAbcmbdfAWGoktCEnV6jvadXlJ4tLa 58ImU4cfbTpYeLZIr8vIoZ2twFgD8bxXO8Fq01IRW0KnE0GG2U y0o0mLVN7IsAECUACkABBgGNtJmyorxdIbDmUJrnKSLf326sxp VwubZWqRw2inBM7jtzl1dQKDRw9my4YzH/fx9z6nVHsmJirmYKxEyws/YM9Yo8d2rg5d/T7YR/MAKvQIgcnC2M+2XLssrmU8xuqgwNDP5joSc8V3iyq5zOnRyzh7 yqXWzt4hAbagU5PUdZG5bjZW3pMCXbXLdodRfecag2+mMHm7Xd klHoAACAASURBVHqFmJwvC5k7yVqZdcvSyfO9sPBh+JVVLZbvP rdQgVBYTgOtaJrCcVGDpiZeYK2k0tHehIehNhM/HmZFJ+FaFaamfXfKuP1q5MrSwMlzFkZOtQCuRThTo2pz/IahkfuO7ThQ62nvuyzCnQWGbmQNJt06fhKFphLjpo5UwLenay2 vMC6p5tHdhjp2Om2BDekX90TGFGBsN/chIfPmL7b/c6uFujRYj//1jnPYeivg2Gw3Zqhy//7jMnf2rGWB3f0BtDwJ3oIEIIG3iEBZQS3HmnTmVuFXC0aaso3E cgpJ1CJ8CiDp6uVx49/T5GHIZCGKkl+WrHkcFrV1HNhWoS2pBEkbl/xYO3H7/kWuxIQ/rpAoafpqP82XT7hCWCfWszIltlfgspriJqMBXH1MmBN79hF70q yRYC5KfHdT+AHLzb8sd+s+rHj5Z/ZYEkclUlxPv2ufQo/5Xu4HHEVxKrWDbU/XLyf5mVI4hmIkKqV/jdJvLaAASAASeDMIiFtFmopcv5Lw/776SqRDnxJoIMbYE8dOd9a/p73b1yyM0Dn+XnrVGeUyLb+pb1HMRv97ia8sftfJXDA3r6iK37 Tih2SBqqfsvbtPohuz1XYCJJIux5Wrj/L/OPDLE6+wuQFswgzJSu+UGfiPcXjtdoJQgKr/uuyEWnqnnXjBde+gvSAXiQztxAvwwJ8gAUhATWDOT58Nendkbi 0ydgj1Ttr5HSdqez9FrTdo6b7DNCaxAa+HRGWP//pnzo1r6b8dvCkWYR4fLxhBOCu9yqQojd60K8tncxixhIu316ad iUenfvWJ2zNHF77Kx0FZkAAkAAm8hQRwY67LSCRqXTLvUUPgMB ZCUvTeVCAUXdYL7EQHTaqpZ+gcz9dHlqRj5mCLJm2Yn2Tu5MLh DAr9cM06GwacT3l9xKFkSAASePsInFyz1YBOivDWiS7GUyr09J TtVBz/O8UxoHEnfbl30tvXcLDGkAAkAAn83xFwQkSjOTpDOEaDZnpcYn oXxyb1YVTxf6cmfBIkAAlAApDAX0egpIVmJESsuIj0Ea+isbW1 tAaair+uNeCTIQFIABJ4IwkwTFj3ysX2huiH1kIZrYXu/afTzRupL1QKEoAEIAFI4P+cgMukEeYOFnFPqJ+lkRjtZF8MhaO K//NGgA+EBCABSODNJmBRkDPAjtVKVshk7XeaSHfFT2Mxv9l6Q+0g AUgAEoAE/s8IzJ50z5acbqfgy1ua/QcUcIyq4Kji/ww+fBAkAAlAAn8PAp+uMzWhIWDDWo5ImZbENTc2gKbi79FyUEt IABKABP7PCHAMDQxMTYRtbRQyWsOv0yOZ9/pgj2d0BCeAXzxysaz9taitbLh3ZP3iRVvuil7Dfo9XKFxRd/vQ6gXz1l+qfYNiAuHy1jbla+D2WloaCoUEIIE3lYDf4PIWMsXK zdLbQc9Ej6ZQvcSyNspPPnnLdPpHjs8cpoFJqgqqDQa69v6gWq 2EwMmocQeusaav2z1bRw/ErmgsrqY7ObJezeEg3YX3a4O3SpgdczTPYeaGg+GGDGq/RGnl0IebuLxFICbpGdBRYVXR3esPmFMXjjPoQ3mYFRKABCABDQ JbzuEKNPMhQ2VmrlKhSGFpc18noND6m7F1AQvmmhEFcaWgMP1B ZRMv6URc0+AJPkjZwzLMfc7ny8Zxnguzo6GJlhu4JP/w1hTuxu22hkA4LsnZ++mGkqn793zA7auS/1O4lgy9v4VWxX6zNfe9vXNsQHi/vzgpyn77fOX5BrUWLN9PNq0J5tCIa1TwICE+9UHGzdx2zwXffD nVvuM4RUxcdP3y/fKHqUXWizdH+DJJaN2V7Sd0Fq0a0y0SyV9cKfh4SAAS+IsJePv 4g+Da7e3tIrmcaoha66n61gtjwvRTue5hY0CUHby9ImHPoZLBE UvHOrTGH+VO+HjJB1x51ncLIo+eCw1c8RIH+EkK4m5jg7/vjGZH0h8469NN5MGcvmnYE+DuwnvKpRJXFNQZu7u8aCCjrE65w rOYQoRq/cuT7PGVR56rd77PodAMza3MDTvOR1fUJG7fUztp/X8mT/XYsHDXyZN54772A8d3KXjnt8WarFi7aAR55XfZ9UpfA/Hd6KyBH66FduIvb0moACTwRhHY8sP/A3YCJLlcDv6jSmWfOmK0Juk6JWg1CFiBy4qOf3tMMPdgsK1O66 2sekMPL9DF421tEiViYW/UJ6mdhOQ1maUU1yld4bURhGHjP/RV0dMQrk2w4snRlV+WfRL1nYu2XzvvYaKi7EbmwAFGr2Za7AVP +p8/4eK864LA2SNduplTtCp+6wnZtJ89DUkklbs/F8mrq2lF/fSpsqJzSZRRW03JmEiGMdkscmv26dv287+wUY9DYIIEIAFIoIv AwIEDn4PRl069veRypvmoycTxsoqKG7caKD4WIMqarPxeOdllu g0dFaQe/k347uebJ6qjb+IKfmbCxcRLN/i+X+9c6knEY0Obci6e/eNhUU4uT8by+ijyq0lcOhieVKaej028kvxQSPZaa6deAcHbazI unLjQOH7DfwbrttdmxZ88Uxawfo2fNPP88d9TGQt3fOqpzocK8 y5EHU8qFsoVUjHF97PtqwK6x3PTJlxdUIsmquZHOUKrcS6dIeV 6yKasyKoic6cwSuJ2RJ1JEw2K2LJmrAVFq0BcUnI1+vS1W9ni4 Rt2LR8M5GphQjzm5RImzIhPyy/PnHeW5Tjyg+ULx3CIAB7SR2fO8zgLvYyIJRQyXR/MBKJ0YlM+KigoJ9tNZZCw1rIK1juhusUXbph/+JndS0wVvpy+sBQkAAn8fQn0YRZFzkstNBnm0NGXYyocIWNKFa KoySyUUNpS9m09mEyasm3v56MtiJWGtocn/ntSHBA2y5veKkZx0G+1l57a9H2Wy7yVG7ZvC7dV6Xj4WNFBBM8 7B7deoYcu/3ZLuD2N60/006ggPfZ0/IXYe/IBXFpzzqVziYnxqQ2MhsQ9O3+7nZHJM/ACEVhBAvMsO38q9Fq+7cChnf8ebuO/NGJkdzuhTThRUFMTSxUv5eSeH09VYGjeqUNH4nKbgVuTZjagsK L2wRM5iDt6rdwqKMQZEeRm1ym151RWX9wdy5i+btvP29fO8SDs hCaTfr03qJQ6YunGzd+sGMd8fHnXD/E1hCuWtPhahthisDMIdw6SSgYCoutacwyIZiZTEZmwDZVXptzn zhgpu3qFMT1UPz8h5uAPW47dbXiD/Lj6RQUWhgQggddBoPemQsnPyKd52umqtdBxmv3tocNfDNFHG/MeiGwm+Lelp1WZDnJkdYxSlLzYfbfZk3xENy61hKxb6q2PqBpS DsZh701z1yfhrY9Sq5hDvM2puDgr6mhN4Ex/ENu6/nGToZt6Wodq5j9t2hAdEtvPzYhmMnjyzAAzCo2B6Pp88sWSACa NO3IgC/SDmODmz9HCgGBnPRImuBedgvs8F15bu3DQfWrRhKZrN3pGkAli GLD08xVLw6d4m1C1ZaMiKmFRfrNR6MrViyf7D3Jm65FMHEzJWg RSEbTu9uWs4qKKVoTl4GwKsGgy6Wd70rmjJo3xcnYdOXfFbA5S U1CvAOaz7kGRhMoZYNYxqSSvKxZQ7IfaEOadahU0f7TiakxCsf PCMPO7cbJxwyoOHm0cMnPeeHJyzJ1GaCv62R6wOCTwTybQa1OB SysL28w4hh0FSAwrR7YuCcFa8u82sEePCRzjghVezRZialaKyu SUeoroesx9ywXr5nkBRyFVw73L5dZBPsRnv7Q8rZzuPpRLx4VZ 8Q8tQ4YYkxAFP+exguPVtVAhr0ov13H3JsYnCNpcWCh3nxce6s pUVWfxmL6eRGg9lSAtodjsXWJxGRcXplYb+7oyu3mt9iRcqyaE zvz7RXK7YbadLsDas+HSivs1DLfhxDQZJuaVtLHf8TNt0lI1om ceNWNoe0Lk0o1nS9vBVgdNJq/staLomxrQrdzBmAdRCsqbEBbbqMNSKGrul6hcgjomoxCKybAFa 1eHTw8ZJE8+3TA6BL8UjwYFsqkUug5dl6nz16+9vDIgUBAkAAm 8agK9NhXKxmIB1US/e4eCNefcw0P+NcHGxG2kHVZ0LbuZsBVYa0megOEyZkHEh/6WdExcWd4iepJerWPLBUMKaVly3AMp25tLa69Mu1Zq4G5H3Ky4 l9Zk5mUH9lK0CKSYojYjV2bnZ6OD4CpU/DijwdLfBThdoY35hSpnH8KegF2A6VV0OxtDtCHz1J6oPKqrR6f nVBcgSUlqmRbhklZtmjQ1tTU9eiC09HUmJsAkonasTWs2fmlWG e4wwlGPsFWZ12udpgZz5Npy1vP4qNW4dXu/mWxYmFoEYo1rMhE9H3YcRxVo1/a5nq61NT8mzE9TvPtJiDUwrBiqxBBSZ6O2l95IU434IMCsWyOj/OvRpcNnuovS8qluXAaCtdXUG/sN7G5otT0H3oMEIIG3l0Cvl7UxKZj27uZViSvq0hIqx64N9wYT 4wwvfzZ+CtiKsSFmmAz0tbKm5jZpU0NOSlodd9xEdk2zUtScGL 0vXoCiFTJcN/fMYT6zlS+RmucmRJ0pri2oUenJ8y6eUZj6hYxQ5mY2YKzixGOP MKdRtrd5TJ/FxhQEby7OqmO4KQqunas1dJELFeLS3/f/Otjfy4xCt/cFduXZhLbwaiVSAw3hQXZaNGl0nzCBVcBXiopvXzgiktmMmTaK oS3bAG/kUQvT0xiRCfKvHr5pvWhVsAVSl/x81aLX70kqcFl5bEeQnrGpY/DsEQCbJpP3bZ7RF5dXJu6IjDP+dGeEl55C+zUwlk8TLq19ImQ6 cfTRpoeJZ4v8vogYqu7rdSxdTJGH2Tn1Sher+ivH8z2Wfe/bsWzRWRRt+ON4rveC1SakojYFziYjaH1GutGkZWpPBJggAUgAE tBOoNc9BKaUy6WtUvWoQZRzYseRu7yaWqGBo2fTBYP57/uYW78zih0TczVTMHaChaV/sEfssUMbN+eOfj/sgxlgFVrk4GRhzCdbjl02l3J+Q3VwYOgHEz3pucKbRfV85vSIJ fxd5XJrB4/QQDvgKaUoa6Ny3OwtPaYEu+sW7Y6ie061BiMJsHm7XiEm58tC5 k6yVmbdsnTyfC8sfBh+ZVWL5bvPLVQgFJbTQCuapnBc1KCpiRd YK6l0tDfhYajNxI+HWdFJuFaFqWnfnTJuvxq5sjRw8pyFkVMtg MsRztSo2hy/YWjkvmM7DtR62vsui3Bnga96sgaTbtNlJApNJcZNHamAb0/XfzYg2Hh+cU9kTAHGdnMfEjJv/mJ7wr+MSHT7KYvHP/gpZuM3Fd42zrO/WzG42/YPsB7/6x3nsPVWwLHZbsxQ5f79x2Xu7FnLArv7A2h/VeBdSAASeHsJkFqEzU9rf+Na4rjx72nCMGSyEEXJL0vWPA6L2j oObKvQllSCpI1LfqyduH3/Ildiwh9XSP5/e2ce19SV9vGb5GaHhFW2IAIqioIiKlXruGttbWuttnVpbW11tJ 2xfdvpaq2t03k/09a2M29XrXa6aEe7UVHrbrWioiCKuADKvkMgQPbk5t73hIMxQN CbADUJz/0jn8u555z7PN/D5/zu2c18qXX+ZjcuxqSqUUtCA4UoG0ZfWdDgN1ghpVXnU3+8FDZn wXjUF6U+uW7Z5yHrv3hmaPtmRTde2lVShtLqGIkUr3PrKpLL4Q xFMWTbUVNd3buc+fWEDE3RHJLXvULpthWQARAAAu5BQN3S3NmQ g/v2TJs5u0O442q/c2JCEJGSKKk4U6x38Kw1iBc0+S8rkvVpH27NQX3zpvK0dav/eUTZsUO+q9RdhHME/mGtOoEujjgiTiGlqn/7/ItriYsXTQizypC+8ESRT8qU6F7XCasBpLS3dKI19xtHEnZ13wU l9sEcLugEe1oQEwgAgesEWHdAEZJhqz7dwpdZF+B1cZFhd73+S cThA6e/23hM3UwPf3zpOOtkpZ68TIXb1n2YlbR+sXXhGGOoyvghjZr72h MubCPSk1ZBXkAACAAB7ybAXioInlh+E53AmMjAhFkLE3oPGUcY FN2fOrT20UPBsYMiIobNeuSlVyJF0J/Se8QhZyAABIAAmv7vWRD4ijmvfjzHs2wGa4EAEAACnk6A9ViFp zsK9gMBIAAEgICrBEAqXCUH6YAAEAACfYYASEWfKWpwFAgAASD gKgGQClfJQTogAASAQJ8hAFLRZ4oaHAUCQAAIuEoApMJVcpAOC AABINBnCIBU9JmiBkeBABAAAq4ScEUq0A7gu77cVWRw9Z03TWe uO/XlmuVPvXOy+fqG3DeN7tzDHszcVJO+6cWlS9bsrnKjM4EYY4vG 7IBbV+HO0YPYQAAI9FkCzi/Bo6qPbP09cN5jMW2HAFnR0dryyxU+Q+L8nc/OnjzaMXXn5wfk817518NCCTq7or6gQhAbI++ZzUHaZ96tBd4WV fb2ry5Ez1+7cZmviOxWVt39x2OMTUo1R+IjoFTleScPnpPNfXK GD8q0q/Duvg/SAwEg0CcJOFu3U7XHUmsmLF0UZE3ImJVXTp8rayg99O3OhpGzk 4iii0V0/MIXnp4RYd2jycmL0eZuefeo4o0N/X1R5oz2/MfPrr0697OPHlI4a6SDF7fP3EEE9kFUeeqb7+bc8/HCSHS8322+TEXfvfDcz3WtVsiTn1j30vSI1gPwugin1XkHfz1b fPF4Xvjy9SuTZRyqZt+Gb4VPPT+l3Ukkt9kpeD0QAAJuR8C5Wp hWnd6RE794CjplhzGU7Plo09WRK1dNi25J+0ox+/EVDymMWW8vfeurn2ZNXO3CBn7ayzvT6ZH/aDvNjiMdsuDZddyREc5Z2BXg9pl3FcuiLrlc4x8/6GYNGXPF0X2l/e63HtV62y99/r5LCS9+cF8Ej+8bHBrse31/dMfhptKf30sNWP3yU+O4z72dXWtO9lGf3JY15JGXQSdue0mCAU DAzQlwGbvrVrZSlYcO8qaOQQdWMPq8b/7+tXLa0un9hbrCrFrf4YmoimfMGq2Z6DfAz5Xq3ViZWciLG3X9 eG2CEEWmjI5o24D8Vpbd6nmnzB0lMF376rkXt9cSN9UAujkvu1 42ZLBfz3SLObKDbRijvnBQOXHB+EExsTGR/Ww6gQ4bdxiuz/vpEG/S6EAubdbTsjA5tyX7+/QBj94diQ/iZvtWiAcEgIDXE7DXBXzvTKVuuPprZvCke63by5pKDv9ex0vqh 05f0xefKuYOmhcpoJTHt3yn+tML6+9uPX2TMVVn7tm1d/fh6uTXP1iVYD2njWo4v+vH3y7mnc8p1csTH3vrtTkKAWqelB3/OXXvviMXVdzEl6NaR0AYQ+WZX779pf6utX8dKTZUZaVt/aFowpqXxugyf/7mv8dFT77/bEJrPEp14ZfN3xwqUBlNOjUv+X82PD+h/XlujjJvTejAEkvjpfOq0BmD0PHa1y8H0cwlWeVcxf2iqzvf3/xDRvOwle+8NK0fz2GGjPbq/m3fH/g9W33H2g+fGYnydcDE9i6nb2jVmbSM3OLMJT/KY8Y/9MyTUyLwOVJdhFPKy8XcqLkiDt1SVCK/c5a44JfDwY/8T5QLXYVOmwoJgAAQ8HACN/2Cbu+bsfT4lYCx0bgupy0MgT5PLYSpMvOKlqc5+um7G49w7n/v4xcm97OONGgufvu/W9UTFi8YIWhRUwyqfQ2FO9b9I2vQkufWbnhvWX+LcHhSqICwKE 9sfHefYNYzf39n2QC+IsVaT1PK06nfp/2Seso4WMFvPL/7p717047Xier2fvTBd+lnMkt9EtEJrOgyVe794P+uJD7z3uebP vjLHZEpq1aOb68TjjK3JuxsSYil9OjWj/69o4SmLuzY9OXOnEY0ralzNGSwqercNSM6d/RAcejUmQMJZU52jdlxTHPFrn+liua98t4nG15eONyqE52ZdOuf h9KR41a9sf7N1TNk+b9++M+0yrapWF2Fc0lCr9JQxrKjZxUPjt fv3yeaN0uau2f7xn++8/XJOjeax9UtKpAYCACB3iDAXirM1Wdy+QlR4lYrhLEP/33Tlr+NklL1F841R85O0ZzOKA8cFiPHrRRzaeqn6WFzkpoP726 a+cqqEVLCUnd04076ngfipRym5dLxctmoEcEko87a/FXlxPkp6Gzr2vwG36Gt3TpkUMoDD4wScsLGDPXjB4y8d/6EIB5fRIiTnvjbigkyvmL8EDlSHlp57JNtqgnTB0o4tPLUtqNM UofjtR1nTji0hC+Omvzg1ADCd8KqF1avWnb/iADSocGERZWX2+g367kXl9+bMmxgmIQTEB3IdeQaQdWk/5pVkFfSQsijBwYiLJ2ZdLM8BYpJc6YkDowbv2j1wxFE5eVaE86 wi3AydOqjk037t+8pGPjk4uCTO/UzxpZs/Kp+1Pwld3GPbD9RD1rRzfKA5EDAmwmwlgpGV3ZFExThixNwRKE xYWIOQTflnqwLmzxl4pRB9JX92Sq6lZWp7MjRWl7zwe1nQ5a+s iQRTRSy1J36tTh8apL1s19XnFEsiB+tEDCqrLSLITNH+XMIU/X5fFNE4vWBCmP56WJh/Ahr+4SgGq9cMcYvWTYrTmapyCqVJSdYj9azKDP2FAT9yTq4zKi vHK/wT46TtZu12lXmDi2x2lx9Ns8YNbZ/2xRgx9EYXcnZStHQO6zdZLS69Kom7M4xgQ0OXEOCFzrpwdGGPW +teuPHQgNa6tCZSY/9W/GkgT6C0HjU5ml/dQjnBYxd+vKLy+bNHGY88n3d5JnM7jRq6sQwkicQCsQy4e0fe+ kxIJAREAACPU2AtVSY6wuUZIC0fYVCN54/xcz88+zIgKHjo+i8A9mNVq2gW65eUIoGTVm68pGUEAGtLituar 52ukLYX4GaFLqiIzvP6cJGKPiGsowDhT7xUdbAklMZDUGJUWgt RZNSR5uqzuToo8ZECgnGQqnzz9SFpAxCk66o+twrloFJVj1Bqw BPlwuiIn2puswdH22+QMYNb5s5dR2Q9urxIgeZa1scWdLQoGm4 dE4VkjzQ2gGmbTbQGofRqguzipjocTESq1ZlHqyKnTs9wugoZm 1pNRU645WP37zX98rxPHTWeGcmzR2PHWcoE3V9+VxX946Kn1bl Zpj+9MTM8A7jTl2EU9UHtxXeMT++OSOXHKoQEbSmstZ/zJD2QuvoPRAGBIBA3yXAelib1jXqiHazKhlTTcaesmkvLxvhyy FEiSlhzA6kFdNmBtF6VNfqGxo1uoa680czahQz7g6rbDQ3N+7d 9mmakqJK9Iw454ct1bKWaq0uOGfP5h8Kqi5XWiTGC7t+MAWOmT nOnJNZR8sL9n59iY6d1D+9VJa03J9HMI0FWTWioabLB36q8h1k VJnUhf/97D8jUxKDeIIByUhX7C+qqbRKq/PplPnUKAeW1MfPni2/XG1uLkj/5ctmfeSUByaJHEUbPIK41CRL8Cf0ytz9W46FP/X89H5EzZGOrm1b89Ghy4Oe+/r9qRL/wJjpD49D2DozuS/Szl7GWLb3/bd2+j/7wcpEicnxPRJL28Xoqq6pZLERUqrh4t4f88b8beXo1rq+q3BbQ qrut29yRix9MYCTpzExYVyCqj1z2m/O060zEeACAkAACDgmwLqGoM1Go65F19pqaD7/7ftfniytrFL5xCQ0/OLz6H1JweF3Tgrbvn1/pnLa7H4hKdOHp3696Y31OZPvW/zQg2gUujk6tp9/NTdk2tOLeD+vrZg+cdZDdycIclTH8mqrZfNWrqj+sNgYHj181s QoNFPKVKQhI4YOCBl+//R4cd6/NgsS5oajlgRavF1rUnNz9TMXzQk3Z/0eEptwz+JlY5l9zzeF/KnDQAXBk8cOCeV3zpxprutsSSIaKymLGRBQSlORdz8+NlTAYRw aTGa8vcPfsP+t5won3rvwybfm9kNTjhhZJ9cWjhlLvfXp1+9/XpUwIPnplfFy1HTjdmLSrruMw+Nb1ExgDIn4dnV/owDRwvNdH721/TIdNjR+1Mwljy4fYJ1fZp045jjclhKNx//nxMDFa0LRxOaoKaPNn332jT4+bMHTE9vPB3D8rwKhQAAI9F0CH FVjg837Iwf3zbjrns4wfGVywnT1ixUv5S/e/O4MtKzC0WVRHnpjxb+r7t7w2VNx1g5/xqQ186V4/qaj+KzCGJOqRi0JDbQur2D0lQUNfoMVUlp1PvXHS2FzFoxHfVH qk+uWfR6y/otnhrZvVrDK3rlIDKXVMRLp9XVuziW+ZWyGohiSxGy7ur9lJre KwNAUzSF57WTqVmngORAAAt5KQN3S3Nm1g/v2TJ1xV4dwx9V+58SEICIlUVJxpljv4FlrEC9o8l9WJOvTPtya g/rmTeVp61b/84iyY4d8V6m7COcI/MNadQJdHHFEnEJKVf/2+RfXEhcvmhBmlSF94Ykin5Qp0b2uE1YDSGlv6URr7m06cZP7L iixD+ZwQSfY04KYQAAIXCfAugOKkAxb9ekWvsy6AK+Liwy76/VPIg4fOP3dxmPqZnr440vHWScr9eRlKty27sOspPWLrbN9GENV xg9p1NzXnnBhG5GetAryAgJAAAh4NwH2UkHwxPKb6ATGRAYmzF qY0HvIOMKg6P7UobWPHgqOHRQRMWzWIy+9EimC/pTeIw45AwEgAATQ9H/PgsBXzHn14zmeZTNYCwSAABDwdAKsxyo83VGwHwgAASAABFwlA FLhKjlIBwSAABDoMwRAKvpMUYOjQAAIAAFXCYBUuEoO0gEBIAA E+gwBkIo+U9TgKBAAAkDAVQIgFa6Sg3RAAAgAgT5DAKSizxQ1O AoEgAAQcJWAK1KBdgDf9eWuIoOr77xpOnPdqS/XLH/qnZPN1zfkvml05x72YOammvRNLy5dsmZ3FZwJ5FwhQGwgAAQ8j 4DzS/Co6iNbfw+c91hM2yFAVp9pbfnlCp8hcf7OZ2ePDO2MuvPzg012 KgAAD75JREFUA/J5r/zrYaEEnV1RX1AhiI2R98zmIO0z79YCb4sqe/tXF6Lnr924zFdEdisrz/uPAYuBABDogwScrdup2mOpNROWLgqyJmTMyiunz5U1lB76dmfD yNlJRNHFIjp+4QtPz4joeCIbC7SMNnfLu0cVb2zo74syZ7TnP3 527dW5n330kMJZIx28rH3mDiKwD6LKU998N+eejxdGouP9bv9F qwuO7j15pfBCRrZx6vp3Hx9qPdeCUp7bk3b83JljOYaEpW++On cA3k6RVucd/PVs8cXjeeHL169MlnGomn0bvhU+9fyUdieR3H6nwAIgAATciwC rTciLrhW4l9VgDRAAAkDAjsCIUWOAhwsEemETchesgCRAAAgAA SDgFQTcoQvFK0CCE0AACAAB7yXAahggZuBg7yUAngEBIODxBBx 2pHi8V+7kALQq3Kk0wBYgAASAgFsSAKlwy2IBo4AAEAAC7kQAp MKdSgNsAQJAAAi4JQGQCrcsFjAKCAABIOBOBEAq3Kk0wBYgAAS AgFsSAKlwy2IBo4AAEAAC7kQApMKdSgNsAQJAAAi4JQFSo9K4p WFgFBAAAkAACNweAp11gZT6+dweW+CtQAAIAAEg4JYEOusCdEC 5ZUGBUUAACAABdyIAUuFOpQG2AAEgAATckgBIhVsWCxgFBIAAE HAnAiAV7lQaYAsQAAJAwP0I0DTNamdZ97McLAICQAAIAIFeJ0B RVH7+ldxLOSAVvc4aXgAEgAAQ8EQC1dWVu/ek1dXWyP39QSo8sQTBZiAABIBALxIwGAzpJ35DOhEWGr5w4aPK xmqQil7EDVkDASAABDyRwLFjR35K/W7Y8MQH5i5Iz9h/tSgfhrU9sRzBZiAABIBALxI4+vuBoKDA6dNmZGUfLCrNk0gF0K roRdyQNRAAAkDAEwmIRYJ77pmn1taVV12T+4mTR6WAVHhiOYLN QAAIuE4ATf0sr6ior683mcyu5+I5KQUCfnBQcGSkgstl240UFz d4QFTUL7u3cEnLkIQosR9FbjpZPUWVuj3LPHrFMyLPcR4sBQJA AAi4RqC8vEKr0yYlJUmlPuxrT9feddtTIV3UajX5+fnI66io/iztiRscW1icr1TWKWJ9+4VJOTwLeWLza+kWGqUv+SZ3STLLfCA aEAACQMBTCdTU1iYnJ4tEYr1ej2pSDofDtF6e6k8nu5FHtjCkh cjTwYPjsrPPspeKsNDQs9k5pJCIiPDz8xNRDI/0ib17+X3azRt+s4h5nd4IAUAACAABbyNgNBolEgnWCeQbrliRZ tju0Q1SDvsK1z0R2Iy06RyWPfSLjUfhyC90SaVSg8HI3guBkMP lMnIfUUCAr49YYiZIcsWri6KPvKQj+FFJCsJYxT4viAkEgAAQ8 EQCuIa1WCy4YsUigQOtjQu75gWO4LY+IvOw8TYLsULYZA//iTxFEez9urVHjBm1RUQioZAvFvODRRwBGUmXHdxfQwmH3JVM60 7eOgeIAQSAABDwaAI2PcB1KPYFVa+t3+LWzij8i2/c2VObI9hy7AI2GLuA7lEc1Adli8nSnZYmZWjYgNr6SxyKMWqpp iYz+Z+3376olCQ/tTxFpPmNZTYQDQgAASDgoQSuj0zgr2zbt7Z9tWt7ZHvq/r5i+7HBNp1AN8jyG4/Qn+yGZBpVleGR/UVCP73eqNaU1TSoydzaoBkvvPr4SImZQsfhXXR/ImAhEAACQMBlAqiyRF/fqPbEvzYxwJ/k6E/76tXlt/zBCXGrAksCbkMgA+wFA/vLVigIwqhtNmhLFRED9drCwH5UQICA/P7rd8xGg5lCQzqwxuIPLl94HRAAAn84AabtKxu/GFev9j1OuFcKC8YfbpyLL8RKgBLb5AHdY9mzuWYVRdaTvMxIFb QlPpJQoymEoYyh/WSkTmdw0TpIBgSAABDwQAK4YrX94loVV6x4rBuHeJxn2CMer91 cVuwXbkKx94hDGCQCtEKxQmcUGXW+BCUgKXRmBeslfOzfBDGBA BAAAm5IgCHaVlGgOhRfuOsJzxpCBqNAm9n2927oCzLJvvWDrbW pHXqKFxjaPEW+I//YOBIb4usr5ZMiMdlg0ur0LQ1iclu+celQMZvEEAcIAAEg4PEEO IRtlALXocgj+5vOla+nuNxB2GydUTZFZK0UxIBIBSHw5Qh9BFJ G2ahWa4zkk8PE1nEKuIAAEAACfYCA9bu666tDX3+3edDa0szjO XVmY93l7CvNsqHTHn5kSozUuhMTrS3JTL9Y3aJWK6+evWIYsXj 1orGhwhurrFm+GssDlrcOUoH+RK0K+7EKtm0KNHAtHaznhdEcW aO6qqmliC/0RR1QLE2CaEAACAABbyBgrxS43wmF2G6whx2qXRfcpnXFv+882 jjo3gcevFPE4c1fcDX1H+/t/EIY/vrCOIHqzNatFeNXPjRRRqrPfnCuInTG6FAB0pWT4sgJLrwLJbG JHLrByoF+cd+aTS3Y51xdWV1Zf80vZNipjKMhoUGh4ZEw64k9P YgJBICANxDo0KhALmGpwDUs9rCbUsGYKg5u2lY/86UnEoVmdKQcypQTOe3BEYc/vXy+2hRp3p1WGLpyscCs12tKsit4UdP8zTXZB9Ob4h9QsBpLcF gMNpvRTQdfsMsOUzkMbLl2oanRmH/xksbEFRDGptoykAqHoCAQCAABbyXQcVgb+dm5Ju2eVFjqT+w4S kxfM4xvNFk31bBeDEX4RfoTeSYLpa2u0aglNVpLuC9Ve6nIGDy +OX1vqWLa/Jkyyki5zL0rm7F3rU/ZdkHJCGKAv0Qu5TQZCD3ByOTBIBUulwskBAJAwAMJdBrWxjUp6 quxLV6zVuzsVjU79t9cfSa9OmjGEJG5ddfu6xdl0JkJnwAJzyd mzOS7AyNImjY35F9W0ep928/d//o0iUlvcpwhi1Dcy4QiYi9ae6HQln/WYRGbg6yVgvD19ZdJ/aV6k0RvEikSq/LPgVSwKASIAgSAgLcQ6GpY21alYke7IxW0pjS/kR/aT9g+E8ZQX9YiiYnz5/HF0+dH0yajydxYmNvgN2HBsIs70jOrxk4N6db23vh1Dn+xWrBt UxCEWhLONVEGg843bLCJK6ooqWR7KJK3/J+AH0AACPR1Arje7L1fSqfSo4/71u1Dbly0tjCr0n/cJAWfoc16nd6I1rRpS3OU/SZNShw7zKcuM6fW3Hs2tebMvuCbdHR5SamGFjbUV9dXlzY2a0E q2NODmEAACHgBgRtjFajTCV9o2RqqSW1/ohtcaduHsL/nkCIeoSsvazbfSGPRlx77rXnckukhHMv1UKoxN5Oe8ugEOS84e bhUmZVdY2T/kraYNnHBf2MvOvza4rSOVbC6SnKy65RNwuD+FEUZmprEIglIBS twEAkIAAGvIWBXdd4Yk8DygHy0PUU3rrnM9R+WHExUH96b04DP 7qa1ZSf3ZIjm/vVeBdfSlilDKS9kaCYvm96PoRle6NiRPqrzWTVOn/Vtb629bHRwBD9i746qokZKCmmDXqOh1HU1IXJ/GKtgTw9iAgEg4PkE2g9r4yoVe4W1waYQLksFwQmc/OfHWral7fz3hpMxA/uH+vtHJs17LJxn0KP5UBZV7q+px4vRudXGwPhRsot+o2P9+Nzg MSN9T2SfKpkTEesCYtvUWGyzbYjbdoPCkRayblQQYXJZlI9EVX BJ2aAW8fgikgdS4UK5QBIgAAQ8mID9lzhuTOCa1FaxYv1wXSoI mpEmzH8+ZTGPsJjNFOreokxGjRZPg+X4Jz20ejzJ5fG4FrPRaN AbUT+VhRM8caoifddhel60C2TxfCdssG0eF7pB3uFf7DL7nGOj o6QMgWyP9A80qfUCCqSCPTyICQSAgOcT4FoPbbhx2TvUoT61fa q74jRt0rWYdPYpb3z5m3Xqdh1N+Il8/OOLyj5x+aU2YcM3WPbwr81b5DvL3TmkHL7ZYJSRQhFX2EiY+Tw htCpc+TeANEAACHgoAbSQGVWd1t6YTsMSuJK1VbU4wh/mJkVJRy1dY9A4dyrEDQFqHVnB2mC/nAJHuN54YjusTegMaAcpgUTMpUlfvljEEyOpcHHo5g8jCC8CAk AACPQUAVRZ4nrT9otytm9P2D7PcXhPvffW+TCUXo2WdrtYIdvM xmqBXodusCLiEHTPWigIPk9AiqVkWBhFc2VMI98EJ1XcugAhBh AAAl5EgNs2WRa7hKpR/N1ta1LgEBz4h7vtok7YbLZvGGG1wC60aSHqgWJ3kRKJ0NeX8Q+ o1Gjr1Fp0Tip0QLEjB7GAABDwFgK43rSvVe2rWptI3Ca1cI6yv bXII+wIygLdYAfxTZtUsM6bEorQtiDNNXW1ZRV8rcXHB8YqWLO DiEAACHgBAU7rKXi23idc1Vo7ZzjWL+4OOmGrcN3TcZu1HczDZ ttGLNBT7C/ynaUjetQTZiEMGrWQL5CHylAHFLQqWKKDaEAACHgJAZFIpNVox BKJyWTCn97oPGr8Se5ZHtrb3JX9SDb4fL5WqxWLRey943G5epK 0UPoAXz+6Ra8DqWDPDmICASDgDQQ4TExMdF5e3uC4OKlU2tWHu Td42uoDkhCdVpuXnx8dHY0aVCz9IvkiBu1SZbYQeiOtoywcAbQ qWKKDaEAACHgDAbS9U3JyclZWVmbmGZ1O7w0u3coHiUQcGxuLv Ea+3ypu23NKozdq9YRUyufyBVxGQoBUsCQH0YAAEPAKAhYKrTw mx90xDl1e4RA7JxjCQqNF42zPTdI2tKD15AIfHm9gf05dk9DAg VYFO9AQCwgAAW8hgGpMC8G20vQWp53zgyS4IlLEoTgNVbUaC63 jkCAVzhGE2EAACAABrycgCw3mc0iawxOZjUqdlvKVglR4faGDg 0AACAAB5wgYI4L5AhGXy5FRTARa9C2EdRXOAYTYQAAIAAHvJ3C uvFgu87PuSovO6qMoNKsYWhXeX+rgIRAAAkDAKQLykHCGotC6R JpDk2Kx3A+ONnKKH0QGAkAACPQBAsNG34G3GsS+ouYFtCr6QLG Di0AACAABJwngfUFsieBsbSf5QXQgAASAQN8jAFLR98ocPAYCQ AAIOEkApMJJYBAdCAABIND3CIBU9L0yB4+BABAAAk4SIPEevE6 mguhAAAgAASDgtQQ660LHGVAH9+3xWu/BMSAABIAAEHCJAKdBWe9SQndJdPTwgRl33eMu1oAdQAAI9AEC6 JN68rSZbu6orlknkUt6ykgYq+gpkpAPEAACQMBrCYBUeG3RgmN AAAgAgZ4iAFLRUyQhHyAABICA1xIAqfDaogXHgAAQAAI9RQCko qdIQj5AAAgAAa8lAFLhtUULjgEBIAAEeooASEVPkYR8gAAQAAJ eSwCkwmuLFhwDAkAACPQUgf8HHObD2mB+RuIAAAAASUVORK5CY II=

installArchives() failed: (Reading database ...
(Reading database ... 5%
(Reading database ... 10%
(Reading database ... 15%
(Reading database ... 20%
(Reading database ... 25%
(Reading database ... 30%
(Reading database ... 35%
(Reading database ... 40%
(Reading database ... 45%
(Reading database ... 50%
(Reading database ... 55%
(Reading database ... 60%
(Reading database ... 65%
(Reading database ... 70%
(Reading database ... 75%
(Reading database ... 80%
(Reading database ... 85%
(Reading database ... 90%
(Reading database ... 95%
(Reading database ... 100%
(Reading database ... 233395 files and directories currently installed.)
Unpacking fglrx (from .../fglrx_2%3a8.960-0ubuntu1.1_amd64.deb) ...
One or more files have been altered since installation.
Uninstall will not be completed. See /etc/ati/fglrx-uninstall.log for details.
dpkg: error processing /var/cache/apt/archives/fglrx_2%3a8.960-0ubuntu1.1_amd64.deb (--unpack):
subprocess new pre-installation script returned error exit status 1
Errors were encountered while processing:
/var/cache/apt/archives/fglrx_2%3a8.960-0ubuntu1.1_amd64.deb
Error in function:
SystemError: E:Sub-process /usr/bin/dpkg returned an error code (1)
dpkg: dependency problems prevent configuration of fglrx-amdcccle:
fglrx-amdcccle depends on fglrx; however:
Package fglrx is not installed.
dpkg: error processing fglrx-amdcccle (--configure):
dependency problems - leaving unconfigured

myrrdinoftheforest
January 23rd, 2013, 04:19 AM
Ok, so I just fixed it. I had to force uninstall the ati driver by going to /user/share/ati and as root ran fglrx-uninstall.sh with --force in front of the normal ./fglrx-uninstall.sh

This forced the uninstall of the previous ati driver, and allowed me to reinstall and when I opened the software centre and clicked repair, it went smooth as silk :KS

tgalati4
January 23rd, 2013, 04:35 AM
Note to self: Any updates to the kernel may mess up your proprietary graphics driver. Either don't upgrade the kernel, or plan for some down time to fix issues after upgrading the kernel. This applies to any proprietary driver: graphics, wireless, etc.

myrrdinoftheforest
January 23rd, 2013, 06:19 AM
Note to self: Any updates to the kernel may mess up your proprietary graphics driver. Either don't upgrade the kernel, or plan for some down time to fix issues after upgrading the kernel. This applies to any proprietary driver: graphics, wireless, etc.

Wouldn't it be cool if there was a pop up warning saying "hey there sparky, if you install this update, be prepared to spend some time fixing your proprietary drivers"... I wonder, has there ever been an ubuntu kaizen event for the update process?? :-P

Landon1976
February 1st, 2013, 08:42 PM
Wouldn't it be cool if there was a pop up warning saying "hey there sparky, if you install this update, be prepared to spend some time fixing your proprietary drivers"... I wonder, has there ever been an ubuntu kaizen event for the update process?? :-P


You should add this to the Ubuntu Brainstorm. This is a interesting topic that I think many people will promote. :)

Dobly
February 2nd, 2013, 07:05 AM
I so agree with the idea of a some warning.. Last night an 'update' (if you can call it that) totally ruined a perfectly good install of 12.10 which was running the AMD 13.1 catalyst drivers.

Took me about 2 hours for the complete backup and reinstall.

Next time the little update thing flashes at me, I'll be very careful before accepting.